Posts

Daily Prelims CA Quiz

UPSC Quiz – 2024 : IASbaba’s Daily Current Affairs Quiz 10th April 2024

For Previous Daily Quiz (ARCHIVES) – CLICK HERE The Current Affairs questions are based on sources like ‘The Hindu’, ‘Indian Express’ and ‘PIB’, which are very important sources for UPSC Prelims Exam. The questions are focused on both the concepts and facts. The topics covered here are generally different from what is being covered under ‘Daily Current Affairs/Daily News Analysis (DNA) and Daily Static Quiz’ to avoid duplication. The questions would be published from Monday to Saturday before 2 PM. One should not spend more than 10 minutes on this initiative. Gear up and Make the Best Use of this initiative. Do remember that, “the difference between Ordinary and EXTRA-Ordinary is PRACTICE!!” Important Note: Don’t forget to post your marks in the comment section. Also, let us know if you enjoyed today’s test 🙂After completing the 5 questions, click on ‘View Questions’ to check your score, time taken, and solutions. To take the Test Click Here

[DAY 32] 60 DAY RAPID REVISION (RaRe) SERIES for UPSC Prelims 2024 – GEOGRAPHY, CURRENT AFFAIRS & CSAT TEST SERIES!

Archives Hello Friends The 60 Days Rapid Revision (RaRe) Series is IASbaba’s Flagship Initiative recommended by Toppers and loved by the aspirants’ community every year. It is the most comprehensive program which will help you complete the syllabus, revise and practice tests on a daily basis. The Programme on a daily basis includes Daily Prelims MCQs from Static (Monday – Saturday) Daily Static Quiz will cover all the topics of static subjects – Polity, History, Geography, Economics, Environment and Science and technology. 20 questions will be posted daily and these questions are framed from the topics mentioned in the schedule. It will ensure timely and streamlined revision of your static subjects. Daily Current Affairs MCQs (Monday – Saturday) Daily 5 Current Affairs questions, based on sources like ‘The Hindu’, ‘Indian Express’ and ‘PIB’, would be published from Monday to Saturday according to the schedule. Daily CSAT Quiz (Monday – Friday) CSAT has been an Achilles heel for many aspirants. Daily 5 CSAT Questions will be published. Note – Daily Test of 20 static questions, 10 current affairs, and 5 CSAT questions. (35 Prelims Questions) in QUIZ FORMAT will be updated on a daily basis. To Know More about 60 Days Rapid Revision (RaRe) Series – CLICK HERE   60 Day Rapid Revision (RaRe) Series Schedule – CLICK HERE  Important Note Comment your Scores in the Comment Section. This will keep you accountable, responsible and sincere in days to come. It will help us come out with the Cut-Off on a Daily Basis. Let us know if you enjoyed today’s test 🙂  You can post your comments in the given format  (1) Your Score (2) Matrix Meter (3) New Learning from the Test Time limit: 0 Test-summary 0 of 35 questions completed Questions: 1 2 3 4 5 6 7 8 9 10 11 12 13 14 15 16 17 18 19 20 21 22 23 24 25 26 27 28 29 30 31 32 33 34 35 Information The following Test is based on the syllabus of 60 Days Plan-2023 for UPSC IAS Prelims 2022. To view Solutions, follow these instructions: Click on – ‘Start Test’ button Solve Questions Click on ‘Test Summary’ button Click on ‘Finish Test’ button Now click on ‘View Questions’ button – here you will see solutions and links. You have already completed the test before. Hence you can not start it again. Test is loading... You must sign in or sign up to start the test. You have to finish following test, to start this test: Results 0 of 35 questions answered correctly Your time: Time has elapsed You have scored 0 points out of 0 points, (0) Average score     Your score     Categories Not categorized 0% Your result has been entered into leaderboard Loading Name: E-Mail: Captcha: maximum of 70 points Pos. Name Entered on Points Result Table is loading No data available 1 2 3 4 5 6 7 8 9 10 11 12 13 14 15 16 17 18 19 20 21 22 23 24 25 26 27 28 29 30 31 32 33 34 35 Answered Review Question 1 of 35 1. Question Consider the following statements with respect to the Five-Year Planning (FYP) phase: Implementation of Family Planning Programs was amongst the major targets of the seventh FYP. The ninth FYP for the first time had a separate plan document on states. The concept of Inclusive growth was introduced for the first time in the eighth FYP. Select the correct answer using the code given below: a) Only one b) Only two c) All three d) None Correct Solution (d) Statement 1 Statement 2 Statement 3 Incorrect Incorrect Incorrect The objective of fourth FYP (1969-74) includes “growth with stability and progressive achievement of self- reliance”. Implementation of Family Planning Programs was amongst the major targets of the Plan. The tenth FYP (2002 – 2007) sought to double national per capita income and create a hundred million jobs in 10 years. The 10th FYP emphasized regional imbalance and for the first time had a separate plan document on states. The eleventh plan (2007 – 2012) introduced the concept of inclusive growth. It focused on human resources, especially health and skill development. Incorrect Solution (d) Statement 1 Statement 2 Statement 3 Incorrect Incorrect Incorrect The objective of fourth FYP (1969-74) includes “growth with stability and progressive achievement of self- reliance”. Implementation of Family Planning Programs was amongst the major targets of the Plan. The tenth FYP (2002 – 2007) sought to double national per capita income and create a hundred million jobs in 10 years. The 10th FYP emphasized regional imbalance and for the first time had a separate plan document on states. The eleventh plan (2007 – 2012) introduced the concept of inclusive growth. It focused on human resources, especially health and skill development. Question 2 of 35 2. Question Which of the following expenditures are considered as Non-Developmental expenditure? Expenditure on Administration Maintenance of law and order Cost of tax collection Select the correct answer using the code given below. a) 1 and 2 only b) 2 and 3 only c) 1 and 3 only d) 1, 2 and 3 Correct Solution (d) Statement 1 Statement 2 Statement 3 Correct Correct Correct Non-developmental expenditure includes expenditures made for administrative service, defence service, debt servicing, subsidies, etc. Non-developmental expenditure includes expenditure made on maintenance of law and order. Cost of tax collection, cost of an audit, printing of notes, internal law and order are also included under Non-development expenditure. Incorrect Solution (d) Statement 1 Statement 2 Statement 3 Correct Correct Correct Non-developmental expenditure includes expenditures made for administrative service, defence service, debt servicing, subsidies, etc. Non-developmental expenditure includes expenditure made on maintenance of law and order. Cost of tax collection, cost of an audit, printing of notes, internal law and order are also included under Non-development expenditure. Question 3 of 35 3. Question Consider the following pairs: Deficit                                                 Description Effective Revenue Deficit:: Fiscal deficit – Interest payments Primary Deficit::            Revenue Deficit – Grants for the creation of capital assets Monetized Deficit::            Government deficit financed by borrowing from the RBI How many of the above pairs is/are correctly matched? a) None of the pairs b) One pair only c) Two pairs only d) Three pairs only Correct Solution (b) Statement 1 Statement 2 Statement 3 Incorrect Incorrect Correct Revenue deficit is the difference between revenue receipts and revenue expenditure. Effective Revenue deficit is a new term introduced in the Union Budget 2011-12.   Effective Revenue Deficit = Revenue Deficit – Grants for the creation of capital assets. Primary Deficit is the difference between the current year’s fiscal deficit (Total income– Total expenditure of the government) and the interest paid on the borrowings of the previous year.   Primary Deficit = Fiscal deficit – Interest payments Monetized deficit is also known as the ‘net reserve bank credit to the government’. It is that part of the government deficit which is financed solely by borrowing from the RBI Incorrect Solution (b) Statement 1 Statement 2 Statement 3 Incorrect Incorrect Correct Revenue deficit is the difference between revenue receipts and revenue expenditure. Effective Revenue deficit is a new term introduced in the Union Budget 2011-12.   Effective Revenue Deficit = Revenue Deficit – Grants for the creation of capital assets. Primary Deficit is the difference between the current year’s fiscal deficit (Total income– Total expenditure of the government) and the interest paid on the borrowings of the previous year.   Primary Deficit = Fiscal deficit – Interest payments Monetized deficit is also known as the ‘net reserve bank credit to the government’. It is that part of the government deficit which is financed solely by borrowing from the RBI Question 4 of 35 4. Question With reference to the First Five-Year Plan in India, consider the following statement: It focused on rapid industrialization using heavy and basic industries. The actual growth rate exceeded the target growth rate set by the five-year plan. Choose the correct code: a) 1 only b) 2 only c) Both 1 and 2 d) Neither 1 nor 2 Correct Solution (b) Statement 1 Statement 2 Incorrect Correct An influx of refugees, severe food shortage & mounting inflation confronted the country at the onset of the First Five-year Plan. The Plan Focused on agriculture, price stability, power, and transport. The First Five-Year Plan (1951-56) kept the Growth rate target of 2.1%. Whereas, the actual Growth rate was 3.6%.   Incorrect Solution (b) Statement 1 Statement 2 Incorrect Correct An influx of refugees, severe food shortage & mounting inflation confronted the country at the onset of the First Five-year Plan. The Plan Focused on agriculture, price stability, power, and transport. The First Five-Year Plan (1951-56) kept the Growth rate target of 2.1%. Whereas, the actual Growth rate was 3.6%.   Question 5 of 35 5. Question Which of the following can lead to the condition of ‘Fiscal Slippage’ in an economy? Farm loan waivers Decline in tax collection Increase in disinvestment proceeds How many of the above statements are correct? a) Only one b) Only two c) All three d) None Correct Solution (b) Statement 1 Statement 2 Statement 3 Correct Correct Incorrect Fiscal slippage in simple terms is any deviation in expenditure from the expected. When the Government’s expenditure surpasses the expected or estimated levels, the nation might face the threat of fiscal slippage. Farm loan waivers increase the expenditure that will lead to the condition of fiscal slippage. Tax rate cuts lead to a decline in tax revenue that will lead to the condition of fiscal slippage.   Increase in disinvestment proceed rather prevents (not lead to) the situation of fiscal slippage Incorrect Solution (b) Statement 1 Statement 2 Statement 3 Correct Correct Incorrect Fiscal slippage in simple terms is any deviation in expenditure from the expected. When the Government’s expenditure surpasses the expected or estimated levels, the nation might face the threat of fiscal slippage. Farm loan waivers increase the expenditure that will lead to the condition of fiscal slippage. Tax rate cuts lead to a decline in tax revenue that will lead to the condition of fiscal slippage.   Increase in disinvestment proceed rather prevents (not lead to) the situation of fiscal slippage Question 6 of 35 6. Question With reference to Revenue Deficit, consider the following statements: It includes such transactions that affect the current income and expenditure of the government. It signifies that government is using up the savings to finance a part of its consumption expenditure. Which of the statements given above is/are correct? a) 1 only b) 2 only c) Both 1 and 2 d) Neither 1 nor 2 Correct Solution (c) Statement 1 Statement 2 Correct Correct The revenue deficit includes only such transactions that affect the current income and expenditure of the government. When the government incurs a revenue deficit, it implies that the government is dissaving and is using up the savings of the other sectors of the economy to finance a part of its consumption expenditure.   This situation means that the government will have to borrow not only to finance its investment but also its consumption requirements. This will lead to a build-up of stock of debt and interest liabilities and force the government, eventually, to cut expenditure. Since a major part of revenue expenditure is committed expenditure, it cannot be reduced. Often the government reduces productive capital expenditure or welfare expenditure. This would mean lower growth and adverse welfare implications. Incorrect Solution (c) Statement 1 Statement 2 Correct Correct The revenue deficit includes only such transactions that affect the current income and expenditure of the government. When the government incurs a revenue deficit, it implies that the government is dissaving and is using up the savings of the other sectors of the economy to finance a part of its consumption expenditure.   This situation means that the government will have to borrow not only to finance its investment but also its consumption requirements. This will lead to a build-up of stock of debt and interest liabilities and force the government, eventually, to cut expenditure. Since a major part of revenue expenditure is committed expenditure, it cannot be reduced. Often the government reduces productive capital expenditure or welfare expenditure. This would mean lower growth and adverse welfare implications. Question 7 of 35 7. Question Consider the following statements regarding ‘Short Term Capital Gains Tax’ in India: It refers to a tax on the gain that arises from the sale of an asset after holding it less than sixty months. Securities Transaction Tax (STT) is an indirect tax levied on sale and purchase of equities. Select the correct answer using the code given below: a) 1 only b) 2 only c) Both 1 and 2 d) Neither 1 nor 2 Correct Solution (d) Statement 1 Statement 2 Incorrect Incorrect Short Term Capital Gains Tax (STCG) is the tax levied on profits generated from the sale of an asset which is held for a short-term period which differs for various items – for example it is 36 months or less in the case of immovable property such as land and building and 12 months or less for equity, bonds, government securities etc. The Securities Transaction Tax (STT) is a type of direct tax which is levied at the time of purchase and sale of securities listed on stock exchanges in India. Securities are tradable investment instruments such as shares bonds, debentures, equity-oriented mutual funds (MFs) and so on and are issued either by companies or by the Indian government. This tax was introduced in the Union Budget of 2004. Incorrect Solution (d) Statement 1 Statement 2 Incorrect Incorrect Short Term Capital Gains Tax (STCG) is the tax levied on profits generated from the sale of an asset which is held for a short-term period which differs for various items – for example it is 36 months or less in the case of immovable property such as land and building and 12 months or less for equity, bonds, government securities etc. The Securities Transaction Tax (STT) is a type of direct tax which is levied at the time of purchase and sale of securities listed on stock exchanges in India. Securities are tradable investment instruments such as shares bonds, debentures, equity-oriented mutual funds (MFs) and so on and are issued either by companies or by the Indian government. This tax was introduced in the Union Budget of 2004. Question 8 of 35 8. Question Which of the following statements is correct with respect to ‘General Anti-Avoidance Rule (GAAR)’? a) It is designed to prevent tax avoidance in cross border transactions only. b) Its provisions are included in the Income Tax Act, 1961. c) Its provisions are effective from January 2019. d) As of now, there are no appellate tribunals to appeal against the decisions of tax authorities regarding GAAR provisions. Correct Solution (b) GAAR is an anti-tax avoidance law under Chapter X-A of the Income Tax Act, 1961 of India. It is framed by the Department of Revenue under the Ministry of Finance. GAAR applies to any arrangement that is considered an Impermissible Avoidance Arrangement (IAA). Furthermore, under its provisions, certain transactions are deemed to lack commercial substance. Once the Revenue authorities decide to treat an arrangement as an IAA, the onus to prove otherwise is on taxpayers. Consequently, they are required to substantiate the commercial reasons for such arrangements and that availing tax benefit was not the main purpose for these transactions. Under the Income Tax act, a taxpayer has the right to appeal to the Income Tax Appellate Tribunal (ITAT) against an order passed by the Revenue authorities along with the direction of the Approving Panel. Incorrect Solution (b) GAAR is an anti-tax avoidance law under Chapter X-A of the Income Tax Act, 1961 of India. It is framed by the Department of Revenue under the Ministry of Finance. GAAR applies to any arrangement that is considered an Impermissible Avoidance Arrangement (IAA). Furthermore, under its provisions, certain transactions are deemed to lack commercial substance. Once the Revenue authorities decide to treat an arrangement as an IAA, the onus to prove otherwise is on taxpayers. Consequently, they are required to substantiate the commercial reasons for such arrangements and that availing tax benefit was not the main purpose for these transactions. Under the Income Tax act, a taxpayer has the right to appeal to the Income Tax Appellate Tribunal (ITAT) against an order passed by the Revenue authorities along with the direction of the Approving Panel. Question 9 of 35 9. Question In the context of Indian economy, what is ‘Economic Capital Framework’? a) It is a part of the annual budget documents, intended to be placed before the parliament, outlining the various strategies for capital generation. b) It is a framework developed by the RBI to determine the appropriate level of risk provisions and distribution of profits between the RBI and the Union Government. c) It is the annual submission of RBI under the Basel Committee outlining the former's commitments. d) It is the framework developed by the NITI Aayog that guides the sharing of revenues between the Centre and the states. Correct Solution (b) The economic capital framework provides a methodology for determining the appropriate level of risk provisions and profit distribution to be made under Section 47 of the RBI Act, 1934. As per this provision, the central bank is required to pay balance of its profits to the central government after making provision for bad and doubtful debts, depreciation in assets, and contributions to staff. Incorrect Solution (b) The economic capital framework provides a methodology for determining the appropriate level of risk provisions and profit distribution to be made under Section 47 of the RBI Act, 1934. As per this provision, the central bank is required to pay balance of its profits to the central government after making provision for bad and doubtful debts, depreciation in assets, and contributions to staff. Question 10 of 35 10. Question Which of the following fiscal policy statements are required to be laid before the Parliament under the Fiscal Responsibility and Budget Management Act, 2003 (FRBMA)? Medium-term Fiscal Policy Fiscal Policy Strategy Outcome Budget Medium-term Expenditure Framework Select the correct answer using the code given below. a) 1 and 3 only b) 1, 2 and 4 only c) 2 and 4 only d) 1, 2, 3 and 4 Correct Solution (b) Statement 1 Statement 2 Statement 3 Statement 4 Correct Correct Incorrect Correct The FRBM Act includes the Medium Term Fiscal Policy. It sets out the three-year rolling targets for five specific fiscal indicators in relation to GDP at market prices, namely (i) Revenue Deficit, (ii) Fiscal Deficit, (iii) Effective Revenue Deficit (iv) Tax to GDP ratio and (v) Total outstanding Central Government Debt at the end of the year. The Fiscal Policy Strategy Statement, presented to Parliament as per the FRBM Act. The Strategy Statement outlines the strategic priorities of Government in the fiscal area for the ensuing financial year relating to taxation, expenditure, lending and investments, administered pricing, borrowings and guarantees. The Statement explains how the current policies are in conformity with sound fiscal management principles and gives the rationale for any major deviation in key fiscal measures. The introduction of Outcome Budget is an executive action by the government. From the year 2017-18 onwards, it has been decided that the output and outcomes of the schemes of 68 Ministries and Departments will be available along with the financial outlays as a part of the Budget documents, so that clearly defined objectives and goals for each scheme can be seen by all. The medium-term expenditure framework (MTEF) statement sets a three-year rolling target for expenditure indicators, along with specifications of underpinning assumptions and risks. This statement is presented in Parliament under Section 3 of the FRBM Act, 2003.   The statement provides an estimate of expenditure for various sectors, including education, health, rural development, energy, subsidies and pension, and so on. Incorrect Solution (b) Statement 1 Statement 2 Statement 3 Statement 4 Correct Correct Incorrect Correct The FRBM Act includes the Medium Term Fiscal Policy. It sets out the three-year rolling targets for five specific fiscal indicators in relation to GDP at market prices, namely (i) Revenue Deficit, (ii) Fiscal Deficit, (iii) Effective Revenue Deficit (iv) Tax to GDP ratio and (v) Total outstanding Central Government Debt at the end of the year. The Fiscal Policy Strategy Statement, presented to Parliament as per the FRBM Act. The Strategy Statement outlines the strategic priorities of Government in the fiscal area for the ensuing financial year relating to taxation, expenditure, lending and investments, administered pricing, borrowings and guarantees. The Statement explains how the current policies are in conformity with sound fiscal management principles and gives the rationale for any major deviation in key fiscal measures. The introduction of Outcome Budget is an executive action by the government. From the year 2017-18 onwards, it has been decided that the output and outcomes of the schemes of 68 Ministries and Departments will be available along with the financial outlays as a part of the Budget documents, so that clearly defined objectives and goals for each scheme can be seen by all. The medium-term expenditure framework (MTEF) statement sets a three-year rolling target for expenditure indicators, along with specifications of underpinning assumptions and risks. This statement is presented in Parliament under Section 3 of the FRBM Act, 2003.   The statement provides an estimate of expenditure for various sectors, including education, health, rural development, energy, subsidies and pension, and so on. Question 11 of 35 11. Question Which of the following actions implemented by the government may lead to reduction in its deficit? Increase in the rate of personal Income Tax. Sale of shares of Public Sector Undertakings. Introduction of new welfare schemes by the government. How many of the above statements are correct? a) Only one b) Only two c) All three d) None Correct Solution (b) Statement 1 Statement 2 Statement 3 Correct Correct Incorrect Government deficit can be reduced by an increase in taxes or reduction in expenditure. In India, the government has been trying to increase tax revenue with greater reliance on direct taxes. There has also been an attempt to raise receipts through the sale of shares in PSUs. The other way is to change the scope of the government by withdrawing from some of the areas where it operated before. Cutting back government programmes in vital areas like agriculture, education, health, poverty alleviation, etc. would adversely affect the economy. Incorrect Solution (b) Statement 1 Statement 2 Statement 3 Correct Correct Incorrect Government deficit can be reduced by an increase in taxes or reduction in expenditure. In India, the government has been trying to increase tax revenue with greater reliance on direct taxes. There has also been an attempt to raise receipts through the sale of shares in PSUs. The other way is to change the scope of the government by withdrawing from some of the areas where it operated before. Cutting back government programmes in vital areas like agriculture, education, health, poverty alleviation, etc. would adversely affect the economy. Question 12 of 35 12. Question The government of a nation performs its income distribution function through which of the following mechanisms? Levying taxes Transfer Payment Controlling money supply How many of the above statements are correct? a) Only one b) Only two c) All three d) None Correct Solution (b) Statement 1 Statement 2 Statement 3 Correct Correct Incorrect Through its tax and expenditure policy, the government attempts to bring about a distribution of income that is considered, fair by society. The government affects the personal disposable income of households by making transfer payments and collecting taxes and, therefore, can alter the income distribution. This is the distribution function. Transfer Payment is a payment made or income received in which no goods or services are being paid for, such as a benefit payment or subsidy. Unlike the exchange transaction which mutually benefits all the parties involved in it, the transfer payment consists of a donor and a recipient, with the donor giving up something of value without receiving anything in return. In any period, the level of expenditures may not be sufficient for full utilisation of labour and other resources of the economy. Since wages and prices are generally rigid downwards (they do not fall below a level), employment cannot be restored automatically. Hence, policy measures are needed to raise aggregate demand. Incorrect Solution (b) Statement 1 Statement 2 Statement 3 Correct Correct Incorrect Through its tax and expenditure policy, the government attempts to bring about a distribution of income that is considered, fair by society. The government affects the personal disposable income of households by making transfer payments and collecting taxes and, therefore, can alter the income distribution. This is the distribution function. Transfer Payment is a payment made or income received in which no goods or services are being paid for, such as a benefit payment or subsidy. Unlike the exchange transaction which mutually benefits all the parties involved in it, the transfer payment consists of a donor and a recipient, with the donor giving up something of value without receiving anything in return. In any period, the level of expenditures may not be sufficient for full utilisation of labour and other resources of the economy. Since wages and prices are generally rigid downwards (they do not fall below a level), employment cannot be restored automatically. Hence, policy measures are needed to raise aggregate demand. Question 13 of 35 13. Question Which of the following is/are included in Public Debt of India? Treasury bills Outstanding external debts Borrowing from small savings How many of the above statements are correct? a) Only one b) Only two c) All three d) None Correct Solution (b) Statement 1 Statement 2 Statement 3 Correct Correct Incorrect In India, public debt refers to a part of the total borrowings by the Union Government which includes such items as market loans, special bearer bonds, treasury bills and special loans and securities issued by the Reserve Bank. It also includes the outstanding external debt. At end-March 2021, India’s external debt was placed at US$ 570.0 billion Public debt does not include the following items of borrowings: ·       small savings, ·       provident funds, ·       other accounts, reserve funds and deposits. Incorrect Solution (b) Statement 1 Statement 2 Statement 3 Correct Correct Incorrect In India, public debt refers to a part of the total borrowings by the Union Government which includes such items as market loans, special bearer bonds, treasury bills and special loans and securities issued by the Reserve Bank. It also includes the outstanding external debt. At end-March 2021, India’s external debt was placed at US$ 570.0 billion Public debt does not include the following items of borrowings: ·       small savings, ·       provident funds, ·       other accounts, reserve funds and deposits. Question 14 of 35 14. Question With reference to Primary Deficit, consider the following statements: The primary deficit is the sum of Fiscal deficit and interest payments. It includes the burden of the past debt. Which of the statements given above is/are correct? a) 1 only b) 2 only c) Both 1 and 2 d) Neither 1 nor 2 Correct Solution (d) Statement 1 Statement 2 Incorrect Incorrect Gross Primary deficit is defined as gross fiscal deficit minus net interest payments. Net primary deficit, is gross primary deficit minus net domestic lending. It excludes the burden of the past debt and shows the net increase in the government’s indebtedness due to the current year’s fiscal operations. A reduction in primary deficit is reflective of government’s efforts at bridging the fiscal gap during a financial year. Incorrect Solution (d) Statement 1 Statement 2 Incorrect Incorrect Gross Primary deficit is defined as gross fiscal deficit minus net interest payments. Net primary deficit, is gross primary deficit minus net domestic lending. It excludes the burden of the past debt and shows the net increase in the government’s indebtedness due to the current year’s fiscal operations. A reduction in primary deficit is reflective of government’s efforts at bridging the fiscal gap during a financial year. Question 15 of 35 15. Question With reference to Minimum Alternate Tax, consider the following statements: It is a direct tax imposed under Income Tax Act, 1961. It does not apply to any income accruing or arising to a company from life insurance business. It is not applicable to public corporate entities. How many of the statements given above is/are correct? a) Only one b) Only two c) All three d) None Correct Solution (b) Statement 1 Statement 2 Statement 3 Correct Correct Incorrect Minimum Alternate Tax (MAT) is a tax effectively introduced in India by the Finance Act of 1987, vide Section 115J of the Income Tax Act, 1961 (IT Act), to facilitate the taxation of ‘zero tax companies’ i.e., those companies which show zero or negligible income to avoid tax. It is a direct tax. Under MAT, such companies are made liable to pay to the government, by deeming a certain percentage of their book profit as taxable income. However, it does not apply to any income accruing or arising to a company from life insurance business. The applicability of MAT extends to all types of companies, including a private company, one-person company, public limited company and Nidhi company. Incorrect Solution (b) Statement 1 Statement 2 Statement 3 Correct Correct Incorrect Minimum Alternate Tax (MAT) is a tax effectively introduced in India by the Finance Act of 1987, vide Section 115J of the Income Tax Act, 1961 (IT Act), to facilitate the taxation of ‘zero tax companies’ i.e., those companies which show zero or negligible income to avoid tax. It is a direct tax. Under MAT, such companies are made liable to pay to the government, by deeming a certain percentage of their book profit as taxable income. However, it does not apply to any income accruing or arising to a company from life insurance business. The applicability of MAT extends to all types of companies, including a private company, one-person company, public limited company and Nidhi company. Question 16 of 35 16. Question In the context of the Indian Economy, what are Ways and Means advances? a) These are part of the infrastructure projects financing mechanism by the National Investment and Infrastructure Fund (NIIF). b) These are loans advanced by the World Bank to developing economies. c) These are temporary loans advanced by RBI to central and state governments. d) These are loans advanced by the IMF to overcome the medium-term balance of payments crisis. Correct Solution (c) The Reserve Bank of India gives temporary loan facilities to the centre and state governments as a banker to the This temporary loan facility is called Ways and Means Advances (WMA). This facility is provided to help them tide over temporary mismatches in the cash flow of their receipts and This is guided under Section 17(5) of the RBI Act, 1934.   Incorrect Solution (c) The Reserve Bank of India gives temporary loan facilities to the centre and state governments as a banker to the This temporary loan facility is called Ways and Means Advances (WMA). This facility is provided to help them tide over temporary mismatches in the cash flow of their receipts and This is guided under Section 17(5) of the RBI Act, 1934.   Question 17 of 35 17. Question Consider the following statements regarding the differences between Proportional and Progressive Taxation: In progressive taxation, tax rate increases with an increase in the income whereas, in proportional taxation same percentage of tax is levied on all taxpayers. As compared to progressive taxation, proportional taxation makes disposable income less sensitive to fluctuations in GDP. Which of the statements given above is/are correct? a) 1 only b) 2 only c) Both 1 and 2 d) Neither 1 nor 2 Correct Solution (c) Statement 1 Statement 2 Correct Correct A proportional tax is an income tax system where the same percentage of tax is levied on all taxpayers, regardless of their income. A proportional tax applies the same tax rate across low, middle, and high-income taxpayers. Whereas, progressive taxation is based on the taxable amount of an individual’s income. They follow an accelerating schedule, so high-income earners pay more than low-income earners. The proportional income tax acts as an automatic stabiliser – a shock absorber because it makes disposable income, and thus consumer spending, less sensitive to fluctuations in GDP as compared to  progressive taxation. When GDP rises, disposable income also rises but by less than the rise in GDP because a part of it is siphoned off as taxes. This helps limit the upward fluctuation in consumption spending. Incorrect Solution (c) Statement 1 Statement 2 Correct Correct A proportional tax is an income tax system where the same percentage of tax is levied on all taxpayers, regardless of their income. A proportional tax applies the same tax rate across low, middle, and high-income taxpayers. Whereas, progressive taxation is based on the taxable amount of an individual’s income. They follow an accelerating schedule, so high-income earners pay more than low-income earners. The proportional income tax acts as an automatic stabiliser – a shock absorber because it makes disposable income, and thus consumer spending, less sensitive to fluctuations in GDP as compared to  progressive taxation. When GDP rises, disposable income also rises but by less than the rise in GDP because a part of it is siphoned off as taxes. This helps limit the upward fluctuation in consumption spending. Question 18 of 35 18. Question Which of the following are the quantitative methods of credit control in India? Regulation of consumer credit Rationing Credit Discount Rate Policy Open Market Operations Select the correct answer using the code given below: a) 1 and 2 only b) 1, 2 and 3 only c) 3 and 4 only d) 1, 2, 3 and 4 Correct Solution (c) Statement 1 Statement 2 Statement 3 Statement 4 Incorrect Incorrect Correct Correct Regulation of consumer credit is a Qualitative method of money control. Rationing Credit credit is a Qualitative method of money control. Discount Rate Policy points to the rate of interest that is going to be paid by the banks and other financial institutions on the credits that they avail from the central bank through discount window loan procedure. This will infuse more liquidity in the market. Open Market Operations refers to buying and selling of bonds issued by the Government in the open market. When thes central bank want to infuse liquidity in the monetary system, it will buy government securities in the open market. Incorrect Solution (c) Statement 1 Statement 2 Statement 3 Statement 4 Incorrect Incorrect Correct Correct Regulation of consumer credit is a Qualitative method of money control. Rationing Credit credit is a Qualitative method of money control. Discount Rate Policy points to the rate of interest that is going to be paid by the banks and other financial institutions on the credits that they avail from the central bank through discount window loan procedure. This will infuse more liquidity in the market. Open Market Operations refers to buying and selling of bonds issued by the Government in the open market. When thes central bank want to infuse liquidity in the monetary system, it will buy government securities in the open market. Question 19 of 35 19. Question Which of the following was/were the financial sector reforms taken up by the Government of India post the economic reforms of 1991? Allowing Foreign Institutional Investors to invest in Indian financial markets. Necessary approval of RBI needed for all banks to set up new branches. Banks were allowed to generate resources from India and abroad. How many of the statements given above is/are correct? a) Only one b) Only two c) All three d) None Correct Solution (b) Statement 1 Statement 2 Statement 3 Correct Incorrect Correct Foreign Institutional Investors (FII), such as merchant bankers, mutual funds and pension funds, are allowed to invest in Indian financial markets. Those banks which fulfill certain conditions have been given freedom to set up new branches without the approval of the RBI and rationalize their existing branch networks.   Though banks have been given permission to generate resources from India and abroad, certain managerial aspects have been retained with the RBI to safeguard the interests of the account-holders and the nation. Incorrect Solution (b) Statement 1 Statement 2 Statement 3 Correct Incorrect Correct Foreign Institutional Investors (FII), such as merchant bankers, mutual funds and pension funds, are allowed to invest in Indian financial markets. Those banks which fulfill certain conditions have been given freedom to set up new branches without the approval of the RBI and rationalize their existing branch networks.   Though banks have been given permission to generate resources from India and abroad, certain managerial aspects have been retained with the RBI to safeguard the interests of the account-holders and the nation. Question 20 of 35 20. Question Which of the following category of people are exempted from income tax? a) Income of a member of a Scheduled Tribes who resides in any area in the State of Nagaland. b) Members of the Scheduled castes who resides in any area in the State of Rajasthan. c) Income of a member of a Scheduled Tribes who resides in any area in the State of Assam. d) Members of the Scheduled Tribes in any area in State of Himachal Pradesh. Correct Solution (a) Income of a member of a Scheduled Tribe [as per article 366(25) of the Constitution] is exempt from tax, if following conditions are satisfied: Such member resides in any area in the State of Nagaland, Manipur, Tripura, Arunachal Pradesh, Mizoram or district of North Cachar Hills, Mikir Hills, Khasi Hills, Jaintia Hills and Garo Hills or in the Ladakh region. Such exemption is available in respect of income which accrues/arises from any source in such areas or income by way of dividends/interest on securities arises from any area.   Incorrect Solution (a) Income of a member of a Scheduled Tribe [as per article 366(25) of the Constitution] is exempt from tax, if following conditions are satisfied: Such member resides in any area in the State of Nagaland, Manipur, Tripura, Arunachal Pradesh, Mizoram or district of North Cachar Hills, Mikir Hills, Khasi Hills, Jaintia Hills and Garo Hills or in the Ladakh region. Such exemption is available in respect of income which accrues/arises from any source in such areas or income by way of dividends/interest on securities arises from any area.   Question 21 of 35 21. Question Consider the following statements about Anthrax: It is a highly infectious disease that is caused by Bacillus anthracis. It affects animals like cows, sheep, goats, and wild herbivores. It can infect humans if they come in contact with infected animals. It can be treated with antibiotics such as ciprofloxacin, doxycycline, or levofloxacin. Choose the correct code: a) Only one b) Only two c) Only three d) All four Correct Solution (d) Anthrax is a highly infectious disease that is caused by Bacillus anthracis. Hence statement 1 is correct. Anthrax bacteria also occur naturally in soil. It affects animals like cows, sheep, goats, and wild herbivores. Hence statement 2 is correct. The disease manifests in three forms depending on the route of infection – cutaneous, gastrointestinal, and inhalational. It can infect humans if they come in contact with infected animals. Hence statement 3 is correct. It can be diagnosed by identifying Bacillus anthracis in blood, skin lesions, or respiratory secretions through laboratory culture, PCR, or ELISA tests. It can be treated with antibiotics such as ciprofloxacin, doxycycline, or levofloxacin. Hence statement 4 is correct. Vaccines are also available for both livestock and humans. Incorrect Solution (d) Anthrax is a highly infectious disease that is caused by Bacillus anthracis. Hence statement 1 is correct. Anthrax bacteria also occur naturally in soil. It affects animals like cows, sheep, goats, and wild herbivores. Hence statement 2 is correct. The disease manifests in three forms depending on the route of infection – cutaneous, gastrointestinal, and inhalational. It can infect humans if they come in contact with infected animals. Hence statement 3 is correct. It can be diagnosed by identifying Bacillus anthracis in blood, skin lesions, or respiratory secretions through laboratory culture, PCR, or ELISA tests. It can be treated with antibiotics such as ciprofloxacin, doxycycline, or levofloxacin. Hence statement 4 is correct. Vaccines are also available for both livestock and humans. Question 22 of 35 22. Question It is located in the state of Tamil Nadu. It lies adjacent to the Bay of Bengal along the east coast. It is one of the major wetlands on the Coromandel Coast after Pulicat Lake. The southern part of the wetland has been reserved land since 2001. It is listed as one of Tamil Nadu’s 141 prioritised wetlands, Kazhuveli is also a wetland of international significance and a potential Ramsar site. It has a feeding ground for long-distance migrants from the cold subarctic regions of Central Asia and Siberia including Black-tailed Godwits, Eurasian Curlew, White Stork, Ruff, and Dunlin. The above paragraph refers to which of the following? a) Kazhuveli Bird Sanctuary b) Vedanthangal Bird Sanctuary c) Vettangudi Bird Sanctuary d) Kanjirankulam Bird Sanctuary Correct Solution (a) Kazhuveli Bird Sanctuary is located in the state of Tamil Nadu. It lies adjacent to the Bay of Bengal along the east coast. It is one of the major wetlands on the Coromandel Coast after Pulicat Lake. The southern part of the wetland has been reserved land since 2001. It is listed as one of Tamil Nadu’s 141 prioritised wetlands, Kazhuveli is also a wetland of international significance and a potential Ramsar site. It has a feeding ground for long-distance migrants from the cold subarctic regions of Central Asia and Siberia including Black-tailed Godwits, Eurasian Curlew, White Stork, Ruff, and Dunlin. Hence option a is correct.   Incorrect Solution (a) Kazhuveli Bird Sanctuary is located in the state of Tamil Nadu. It lies adjacent to the Bay of Bengal along the east coast. It is one of the major wetlands on the Coromandel Coast after Pulicat Lake. The southern part of the wetland has been reserved land since 2001. It is listed as one of Tamil Nadu’s 141 prioritised wetlands, Kazhuveli is also a wetland of international significance and a potential Ramsar site. It has a feeding ground for long-distance migrants from the cold subarctic regions of Central Asia and Siberia including Black-tailed Godwits, Eurasian Curlew, White Stork, Ruff, and Dunlin. Hence option a is correct.   Question 23 of 35 23. Question Consider the following statements about the Directorate of Revenue Intelligence: It is the premier intelligence and enforcement agency on anti-smuggling matters. It works under the aegis of the Central Bureau of Investigation (CBI). It is headquartered in New Delhi. How many of the above statements are correct? a) Only one b) Only two c) All three d) None Correct Solution (b) The Directorate of Revenue Intelligence is the premier intelligence and enforcement agency on anti-smuggling matters. Hence statement 1 is correct. It has been carrying out its mandate of preventing and detecting cases of smuggling of narcotic drugs & psychotropic substances, wildlife items, notes, foreign currency, hazardous & environmentally sensitive materials, antiques, etc., and taking punitive action against the organised crime groups engaged therein. It works under the aegis of the Central Board of Indirect Taxes & Customs (CBIC), Government of India. Hence statement 2 is incorrect. It is also engaged in unearthing commercial frauds and instances of customs duty evasion. It is headquartered in New Delhi. Hence statement 3 is correct. Incorrect Solution (b) The Directorate of Revenue Intelligence is the premier intelligence and enforcement agency on anti-smuggling matters. Hence statement 1 is correct. It has been carrying out its mandate of preventing and detecting cases of smuggling of narcotic drugs & psychotropic substances, wildlife items, notes, foreign currency, hazardous & environmentally sensitive materials, antiques, etc., and taking punitive action against the organised crime groups engaged therein. It works under the aegis of the Central Board of Indirect Taxes & Customs (CBIC), Government of India. Hence statement 2 is incorrect. It is also engaged in unearthing commercial frauds and instances of customs duty evasion. It is headquartered in New Delhi. Hence statement 3 is correct. Question 24 of 35 24. Question Consider the following statements about Pompe Disease: It is caused due to mutations in the GAA gene. It affects males and females equally. It is treated through enzyme replacement therapy (ERT). How many of the above statements are correct? a) Only one b) Only two c) All three d) None Correct Solution (c) Pompe Disease is caused due to mutations in the GAA gene. Hence statement 1 is correct. The GAA gene provides instructions for producing an enzyme called acid alpha-glucosidase (also known as acid maltase). This enzyme is active in lysosomes, which are structures that serve as recycling centers within cells. The enzyme normally breaks down glycogen into a simpler sugar called glucose, which is the main energy source for most cells. Mutations in the GAA gene prevent acid alpha-glucosidase from breaking down glycogen effectively, which allows this sugar to build up to toxic levels in lysosomes. This buildup damages organs and tissues throughout the body, particularly the muscles, leading to the progressive signs and symptoms of Pompe disease. It affects males and females equally. Hence statement 2 is correct. Some common side effects and symptoms include muscle weakness, respiratory issues, heart problems, and difficulty swallowing. It is treated through enzyme replacement therapy (ERT). Hence statement 3 is correct. Incorrect Solution (c) Pompe Disease is caused due to mutations in the GAA gene. Hence statement 1 is correct. The GAA gene provides instructions for producing an enzyme called acid alpha-glucosidase (also known as acid maltase). This enzyme is active in lysosomes, which are structures that serve as recycling centers within cells. The enzyme normally breaks down glycogen into a simpler sugar called glucose, which is the main energy source for most cells. Mutations in the GAA gene prevent acid alpha-glucosidase from breaking down glycogen effectively, which allows this sugar to build up to toxic levels in lysosomes. This buildup damages organs and tissues throughout the body, particularly the muscles, leading to the progressive signs and symptoms of Pompe disease. It affects males and females equally. Hence statement 2 is correct. Some common side effects and symptoms include muscle weakness, respiratory issues, heart problems, and difficulty swallowing. It is treated through enzyme replacement therapy (ERT). Hence statement 3 is correct. Question 25 of 35 25. Question Consider the following statements: The Doctrine of Laches states that the court will only assist those individuals who are vigilant about their rights and not those who are negligent. The Doctrine of Eclipse asserts that a law that infringes fundamental rights is not null or void from the outset, but is merely non-enforceable. The Doctrine of Territorial Nexus dictates that laws made by a state legislature are not applicable outside that state unless there is a sufficient nexus between the state and the object. The Doctrine of Casus Omissus asserts that a matter which should have been provided for in a statute cannot be supplied by the courts. How many of the above statements are correct? a) Only one b) Only two c) Only three d) All four Correct Solution (d) The Doctrine of Laches states that the court will only assist those individuals who are vigilant about their rights and not those who are negligent. Hence statement 1 is correct. The Doctrine of Eclipse asserts that a law that infringes fundamental rights is not null or void from the outset, but is merely non-enforceable. Hence statement 2 is correct. The Doctrine of Territorial Nexus dictates that laws made by a state legislature are not applicable outside that state unless there is a sufficient nexus between the state and the object. Hence statement 3 is correct. The Doctrine of Casus Omissus asserts that a matter which should have been provided for in a statute cannot be supplied by the courts. Hence statement 4 is correct. The Doctrine of Promissory Estoppel is a legal principle that prevents a person from retracting a promise they made, even if the promise was not supported by a contract. The Doctrine of Colourable Legislation is a principle used to determine the legislative competence of laws enacted by various legislatures. The Doctrine of Harmonious Construction posits that a provision of the statute should not be interpreted in isolation but as a whole, to eliminate any inconsistency or repugnancy. The Doctrine of Incidental or Ancillary Powers suggests that the power to legislate on a particular issue also includes the power to legislate on ancillary matters that are reasonably connected to that issue. The Doctrine of Waiver implies that a person can intentionally relinquish their right or privilege or choose not to exercise their right or privilege conferred on them by the state Incorrect Solution (d) The Doctrine of Laches states that the court will only assist those individuals who are vigilant about their rights and not those who are negligent. Hence statement 1 is correct. The Doctrine of Eclipse asserts that a law that infringes fundamental rights is not null or void from the outset, but is merely non-enforceable. Hence statement 2 is correct. The Doctrine of Territorial Nexus dictates that laws made by a state legislature are not applicable outside that state unless there is a sufficient nexus between the state and the object. Hence statement 3 is correct. The Doctrine of Casus Omissus asserts that a matter which should have been provided for in a statute cannot be supplied by the courts. Hence statement 4 is correct. The Doctrine of Promissory Estoppel is a legal principle that prevents a person from retracting a promise they made, even if the promise was not supported by a contract. The Doctrine of Colourable Legislation is a principle used to determine the legislative competence of laws enacted by various legislatures. The Doctrine of Harmonious Construction posits that a provision of the statute should not be interpreted in isolation but as a whole, to eliminate any inconsistency or repugnancy. The Doctrine of Incidental or Ancillary Powers suggests that the power to legislate on a particular issue also includes the power to legislate on ancillary matters that are reasonably connected to that issue. The Doctrine of Waiver implies that a person can intentionally relinquish their right or privilege or choose not to exercise their right or privilege conferred on them by the state Question 26 of 35 26. Question Consider the following statements about the European Free Trade Association (EFTA): It is an intergovernmental organization established by the Washington Convention. It promotes free trade and economic integration between its members within Europe and globally. It currently has four member countries – Iceland, Liechtenstein, Norway, and Switzerland. How many of the above statements are correct? a) Only one b) Only two c) All three d) None Correct Solution (b) The European Free Trade Association (EFTA) is an intergovernmental organization established by the Stockholm Convention. Hence statement 1 is incorrect. EFTA is not a customs union because the individual EFTA States are free to set their own customs tariffs and arrange other foreign trade measures vis-à-vis the non-EFTA States. It promotes free trade and economic integration between its members within Europe and globally. Hence statement 2 is correct. The EFTA countries have developed one of the largest networks of Free Trade Agreements (FTAs). These FTAs span over 60 countries and territories, including the EU. The EFTA members are all open, competitive economies committed to the progressive liberalization of trade in the multinational arena as well as in free trade agreements. It currently has four member countries – Iceland, Liechtenstein, Norway, and Switzerland. Hence statement 3 is correct.   Note: Customs unions are groups of countries that apply one common system of procedures, rules, and tariffs for all or almost all their imports, exports, and transit goods. Usually, countries participating in customs unions share common trade and competition policies.   Incorrect Solution (b) The European Free Trade Association (EFTA) is an intergovernmental organization established by the Stockholm Convention. Hence statement 1 is incorrect. EFTA is not a customs union because the individual EFTA States are free to set their own customs tariffs and arrange other foreign trade measures vis-à-vis the non-EFTA States. It promotes free trade and economic integration between its members within Europe and globally. Hence statement 2 is correct. The EFTA countries have developed one of the largest networks of Free Trade Agreements (FTAs). These FTAs span over 60 countries and territories, including the EU. The EFTA members are all open, competitive economies committed to the progressive liberalization of trade in the multinational arena as well as in free trade agreements. It currently has four member countries – Iceland, Liechtenstein, Norway, and Switzerland. Hence statement 3 is correct.   Note: Customs unions are groups of countries that apply one common system of procedures, rules, and tariffs for all or almost all their imports, exports, and transit goods. Usually, countries participating in customs unions share common trade and competition policies.   Question 27 of 35 27. Question Consider the following statements about Exchange-Traded Funds (ETFs): It is a collection of marketable securities that tracks an index, a commodity, bonds, or a basket of assets. It can be purchased or sold on a stock exchange in the same way that regular stocks can. Choose the correct code: a) 1 only b) 2 only c) Both 1 and 2 d) Neither 1 nor 2 Correct Solution (c) Exchange-traded funds (ETFs) are a collection of marketable securities that track an index, a commodity, bonds, or a basket of assets. Hence statement 1 is correct. It can be structured to track anything from the price of an individual commodity to a large and diverse collection of securities. ETFs can even be structured to track specific investment strategies. It can be purchased or sold on a stock exchange in the same way that regular stocks can. Hence statement 2 is correct. The traded price of an ETF changes throughout the day like any other stock, as it is bought and sold on the stock exchange. The trading value of an ETF is based on the net asset value of the underlying stocks that it represents. Incorrect Solution (c) Exchange-traded funds (ETFs) are a collection of marketable securities that track an index, a commodity, bonds, or a basket of assets. Hence statement 1 is correct. It can be structured to track anything from the price of an individual commodity to a large and diverse collection of securities. ETFs can even be structured to track specific investment strategies. It can be purchased or sold on a stock exchange in the same way that regular stocks can. Hence statement 2 is correct. The traded price of an ETF changes throughout the day like any other stock, as it is bought and sold on the stock exchange. The trading value of an ETF is based on the net asset value of the underlying stocks that it represents. Question 28 of 35 28. Question Consider the following statements: The Global Cooling Watch Report was released by the UN Environment Programme (UNEP)-led Cool Coalition. The Global Cooling Pledge was signed at COP25 to address the growing demand for cooling while mitigating its climate impacts. Choose the correct code: a) 1 only b) 2 only c) Both 1 and 2 d) Neither 1 nor 2 Correct Solution (a) The Global Cooling Watch Report was released by the UN Environment Programme (UNEP)-led Cool Coalition. Hence statement 1 is correct. It is a comprehensive analysis of the global cooling sector and focuses not only on space cooling but also on cold chains for food and health. Its key highlights are: Currently, cooling equipment currently accounts for 20% of total electricity consumption and is expected to more than double by 2050. Under a business-as-usual scenario, emissions from cooling are predicted to account for more than 10 percent of global emissions in 2050. The Global Cooling Pledge was signed at COP28 to address the growing demand for cooling while mitigating its climate impacts. Hence statement 2 is incorrect. Around 63 countries have signed the pledge. However, India has not signed it yet. Incorrect Solution (a) The Global Cooling Watch Report was released by the UN Environment Programme (UNEP)-led Cool Coalition. Hence statement 1 is correct. It is a comprehensive analysis of the global cooling sector and focuses not only on space cooling but also on cold chains for food and health. Its key highlights are: Currently, cooling equipment currently accounts for 20% of total electricity consumption and is expected to more than double by 2050. Under a business-as-usual scenario, emissions from cooling are predicted to account for more than 10 percent of global emissions in 2050. The Global Cooling Pledge was signed at COP28 to address the growing demand for cooling while mitigating its climate impacts. Hence statement 2 is incorrect. Around 63 countries have signed the pledge. However, India has not signed it yet. Question 29 of 35 29. Question Consider the following statements: Convertible preference shares are those shares that can be easily converted into equity shares. Non-convertible preference shares are those shares that cannot be converted into equity shares. Redeemable preference shares are those shares that can be repurchased or redeemed by the issuing company at a fixed rate and date. Non-redeemable preference shares are those shares that cannot be redeemed or repurchased by the issuing company at a fixed date. Choose the correct code: a) Only one b) Only two c) Only three d) All four Correct Solution (d) Convertible preference shares are those shares that can be easily converted into equity shares. Hence statement 1 is correct. Non-convertible preference shares are those shares that cannot be converted into equity shares. Hence statement 2 is correct. Redeemable preference shares are those shares that can be repurchased or redeemed by the issuing company at a fixed rate and date. Hence statement 3 is correct. These types of shares help the company by providing a cushion during times of inflation. Non-redeemable preference shares are those shares that cannot be redeemed or repurchased by the issuing company at a fixed date. Hence statement 4 is correct. These types of shares help companies by acting as a lifesaver during times of inflation. Incorrect Solution (d) Convertible preference shares are those shares that can be easily converted into equity shares. Hence statement 1 is correct. Non-convertible preference shares are those shares that cannot be converted into equity shares. Hence statement 2 is correct. Redeemable preference shares are those shares that can be repurchased or redeemed by the issuing company at a fixed rate and date. Hence statement 3 is correct. These types of shares help the company by providing a cushion during times of inflation. Non-redeemable preference shares are those shares that cannot be redeemed or repurchased by the issuing company at a fixed date. Hence statement 4 is correct. These types of shares help companies by acting as a lifesaver during times of inflation. Question 30 of 35 30. Question Consider the following statements about Chousingha: It is endemic to the Indian subcontinent. It is the smallest antelope found in Asia. It is listed as endangered on the IUCN Red List. How many of the above statements are correct? a) Only one b) Only two c) All three d) None Correct Solution (b) Chousingha is endemic to the Indian subcontinent. Hence statement 1 is correct. The four-horned antelope, or chousingha, is a small antelope found in India and Nepal. It is the smallest antelope found in Asia. Hence statement 2 is correct. They are usually diurnal and solitary by nature; however, they can be spotted in loose groups of three to four. Animals are sedentary, inhabiting more or less the same region throughout their lives. It is listed as vulnerable on the IUCN Red List. Hence statement 3 is incorrect. Incorrect Solution (b) Chousingha is endemic to the Indian subcontinent. Hence statement 1 is correct. The four-horned antelope, or chousingha, is a small antelope found in India and Nepal. It is the smallest antelope found in Asia. Hence statement 2 is correct. They are usually diurnal and solitary by nature; however, they can be spotted in loose groups of three to four. Animals are sedentary, inhabiting more or less the same region throughout their lives. It is listed as vulnerable on the IUCN Red List. Hence statement 3 is incorrect. Question 31 of 35 31. Question The diagonal of a rectangle is √21 cm and its area is 50 sq. cm. What is the perimeter of the rectangle? (Note – Diagonal equals the square root of the width squared plus the height squared) a) 11 b) 22 c) 14 d) 28 Correct Solution (b) For a rectangle, d^2 = l^2 + b^2 where l= length, b = breadth and d = diagonal of the of the rectangle   d= √21 d^2 = l^2 + b^2 l^2 + b^2= 21 ….. (1) Area = l × b = 50 …..(2)   We know that (a+b)^2 = a^2 + b^2 + 2ab Using the same formula, we have (l+b)^2 = l^2 + b^2 + 2lb (l+b)^2 = 21 + 2*50 (l+b) = 11 Perimeter of the rectangle = 2(l + b) = 2(11) = 22 cm   Incorrect Solution (b) For a rectangle, d^2 = l^2 + b^2 where l= length, b = breadth and d = diagonal of the of the rectangle   d= √21 d^2 = l^2 + b^2 l^2 + b^2= 21 ….. (1) Area = l × b = 50 …..(2)   We know that (a+b)^2 = a^2 + b^2 + 2ab Using the same formula, we have (l+b)^2 = l^2 + b^2 + 2lb (l+b)^2 = 21 + 2*50 (l+b) = 11 Perimeter of the rectangle = 2(l + b) = 2(11) = 22 cm   Question 32 of 35 32. Question A and B playing a game with rolling a fare dice, one after the other replacing it every time till one of them gets a six. If A begins the game, then the probability that A wins the game is a) 1/6 b) 2/6 c) 5/11 d) 6/11 Correct Solution (c) Let  S  denote  the  success  (getting  a  ‘6’)  and  F  denote  the  failure  (not  getting  a  ‘6’)  . Thus, P(S)= 1/6 ​ =p, P(F)= 5/6​ =q P(A wins  in  first  throw)=P(S)=p P(A  wins  in  third  throw)=P(FFS)=qqp P(A  wins  in  fifth  throw)=P(FFFFS)=qqqqp So,  P(A  wins)=p+qqp+qqqqp+… =p(1+q^2 +q^4 +…) = 1−q^2 p ​ = (1/6)/(1− 25/36) ​ ​ =  6/11 ​ P(B  wins)=1–P(A  wins) P(B  wins)  =1− 6/11 ​ = 5/11 ​ So,  P(A  wins)= 6/11 ​ ,  P(B  wins)= 5/11 Incorrect Solution (c) Let  S  denote  the  success  (getting  a  ‘6’)  and  F  denote  the  failure  (not  getting  a  ‘6’)  . Thus, P(S)= 1/6 ​ =p, P(F)= 5/6​ =q P(A wins  in  first  throw)=P(S)=p P(A  wins  in  third  throw)=P(FFS)=qqp P(A  wins  in  fifth  throw)=P(FFFFS)=qqqqp So,  P(A  wins)=p+qqp+qqqqp+… =p(1+q^2 +q^4 +…) = 1−q^2 p ​ = (1/6)/(1− 25/36) ​ ​ =  6/11 ​ P(B  wins)=1–P(A  wins) P(B  wins)  =1− 6/11 ​ = 5/11 ​ So,  P(A  wins)= 6/11 ​ ,  P(B  wins)= 5/11 Question 33 of 35 33. Question When a particular positive number is divided by 3, the remainder is 1. If the same number is divided by 5, the remainder is 4. If the difference between the quotients of the division is 3, then find the number. a) 34 b) 19 c) 49 d) 64 Correct Solution (b) Let the quotients when this number is divided by 3 and 5 be x and y respectively. (Note that x will be greater than y as 3 is smaller than 5) Number = 3x + 1 = 5y + 4 Given that, x – y = 3 On solving both equation we get, x = 6, y=3 Thus the number is 5 × 3 + 4= 19. Incorrect Solution (b) Let the quotients when this number is divided by 3 and 5 be x and y respectively. (Note that x will be greater than y as 3 is smaller than 5) Number = 3x + 1 = 5y + 4 Given that, x – y = 3 On solving both equation we get, x = 6, y=3 Thus the number is 5 × 3 + 4= 19. Question 34 of 35 34. Question In 10 years, A will be thrice as old as B was 5 years ago. If A is now 9 years older than B, the present age of B is a) 19 b) 21 c) 17 d) 15 Correct Solution (c) Let B’s present age = x years. Then, A’s present age = (x + 9) years. (x + 9) + 10 = 3(x – 5) => x + 19 = 3x – 15 => x =17 Incorrect Solution (c) Let B’s present age = x years. Then, A’s present age = (x + 9) years. (x + 9) + 10 = 3(x – 5) => x + 19 = 3x – 15 => x =17 Question 35 of 35 35. Question Out of the four annual examinations, each with a total of 100 marks, a student secured average marks of 35%, 65% and 50% in the first, second and third annual examinations. To have an overall average of 60%, how many marks does the student need to secure in the fourth annual examination? a) 60 b) 70 c) 80 d) 90 Correct Solution (d) Let the average marks in the third Annual examination be x. Total marks = (Marks in first + second + third + forth) Annual examination = > 4(60) ( 100/100) = ( 35/100) (100) + ( 65/100) (100) + ( 50/100) (100) + ( x/100) (100) => 4(60) = 35 + 65 +50 + x =>x = 90 So, the student must score 90% in the fourth annual examination to secure 60% overall average. ∴ Average marks in the third annual examination (90/100) × 100 = 90 marks.   Incorrect Solution (d) Let the average marks in the third Annual examination be x. Total marks = (Marks in first + second + third + forth) Annual examination = > 4(60) ( 100/100) = ( 35/100) (100) + ( 65/100) (100) + ( 50/100) (100) + ( x/100) (100) => 4(60) = 35 + 65 +50 + x =>x = 90 So, the student must score 90% in the fourth annual examination to secure 60% overall average. ∴ Average marks in the third annual examination (90/100) × 100 = 90 marks.   window.wpProQuizInitList = window.wpProQuizInitList || []; window.wpProQuizInitList.push({ id: '#wpProQuiz_3623', init: { quizId: 3623, mode: 1, globalPoints: 70, timelimit: 1800, resultsGrade: [0], bo: 704, qpp: 0, catPoints: [70], formPos: 0, lbn: "Test-summary", json: {"31922":{"type":"single","id":31922,"catId":0,"points":2,"correct":[0,0,0,1]},"31924":{"type":"single","id":31924,"catId":0,"points":2,"correct":[0,0,0,1]},"31925":{"type":"single","id":31925,"catId":0,"points":2,"correct":[0,1,0,0]},"31927":{"type":"single","id":31927,"catId":0,"points":2,"correct":[0,1,0,0]},"31930":{"type":"single","id":31930,"catId":0,"points":2,"correct":[0,1,0,0]},"31932":{"type":"single","id":31932,"catId":0,"points":2,"correct":[0,0,1,0]},"31934":{"type":"single","id":31934,"catId":0,"points":2,"correct":[0,0,0,1]},"31936":{"type":"single","id":31936,"catId":0,"points":2,"correct":[0,1,0,0]},"31937":{"type":"single","id":31937,"catId":0,"points":2,"correct":[0,1,0,0]},"31939":{"type":"single","id":31939,"catId":0,"points":2,"correct":[0,1,0,0]},"31941":{"type":"single","id":31941,"catId":0,"points":2,"correct":[0,1,0,0]},"31943":{"type":"single","id":31943,"catId":0,"points":2,"correct":[0,1,0,0]},"31945":{"type":"single","id":31945,"catId":0,"points":2,"correct":[0,1,0,0]},"31948":{"type":"single","id":31948,"catId":0,"points":2,"correct":[0,0,0,1]},"31950":{"type":"single","id":31950,"catId":0,"points":2,"correct":[0,1,0,0]},"31952":{"type":"single","id":31952,"catId":0,"points":2,"correct":[0,0,1,0]},"31955":{"type":"single","id":31955,"catId":0,"points":2,"correct":[0,0,1,0]},"31957":{"type":"single","id":31957,"catId":0,"points":2,"correct":[0,0,1,0]},"31960":{"type":"single","id":31960,"catId":0,"points":2,"correct":[0,1,0,0]},"31961":{"type":"single","id":31961,"catId":0,"points":2,"correct":[1,0,0,0]},"31963":{"type":"single","id":31963,"catId":0,"points":2,"correct":[0,0,0,1]},"31966":{"type":"single","id":31966,"catId":0,"points":2,"correct":[1,0,0,0]},"31968":{"type":"single","id":31968,"catId":0,"points":2,"correct":[0,1,0,0]},"31969":{"type":"single","id":31969,"catId":0,"points":2,"correct":[0,0,1,0]},"31970":{"type":"single","id":31970,"catId":0,"points":2,"correct":[0,0,0,1]},"31972":{"type":"single","id":31972,"catId":0,"points":2,"correct":[0,1,0,0]},"31975":{"type":"single","id":31975,"catId":0,"points":2,"correct":[0,0,1,0]},"31976":{"type":"single","id":31976,"catId":0,"points":2,"correct":[1,0,0,0]},"31979":{"type":"single","id":31979,"catId":0,"points":2,"correct":[0,0,0,1]},"31981":{"type":"single","id":31981,"catId":0,"points":2,"correct":[0,1,0,0]},"31982":{"type":"single","id":31982,"catId":0,"points":2,"correct":[0,1,0,0]},"31984":{"type":"single","id":31984,"catId":0,"points":2,"correct":[0,0,1,0]},"31986":{"type":"single","id":31986,"catId":0,"points":2,"correct":[0,1,0,0]},"31988":{"type":"single","id":31988,"catId":0,"points":2,"correct":[0,0,1,0]},"31991":{"type":"single","id":31991,"catId":0,"points":2,"correct":[0,0,0,1]}} } }); All the Best IASbaba

Daily Prelims CA Quiz

UPSC Quiz – 2024 : IASbaba’s Daily Current Affairs Quiz 9th April 2024

For Previous Daily Quiz (ARCHIVES) – CLICK HERE The Current Affairs questions are based on sources like ‘The Hindu’, ‘Indian Express’ and ‘PIB’, which are very important sources for UPSC Prelims Exam. The questions are focused on both the concepts and facts. The topics covered here are generally different from what is being covered under ‘Daily Current Affairs/Daily News Analysis (DNA) and Daily Static Quiz’ to avoid duplication. The questions would be published from Monday to Saturday before 2 PM. One should not spend more than 10 minutes on this initiative. Gear up and Make the Best Use of this initiative. Do remember that, “the difference between Ordinary and EXTRA-Ordinary is PRACTICE!!” Important Note: Don’t forget to post your marks in the comment section. Also, let us know if you enjoyed today’s test 🙂After completing the 5 questions, click on ‘View Questions’ to check your score, time taken, and solutions.To take the Test Click Here

[DAY 31] 60 DAY RAPID REVISION (RaRe) SERIES for UPSC Prelims 2024 – GEOGRAPHY, CURRENT AFFAIRS & CSAT TEST SERIES!

Archives Hello Friends The 60 Days Rapid Revision (RaRe) Series is IASbaba’s Flagship Initiative recommended by Toppers and loved by the aspirants’ community every year. It is the most comprehensive program which will help you complete the syllabus, revise and practice tests on a daily basis. The Programme on a daily basis includes Daily Prelims MCQs from Static (Monday – Saturday) Daily Static Quiz will cover all the topics of static subjects – Polity, History, Geography, Economics, Environment and Science and technology. 20 questions will be posted daily and these questions are framed from the topics mentioned in the schedule. It will ensure timely and streamlined revision of your static subjects. Daily Current Affairs MCQs (Monday – Saturday) Daily 5 Current Affairs questions, based on sources like ‘The Hindu’, ‘Indian Express’ and ‘PIB’, would be published from Monday to Saturday according to the schedule. Daily CSAT Quiz (Monday – Friday) CSAT has been an Achilles heel for many aspirants. Daily 5 CSAT Questions will be published. Note – Daily Test of 20 static questions, 10 current affairs, and 5 CSAT questions. (35 Prelims Questions) in QUIZ FORMAT will be updated on a daily basis. To Know More about 60 Days Rapid Revision (RaRe) Series – CLICK HERE   60 Day Rapid Revision (RaRe) Series Schedule – CLICK HERE  Important Note Comment your Scores in the Comment Section. This will keep you accountable, responsible and sincere in days to come. It will help us come out with the Cut-Off on a Daily Basis. Let us know if you enjoyed today’s test 🙂  You can post your comments in the given format  (1) Your Score (2) Matrix Meter (3) New Learning from the Test Time limit: 0 Test-summary 0 of 35 questions completed Questions: 1 2 3 4 5 6 7 8 9 10 11 12 13 14 15 16 17 18 19 20 21 22 23 24 25 26 27 28 29 30 31 32 33 34 35 Information The following Test is based on the syllabus of 60 Days Plan-2023 for UPSC IAS Prelims 2022. To view Solutions, follow these instructions: Click on – ‘Start Test’ button Solve Questions Click on ‘Test Summary’ button Click on ‘Finish Test’ button Now click on ‘View Questions’ button – here you will see solutions and links. You have already completed the test before. Hence you can not start it again. Test is loading... You must sign in or sign up to start the test. You have to finish following test, to start this test: Results 0 of 35 questions answered correctly Your time: Time has elapsed You have scored 0 points out of 0 points, (0) Average score     Your score     Categories Not categorized 0% Your result has been entered into leaderboard Loading Name: E-Mail: Captcha: maximum of 70 points Pos. Name Entered on Points Result Table is loading No data available 1 2 3 4 5 6 7 8 9 10 11 12 13 14 15 16 17 18 19 20 21 22 23 24 25 26 27 28 29 30 31 32 33 34 35 Answered Review Question 1 of 35 1. Question Consider the following: Its colour varies from light grey to ash grey. It is rich in potash nut poor in phosphorus. It is present in Lower Ganga valley Which of the following soils satisfies the above features? a) Alluvial soil b) Laterite soil c) Marshy Soil d) Red Soil Correct Solution (a) Alluvial soil This soil is well-drained and poorly drained with an immature profile in undulating areas. This soil has a phosphorous deficiency and is rich in potash and lime. It is present in Lower Ganga valley. The colour of soil varies from light grey to ash grey. This soil is suited for Rice, maize, wheat, sugarcane, oilseeds, etc. Incorrect Solution (a) Alluvial soil This soil is well-drained and poorly drained with an immature profile in undulating areas. This soil has a phosphorous deficiency and is rich in potash and lime. It is present in Lower Ganga valley. The colour of soil varies from light grey to ash grey. This soil is suited for Rice, maize, wheat, sugarcane, oilseeds, etc. Question 2 of 35 2. Question Consider the following statements They are formed as the result of intense leaching due to tropical rains. They are poor in organic matter, nitrogen, phosphate and calcium. They are widely used for bricks making which is used for house construction. The humus content is removed by bacteria. Which of the following Soil best suits the description given in the above statements? a) Alluvial Soil b) Arid Soil c) Forest Soil d) Laterite Soil Correct Solution (d) Laterite Soil They are formed as the result of intense leaching due to tropical rains. It is well developed in areas with high temperature and high rainfall. They are poor in organic matter, nitrogen, phosphate and calcium. They are rich in potash and iron oxide. They are widely used for bricks making which is used for house construction. The humus content is removed by bacteria. With the application of manures and fertilizers it can be made use for cultivation. Incorrect Solution (d) Laterite Soil They are formed as the result of intense leaching due to tropical rains. It is well developed in areas with high temperature and high rainfall. They are poor in organic matter, nitrogen, phosphate and calcium. They are rich in potash and iron oxide. They are widely used for bricks making which is used for house construction. The humus content is removed by bacteria. With the application of manures and fertilizers it can be made use for cultivation. Question 3 of 35 3. Question Arrange the following soils in descending order of their prevalence in the country. Laterite soil Alluvial soil Black soil Select the correct answer using the code given below. a) 3-2-1 b) 2-3-1 c) 3-1-2 d) 2-1-3 Correct Solution (b) Alluvial soils are by far the largest and the most important soil group of India. Covering about 15 lakh sq km or about 45.6 per cent of the total land area of the country. Black soils are spread over 5.46 lakh sq km i.e. 16.6 per cent of the total geographical area of the country. Laterite and lateritic soils are widely spread in India and cover an area of 2.48 lakh sq km. Incorrect Solution (b) Alluvial soils are by far the largest and the most important soil group of India. Covering about 15 lakh sq km or about 45.6 per cent of the total land area of the country. Black soils are spread over 5.46 lakh sq km i.e. 16.6 per cent of the total geographical area of the country. Laterite and lateritic soils are widely spread in India and cover an area of 2.48 lakh sq km. Question 4 of 35 4. Question Consider the following statements The Gondwana rocks are young metamorphic rocks. The Cuddapah rocks are ancient igneous rocks. Choose the correct code: a) 1 only b) 2 only c) Both 1 and 2 d) Neither 1 nor 2 Correct Solution (d) Statement 1 Statement 2 Incorrect Incorrect The Gondwana rocks are young sedimentary rocks. After the process of weathering they give rise to comparatively less mature soils. The soil found here is more or less of uniform character but of low fertility. The Cuddapah rocks are ancient sedimentary rocks. After the process of weathering, they give calcareous and argillaceous soils. The soil found here is mostly devoid of metalliferous minerals. Incorrect Solution (d) Statement 1 Statement 2 Incorrect Incorrect The Gondwana rocks are young sedimentary rocks. After the process of weathering they give rise to comparatively less mature soils. The soil found here is more or less of uniform character but of low fertility. The Cuddapah rocks are ancient sedimentary rocks. After the process of weathering, they give calcareous and argillaceous soils. The soil found here is mostly devoid of metalliferous minerals. Question 5 of 35 5. Question Consider the following statements about a Soil type: It swells and become sticky when wet and shrinks when dried. It retains moisture for a very long time. It is covers most of the Deccan Plateau. Identify the soil type based on above characteristics. a) Laterite soil b) Alluvial Soil c) Red Soil d) Black Soil Correct Solution (d) Black soil covers most of the Deccan Plateau and the upper reaches of the Godavari and the Krishna. It is also known as Regur Soil or Black Cotton Soil. It is clayey, deep and impermeable. It swells and becomes sticky when wet and shrink when dried. It has characteristics of ‘self-ploughing’. The soil retains the moisture for a very long time. They are rich in lime, iron, magnesia and alumina. Incorrect Solution (d) Black soil covers most of the Deccan Plateau and the upper reaches of the Godavari and the Krishna. It is also known as Regur Soil or Black Cotton Soil. It is clayey, deep and impermeable. It swells and becomes sticky when wet and shrink when dried. It has characteristics of ‘self-ploughing’. The soil retains the moisture for a very long time. They are rich in lime, iron, magnesia and alumina. Question 6 of 35 6. Question Consider the following pairs: Soil Layer Region 1.  Horizon A Topmost Zone, where organic materials get incorporated with the mineral matter 2.  Horizon E Bedrock of the soil 3.  Horizon C Loose/ weathered parent material. Which of the pairs given above are correctly matched? a) 1 and 3 only b) 2 and 3 only c) 1 and 2 only d) 1, 2 and 3 Correct Solution (a) A (topsoil): Mostly minerals from parent material with organic matter incorporated. A good material for plants and other organisms to live. E (eluviated): Leached of clay, minerals, and organic matter, leaving a concentration of sand and silt particles of quartz or other resistant materials – missing in some soils but often found in older soils and forest soils. B (subsoil): Rich in minerals that leached (moved down) from the A or E horizons and accumulated here. C (parent material): The deposit at Earth’s surface from which the soil developed. R (bedrock): A mass of rock such as granite, basalt, quartzite, limestone or sandstone that forms the parent material for some soils – if the bedrock is close enough to the surface to weather. This is not soil and is located under the C horizon. Incorrect Solution (a) A (topsoil): Mostly minerals from parent material with organic matter incorporated. A good material for plants and other organisms to live. E (eluviated): Leached of clay, minerals, and organic matter, leaving a concentration of sand and silt particles of quartz or other resistant materials – missing in some soils but often found in older soils and forest soils. B (subsoil): Rich in minerals that leached (moved down) from the A or E horizons and accumulated here. C (parent material): The deposit at Earth’s surface from which the soil developed. R (bedrock): A mass of rock such as granite, basalt, quartzite, limestone or sandstone that forms the parent material for some soils – if the bedrock is close enough to the surface to weather. This is not soil and is located under the C horizon. Question 7 of 35 7. Question Which of the below given pairs is/are correctly matched? Soil                                                 Properties Red soil                                                 Calcareous Alkali Soil                                               pH – 9 Black soil                                           rich in Kaolinite Laterite soil                                       rich in iron and aluminium Choose the correct code: a) Only one b) Only two c) Only three d) All four Correct Solution (b) Match 1 Match 2 Match 3 Match 4 Incorrect Correct Incorrect Correct Red soils are red in color and are due to presence of iron oxide and are not Calcareous. They are acidic mainly due to the parent rock. Alkali, or Alkaline, soils are clay soils with high pH (greater than 8.5), a poor soil structure and a low infiltration capacity. Often they have a hard calcareous layer at 0.5 to 1 metre depth. Kaolinite is a clay mineral is the weathering product of feldspars Laterite is both a soil and a rock type rich in iron and aluminium and is commonly considered to have formed in hot and wet tropical areas. Incorrect Solution (b) Match 1 Match 2 Match 3 Match 4 Incorrect Correct Incorrect Correct Red soils are red in color and are due to presence of iron oxide and are not Calcareous. They are acidic mainly due to the parent rock. Alkali, or Alkaline, soils are clay soils with high pH (greater than 8.5), a poor soil structure and a low infiltration capacity. Often they have a hard calcareous layer at 0.5 to 1 metre depth. Kaolinite is a clay mineral is the weathering product of feldspars Laterite is both a soil and a rock type rich in iron and aluminium and is commonly considered to have formed in hot and wet tropical areas. Question 8 of 35 8. Question Consider the following statements: In India red soil and alluvial soil consist for more than 50% of the soil in the country. Red and lateritic soil is mainly found in North India. Alluvial Soil is found in abundance at the upper banks of Cauvery. How many of the above statements are correct? a) Only one b) Only two c) All three d) None Correct Solution (a) Statement 1 Statement 2 Statement 3 Correct Incorrect Incorrect In India red soil and alluvial soil consist for more than 50% of the soil in the country. Alluvial Soil – 40% Red Soil – 18% Red is mainly found in eastern and southern parts of Deccan plateau, Odisha and Chattisgarh. And the Laterite soil is found in Karnataka, Tamil Nadu, Andhra Pradesh, Kerala and Madhya Pradesh. Alluvial Soil is found in abundance in Ganga – Brahmaputra plains. Incorrect Solution (a) Statement 1 Statement 2 Statement 3 Correct Incorrect Incorrect In India red soil and alluvial soil consist for more than 50% of the soil in the country. Alluvial Soil – 40% Red Soil – 18% Red is mainly found in eastern and southern parts of Deccan plateau, Odisha and Chattisgarh. And the Laterite soil is found in Karnataka, Tamil Nadu, Andhra Pradesh, Kerala and Madhya Pradesh. Alluvial Soil is found in abundance in Ganga – Brahmaputra plains. Question 9 of 35 9. Question Which of the following statements is/are correct regarding red soils? Red soil occurs mostly in eastern and southern part of Deccan Plateau. It is red in colour because of the presence of copper oxides. It looks yellow when it occurs in a hydrated form. How many of the above statements are correct? a) Only one b) Only two c) All three d) None Correct Solution (b) Statement 1 Statement 2 Statement 3 Correct Incorrect Correct Red soil occurs mostly in eastern and southern part of Deccan Plateau. It is also found in parts of Odisha and Chhattisgarh. It is red in colour because of the presence of iron oxides. It looks yellow when it occurs in a hydrated form. Incorrect Solution (b) Statement 1 Statement 2 Statement 3 Correct Incorrect Correct Red soil occurs mostly in eastern and southern part of Deccan Plateau. It is also found in parts of Odisha and Chhattisgarh. It is red in colour because of the presence of iron oxides. It looks yellow when it occurs in a hydrated form. Question 10 of 35 10. Question Which of the following are the reasons for division of Indo-Brahma River system into 3 branches today (i.e.Indus, Ganga and Brahmaputra)? The upheaval in the western Himalayas Uplift of the Potwar Plateau Down thrusting of the Malda gap Select the correct statements a) 1 and 2 only b) 2 and 3 only c) 1 and 3 only d) All of the above Correct Solution (d) Statement 1 Statement 2 Statement 3 Correct Correct Correct Pleistocene upheaval in the western Himalayas Uplift of the Potwar Plateau (Delhi Ridge), which acted as the water divide between the Indus and Ganga drainage systems The down thrusting of the Malda gap area between the Rajmahal hills and the Meghalaya plateau during the mid-Pleistocene period, diverted the Ganga and the Brahmaputra systems to flow towards the Bay of Bengal Incorrect Solution (d) Statement 1 Statement 2 Statement 3 Correct Correct Correct Pleistocene upheaval in the western Himalayas Uplift of the Potwar Plateau (Delhi Ridge), which acted as the water divide between the Indus and Ganga drainage systems The down thrusting of the Malda gap area between the Rajmahal hills and the Meghalaya plateau during the mid-Pleistocene period, diverted the Ganga and the Brahmaputra systems to flow towards the Bay of Bengal Question 11 of 35 11. Question Consider the following statements River Luni originates from Aravalli range near Ajmer The water of river Luni is both fresh and saline The Luni River drains in gulf of Khambhat How many of the above statements are correct? a) Only one b) Only two c) All three d) None Correct Solution (b) Statement 1 Statement 2 Statement 3 Correct Correct Incorrect River Luni originates from Naga hills in Aravalli range near Ajmer   The river has fresh water initially but by the time it reaches Barmer, the water starts to turn saline due to rich salt content of the soil. The river Luni ends in marshy lands of Rann of Kutch. Incorrect Solution (b) Statement 1 Statement 2 Statement 3 Correct Correct Incorrect River Luni originates from Naga hills in Aravalli range near Ajmer   The river has fresh water initially but by the time it reaches Barmer, the water starts to turn saline due to rich salt content of the soil. The river Luni ends in marshy lands of Rann of Kutch. Question 12 of 35 12. Question Consider the following statements about Mahadayi/Mandovi river: Kalasa and Banduri are its tributaries. The famous Dudhsagar falls is located on this river. Mahadayi river water dispute is between Karnataka, Goa and Maharashtra. How many of the above statements are correct? a) Only one b) Only two c) All three d) None Correct Solution (c) The Mahadayi/Mandovi River also known as Mahadayi or Mhadei river, is described as the lifeline of the Indian state of Goa. The river has a length of 77 kilometres (48 miles), 29 kilometres (18 miles) in Karnataka and 52 kilometres (32 miles) in Goa. It originates from a cluster of 30 springs at Bhimgad in the Western Ghats in the Belagavi district of Karnataka Dudhsagar falls and Vajrpoha falls located on this river. The Kalasa-Banduri Nala is a project undertaken by the Government of Karnataka. Incorrect Solution (c) The Mahadayi/Mandovi River also known as Mahadayi or Mhadei river, is described as the lifeline of the Indian state of Goa. The river has a length of 77 kilometres (48 miles), 29 kilometres (18 miles) in Karnataka and 52 kilometres (32 miles) in Goa. It originates from a cluster of 30 springs at Bhimgad in the Western Ghats in the Belagavi district of Karnataka Dudhsagar falls and Vajrpoha falls located on this river. The Kalasa-Banduri Nala is a project undertaken by the Government of Karnataka. Question 13 of 35 13. Question West flowing rivers of peninsular India do not form deltas but form only estuaries. Which of the following reasons can be attributed to this phenomenon? Non-perennial flow of water Absence of meanders Passage through hard rocks. How many of the above statements are correct? a) Only one b) Only two c) All three d) None Correct Solution (a) Statement-1 and 2: incorrect Statement-3: correct Absence of meanders and non-perennial flow of water are characteristics of Peninsular rivers, including the east flowing rivers which form deltas. The west flowing rivers of India do not form delta because they passes through hard rocks. This means that these are devoid of sediments which are essential for formation of delta Incorrect Solution (a) Statement-1 and 2: incorrect Statement-3: correct Absence of meanders and non-perennial flow of water are characteristics of Peninsular rivers, including the east flowing rivers which form deltas. The west flowing rivers of India do not form delta because they passes through hard rocks. This means that these are devoid of sediments which are essential for formation of delta Question 14 of 35 14. Question Tel, Ib, Jonk, Ong are tributaries of which of the following river? a) Subarnarekha b) Baitrani c) Damodar d) Mahanadi Correct Solution (d) The Seonath, the Hasdeo, the Mand and the Ib joins Mahanadi from left. The Ong, the Tel and the Jonk joins it from right. Incorrect Solution (d) The Seonath, the Hasdeo, the Mand and the Ib joins Mahanadi from left. The Ong, the Tel and the Jonk joins it from right. Question 15 of 35 15. Question ‘River Yamuna’ passes through which of the following states? Uttarakhand Punjab Uttar Pradesh Haryana Select the correct code: a) 1, 2 and 3 b) 2, 3 and 4 c) 1, 3 and 4 d) 1, 2 and 4 Correct Solution (c) River Yamuna passes through Uttarakhand, Himachal Pradesh, Uttar Pradesh, Haryana, Delhi. Incorrect Solution (c) River Yamuna passes through Uttarakhand, Himachal Pradesh, Uttar Pradesh, Haryana, Delhi. Question 16 of 35 16. Question Which of the following ‘Panch Prayag’s’ are correctly matched? Karnaprayag: Pindar and Alaknanda Rudraprayag: Nandakini and Alaknanda Nandaprayag: Mandakini and Alaknanda Vishnuprayag: Dhauliganga and Alaknanda Choose correct option from below: a) 1 only b) 1 and 2 only c) 1 and 4 only d) 3 and 4 only Correct Solution (c) The five confluences, known as the Panch Prayag, are all along the Alaknanda. They are, in downstream order: Vishnuprayag, where the Dhauliganga joins the Alaknanda; Nandprayag, where the Nandakini joins; Karnaprayag, where the Pindar joins, Rudraprayag, where the Mandakini joins; Devprayag, where the Bhagirathi joins the Alaknanda to form the Ganges River proper. Incorrect Solution (c) The five confluences, known as the Panch Prayag, are all along the Alaknanda. They are, in downstream order: Vishnuprayag, where the Dhauliganga joins the Alaknanda; Nandprayag, where the Nandakini joins; Karnaprayag, where the Pindar joins, Rudraprayag, where the Mandakini joins; Devprayag, where the Bhagirathi joins the Alaknanda to form the Ganges River proper. Question 17 of 35 17. Question With reference to the Peninsular rivers, consider the following statements: They are characterized by trellis and radial drainage patterns. They have larger catchment area than the Himalayan rivers Which of the statements given above is/are correct? a) 1 only b) 2 only c) Both 1 and 2 d) Neither 1 nor 2 Correct Solution (a) Statement-1: Correct Statement-2: Incorrect Peninsular rivers are characterized by trellis, radial and rectangular drainage patterns while Himalayan rivers are characterized by dendritic pattern in plains Himalayan rivers have larger catchment area than the peninsular rivers Incorrect Solution (a) Statement-1: Correct Statement-2: Incorrect Peninsular rivers are characterized by trellis, radial and rectangular drainage patterns while Himalayan rivers are characterized by dendritic pattern in plains Himalayan rivers have larger catchment area than the peninsular rivers Question 18 of 35 18. Question Consider the following statements River Sone has its origin in the Pachmarhi Biosphere Reserve. It flows through Madhya Pradesh and Bihar only. Which of the above statements is/are correct? a) 1 only b) 2 only c) Both 1 and 2 d) Neither 1 nor 2 Correct Solution (d) Statement 1 Statement 2 Incorrect Incorrect The Sone originates near Amarkantak in Anuppur district of Madhya Pradesh, just east of the headwater of the Narmada River, and flows north-northwest through Shahdol district in Madhya Pradesh state before turning sharply eastward where it encounters the southwest-northeast-Kaimur Range. The Sone parallels the Kaimur hills, flowing east-northeast through Uttar Pradesh, Jharkhand and Bihar states to join the Ganges just west of Patna. Incorrect Solution (d) Statement 1 Statement 2 Incorrect Incorrect The Sone originates near Amarkantak in Anuppur district of Madhya Pradesh, just east of the headwater of the Narmada River, and flows north-northwest through Shahdol district in Madhya Pradesh state before turning sharply eastward where it encounters the southwest-northeast-Kaimur Range. The Sone parallels the Kaimur hills, flowing east-northeast through Uttar Pradesh, Jharkhand and Bihar states to join the Ganges just west of Patna. Question 19 of 35 19. Question Consider the following statements about types of Delta Arcuate delta results when light depositions give rise to shallow, shifting distributaries and a general fan-shaped profile. Bird’s Foot Delta emerges when limestone sediment deposits do not allow downward seepage of water. Lobate Delta develops when fluvial action and depositional process dominate the system. How many of the statements given above is/are correct? a) Only one b) Only two c) All three d) None Correct Solution (c) Statement 1 Statement 2 Statement 3 Correct Correct Correct Arcuate or Fan-shaped (Curved) This type of delta results when light depositions give rise to shallow, shifting distributaries and a general fan-shaped profile. Examples: Nile, Ganga, Indus Bird’s Foot Delta (Elongated) This type of delta emerges when limestone sediment deposits do not allow downward seepage of water. The distributaries seem to be flowing over projections of these deposits which appear as a bird’s foot. Example: Mississippi River High-constructive deltas – Elongate and Lobate Delta Develops when fluvial action and depositional process dominate the system. The Godavari River represents lobate delta. Incorrect Solution (c) Statement 1 Statement 2 Statement 3 Correct Correct Correct Arcuate or Fan-shaped (Curved) This type of delta results when light depositions give rise to shallow, shifting distributaries and a general fan-shaped profile. Examples: Nile, Ganga, Indus Bird’s Foot Delta (Elongated) This type of delta emerges when limestone sediment deposits do not allow downward seepage of water. The distributaries seem to be flowing over projections of these deposits which appear as a bird’s foot. Example: Mississippi River High-constructive deltas – Elongate and Lobate Delta Develops when fluvial action and depositional process dominate the system. The Godavari River represents lobate delta. Question 20 of 35 20. Question Which of the above pairs are correctly matched? Rivers                                                Dams The Uri Dam                                        River Indus The Salal Dam                                    River Chenab The Chamera dam                              River Sutlej The Tulbul Project                              River Jhelum Choose the correct code: a) 1 and 3 b) 1, 2, and 3 c) 2 and 4 d) 1, 2, 3 and 4 Correct Solution (c) Match 1 Match 2 Match 3 Match 4 Incorrect Correct Incorrect Correct The Uri Dam– The dam has been constructed on the river Jhelum in the state of Jammu and Kashmir.   The Salal Dam– The dam has been constructed on the river Chenab at Dhyangarh in the state of Jammu and Kashmir.   The Chamera Dam– The dam has been constructed on the river Ravi in Chamba district, Himachal Pradesh. The dam fulfils the hydroelectric needs of the region.   The Tulbul Project– The project has been commissioned on the river Jhelum in the state of Jammu and Kashmir. Wular Barrage on the Wular Lake(a Ramsar wetland site) is constructed.   Incorrect Solution (c) Match 1 Match 2 Match 3 Match 4 Incorrect Correct Incorrect Correct The Uri Dam– The dam has been constructed on the river Jhelum in the state of Jammu and Kashmir.   The Salal Dam– The dam has been constructed on the river Chenab at Dhyangarh in the state of Jammu and Kashmir.   The Chamera Dam– The dam has been constructed on the river Ravi in Chamba district, Himachal Pradesh. The dam fulfils the hydroelectric needs of the region.   The Tulbul Project– The project has been commissioned on the river Jhelum in the state of Jammu and Kashmir. Wular Barrage on the Wular Lake(a Ramsar wetland site) is constructed.   Question 21 of 35 21. Question Consider the following statements: The International Civil Aviation Organization (ICAO)has established the Carbon Offsetting and Reduction Scheme for International Aviation (CORSIA) to address aviation emissions. TheWorld Economic Forum has launched the Clean Skies for Tomorrow initiative, which aims to accelerate the production and use of Sustainable Aviation Fuel (SAF). Choose the correct code: a) 1 only b) 2 only c) Both 1 and 2 d) Neither 1 nor 2 Correct Solution (c) Sustainable Aviation Fuel (SAF), also referred to as bio-jet fuel, is created using domestically developed methods using cooking oil and oil-rich seeds from plants. Scaling up the production and use of SAF in India can bring several benefits, including reducing GHG emissions, improving air quality, enhancing energy security, creating jobs in the renewable energy sector, and promoting sustainable development. The International Civil Aviation Organization (ICAO) has established the Carbon Offsetting and Reduction Scheme for International Aviation (CORSIA) to address aviation emissions. Hence statement 1 is correct. The World Economic Forum has launched the Clean Skies for Tomorrow initiative, which aims to accelerate the production and use of Sustainable Aviation Fuel (SAF). Hence statement 2 is correct. Incorrect Solution (c) Sustainable Aviation Fuel (SAF), also referred to as bio-jet fuel, is created using domestically developed methods using cooking oil and oil-rich seeds from plants. Scaling up the production and use of SAF in India can bring several benefits, including reducing GHG emissions, improving air quality, enhancing energy security, creating jobs in the renewable energy sector, and promoting sustainable development. The International Civil Aviation Organization (ICAO) has established the Carbon Offsetting and Reduction Scheme for International Aviation (CORSIA) to address aviation emissions. Hence statement 1 is correct. The World Economic Forum has launched the Clean Skies for Tomorrow initiative, which aims to accelerate the production and use of Sustainable Aviation Fuel (SAF). Hence statement 2 is correct. Question 22 of 35 22. Question Consider the following pairs: Volcano Country Marapi Volcano Japan Mount Etna Italy Mauna Loa Hawaii Sakurajima Volcano Indonesia How many pairs are correctly matched? a) One pair b) Two pairs c) Three pairs d) Four pairs Correct Solution (b) Volcano Country Marapi Volcano Indonesia Mount Etna Italy Mauna Loa Hawaii Sakurajima Volcano Japan Hence option b is correct. Incorrect Solution (b) Volcano Country Marapi Volcano Indonesia Mount Etna Italy Mauna Loa Hawaii Sakurajima Volcano Japan Hence option b is correct. Question 23 of 35 23. Question Consider the following statements about the Biodiversity Heritage Sites (BHS): They arerecognised under the Environment Protection Act of 1986. They are notified by the central government in consultation with the state government. Which of the statements given above is/are correct? a) 1 only b) 2 only c) Both 1 and 2 d) Neither 1 nor 2 Correct Solution (d) The Biodiversity Heritage Sites (BHS) are areas that are unique, ecologically fragile ecosystems that have a rich biodiversity. BHS must comprise any one or more of the following components – species richness, high endemism, presence of rare, endemic, and threatened species, keystone species, species of evolutionary significance, wild ancestors of domestic/cultivated species, etc. They are recognised under Section 37 of the Biological Diversity Act, of 2002. Hence statement 1 is incorrect. The creation of BHS may not put any restriction on the prevailing practices and usages of the local communities, other than those voluntarily decided by them. The purpose is to enhance the quality of life of the local communities through conservation measures. They are notified by the State Government in consultation with the local bodies. Hence statement 2 is incorrect. Nallur Tamarind Grove in Bengaluru, Karnataka was the first Biodiversity Heritage Site of India, declared in 2007. Incorrect Solution (d) The Biodiversity Heritage Sites (BHS) are areas that are unique, ecologically fragile ecosystems that have a rich biodiversity. BHS must comprise any one or more of the following components – species richness, high endemism, presence of rare, endemic, and threatened species, keystone species, species of evolutionary significance, wild ancestors of domestic/cultivated species, etc. They are recognised under Section 37 of the Biological Diversity Act, of 2002. Hence statement 1 is incorrect. The creation of BHS may not put any restriction on the prevailing practices and usages of the local communities, other than those voluntarily decided by them. The purpose is to enhance the quality of life of the local communities through conservation measures. They are notified by the State Government in consultation with the local bodies. Hence statement 2 is incorrect. Nallur Tamarind Grove in Bengaluru, Karnataka was the first Biodiversity Heritage Site of India, declared in 2007. Question 24 of 35 24. Question Consider the following statements about the Arsenic and Metal Removal by Indian Technology (AMRIT): It is a water purifier developed for the removal of arsenic and metal ions from water. It can be used for both domestic and community levels. It is developed by the Ministry of Jal Shakti. How many of the above statements are correct? a) Only one b) Only two c) All three d) None Correct Solution (b) The Arsenic and Metal Removal by Indian Technology (AMRIT) is a water purifier developed for the removal of arsenic and metal ions from water. Hence statement 1 is correct. The technology uses nano-scale iron oxy-hydroxide which selectively removes arsenic when water is passed through it. It can be used for both domestic and community levels. Hence statement 2 is correct. It was developed by the Indian Institute of Technology (IIT), Madras. Hence statement 3 is incorrect.   Incorrect Solution (b) The Arsenic and Metal Removal by Indian Technology (AMRIT) is a water purifier developed for the removal of arsenic and metal ions from water. Hence statement 1 is correct. The technology uses nano-scale iron oxy-hydroxide which selectively removes arsenic when water is passed through it. It can be used for both domestic and community levels. Hence statement 2 is correct. It was developed by the Indian Institute of Technology (IIT), Madras. Hence statement 3 is incorrect.   Question 25 of 35 25. Question Consider the following statements about Kawal Tiger Reserve: It forms part of the Deccan peninsula-central highlands. It has the catchment for the rivers Narmada and Tapi. It has connectivity to the Tadoba-Andhari Tiger Reserve of Maharashtra. Its vegetation includes southern tropical dry deciduous forests. How many of the above statements are correct? a) Only one b) Only two c) Only three d) All four Correct Solution (c) Kawal Tiger Reserve forms part of the Deccan peninsula-central highlands. Hence statement 1 is correct. It is located in the north-eastern part of Telangana, with the Godavari River on one side and the Maharashtra border on the other. It has the catchment for the rivers Godavari and Kadam, which flow towards the south of the sanctuary. Hence statement 2 is incorrect. It has connectivity to the Tadoba-Andhari Tiger Reserve of Maharashtra in the North and to the Indravati Tiger Reserve of Chhattisgarh towards its north-eastern side. Hence statement 3 is correct. It has diverse habitats, comprising dense forests, grasslands, open areas, rivers, streams, and water bodies. Its vegetation includes southern tropical dry deciduous forests. Hence statement 4 is correct. Incorrect Solution (c) Kawal Tiger Reserve forms part of the Deccan peninsula-central highlands. Hence statement 1 is correct. It is located in the north-eastern part of Telangana, with the Godavari River on one side and the Maharashtra border on the other. It has the catchment for the rivers Godavari and Kadam, which flow towards the south of the sanctuary. Hence statement 2 is incorrect. It has connectivity to the Tadoba-Andhari Tiger Reserve of Maharashtra in the North and to the Indravati Tiger Reserve of Chhattisgarh towards its north-eastern side. Hence statement 3 is correct. It has diverse habitats, comprising dense forests, grasslands, open areas, rivers, streams, and water bodies. Its vegetation includes southern tropical dry deciduous forests. Hence statement 4 is correct. Question 26 of 35 26. Question Consider the following statements about the Project Q*: It refers to anew artificial intelligence model being developed the OpenAI. It demonstrates advanced reasoning capabilities similar to humans. Virtual assistants and spam filters are examples of general artificial intelligence. Self-driving cars and advanced chatbots are examples of narrow artificial intelligence. Choose the correct code: a) Only one b) Only two c) Only three d) All four Correct Solution (b) Project Q* refers to a new artificial intelligence model being developed the OpenAI. Hence statement 1 is correct. It demonstrates advanced reasoning capabilities similar to humans. Hence statement 2 is correct. Virtual assistants and spam filters are examples of narrow artificial intelligence. Hence statement 3 is incorrect. Self-driving cars and advanced chatbots are examples of general artificial intelligence. Hence statement 4 is incorrect. Specifications Narrow AI General AI Learning ability Narrow AI focuses on a single task under various constraints and limitations. AGI is envisioned to have the capacity for generalization and adaptation across diverse domains. Operational domain Fixed domain models provided by programmers Self-learn and reasons with its operating environment Data processing Narrow AI analyses data by using ML, Natural Language Processing, Deep Learning, and Artificial Neural Networks General AI uses an advanced version of these technologies for analysis. Limitation Limited to a single task, lacks broader intelligence May lack depth in certain areas and have ethical concerns Incorrect Solution (b) Project Q* refers to a new artificial intelligence model being developed the OpenAI. Hence statement 1 is correct. It demonstrates advanced reasoning capabilities similar to humans. Hence statement 2 is correct. Virtual assistants and spam filters are examples of narrow artificial intelligence. Hence statement 3 is incorrect. Self-driving cars and advanced chatbots are examples of general artificial intelligence. Hence statement 4 is incorrect. Specifications Narrow AI General AI Learning ability Narrow AI focuses on a single task under various constraints and limitations. AGI is envisioned to have the capacity for generalization and adaptation across diverse domains. Operational domain Fixed domain models provided by programmers Self-learn and reasons with its operating environment Data processing Narrow AI analyses data by using ML, Natural Language Processing, Deep Learning, and Artificial Neural Networks General AI uses an advanced version of these technologies for analysis. Limitation Limited to a single task, lacks broader intelligence May lack depth in certain areas and have ethical concerns Question 27 of 35 27. Question Consider the following statements about the Cyber Surakshit Bharat Initiative: It is an initiative of the Ministry of Electronics and Information Technology (MeitY). It is an initiative to fortify the cyber security system in India with regard to the Government’s vision of a Digital India. It will be operated on three principles – education, awareness, and enablement. How many of the above statements are correct? a) Only one b) Only two c) All three d) None Correct Solution (c) The Cyber Surakshit Bharat Initiative is an initiative of the Ministry of Electronics and Information Technology (MeitY). Hence statement 1 is correct. It was launched in cooperation with the National e-Governance Division (NeGD) and various industry partners in India. It is an initiative to fortify the cyber security system in India with regard to the Government’s vision of a Digital India. Hence statement 2 is correct. It can be entitled as the first public-private enterprise of its kind. The partners involved in the origination of this scheme include chief IT companies like Intel, Microsoft, etc. It will be operated on three principles – education, awareness, and enablement. Hence statement 3 is correct. It will comprise a program of awareness on the importance of cybersecurity. The scheme will also include a number of workshops on the best enablement and practices of the officials with cybersecurity health tool kits for the management and mitigation of cyber threats. Incorrect Solution (c) The Cyber Surakshit Bharat Initiative is an initiative of the Ministry of Electronics and Information Technology (MeitY). Hence statement 1 is correct. It was launched in cooperation with the National e-Governance Division (NeGD) and various industry partners in India. It is an initiative to fortify the cyber security system in India with regard to the Government’s vision of a Digital India. Hence statement 2 is correct. It can be entitled as the first public-private enterprise of its kind. The partners involved in the origination of this scheme include chief IT companies like Intel, Microsoft, etc. It will be operated on three principles – education, awareness, and enablement. Hence statement 3 is correct. It will comprise a program of awareness on the importance of cybersecurity. The scheme will also include a number of workshops on the best enablement and practices of the officials with cybersecurity health tool kits for the management and mitigation of cyber threats. Question 28 of 35 28. Question Consider the following statements about Stonehenge: It is a massive stone circle monument located on Salisbury Plain in southern England. It was declared a UNESCO World Heritage Site of outstanding universal value in 1986. Choose the correct code: a) 1 only b) 2 only c) Both 1 and 2 d) Neither 1 nor 2 Correct Solution (c) Stonehenge is a massive stone circle monument located on Salisbury Plain in southern England. Hence statement 1 is correct. Work started on this super stone circle around 5,000 years ago in the late Neolithic Age, but it took over 1,000 years to build in four long stages. Archaeologists believe the final changes were made around 1,500 BC, in the early Bronze Age. It was declared a UNESCO World Heritage Site of outstanding universal value in 1986. Hence statement 2 is correct. Though there is no definite evidence as to the intended purpose of Stonehenge, it was presumably a religious site and an expression of the power and wealth of the chieftains, aristocrats, and priests who had it built. Incorrect Solution (c) Stonehenge is a massive stone circle monument located on Salisbury Plain in southern England. Hence statement 1 is correct. Work started on this super stone circle around 5,000 years ago in the late Neolithic Age, but it took over 1,000 years to build in four long stages. Archaeologists believe the final changes were made around 1,500 BC, in the early Bronze Age. It was declared a UNESCO World Heritage Site of outstanding universal value in 1986. Hence statement 2 is correct. Though there is no definite evidence as to the intended purpose of Stonehenge, it was presumably a religious site and an expression of the power and wealth of the chieftains, aristocrats, and priests who had it built. Question 29 of 35 29. Question Which of the following products have obtained Geographical Indication from Meghalaya? Tawlhlohpuan Lakadong Turmeric Garo Dakmanda Larnai Pottery Choose the correct code: a) Only one b) Only two c) Only three d) All four Correct Solution (c) The following products have obtained Geographical Indication from Meghalaya Lakadong Turmeric Garo Dakmanda Larnai Pottery Garo Chubitchi Tawlhlohpuan – Mizoram Hence option c is correct.   Incorrect Solution (c) The following products have obtained Geographical Indication from Meghalaya Lakadong Turmeric Garo Dakmanda Larnai Pottery Garo Chubitchi Tawlhlohpuan – Mizoram Hence option c is correct.   Question 30 of 35 30. Question Consider the following statements about the Painganga River: It originates in the Ajantha ranges in Madhya Pradesh. It is a major tributary of the Wardha River. Its major tributaries include the Adan, Kas, and Arunavati. How many of the above statements are correct? a) Only one b) Only two c) All three d) None Correct Solution (b) The Painganga River originates in the Ajantha ranges in Maharashtra. Hence statement 1 is incorrect. It provides irrigation to the Washim and Yavatmal districts in Maharashtra. It is a major tributary of the Wardha River. Hence statement 2 is correct. The Wardha River flows into the Wain Ganga River to form the Pranhita River, which finally joins the Godavari River. Its major tributaries include the Adan, Kas, Kayadhu, Pus, and Arunavati. Hence statement 3 is correct. Two dams are being constructed on the river, namely Upper Painganga and Lower Painganga. This dam is also known as the Isapur Dam.   Incorrect Solution (b) The Painganga River originates in the Ajantha ranges in Maharashtra. Hence statement 1 is incorrect. It provides irrigation to the Washim and Yavatmal districts in Maharashtra. It is a major tributary of the Wardha River. Hence statement 2 is correct. The Wardha River flows into the Wain Ganga River to form the Pranhita River, which finally joins the Godavari River. Its major tributaries include the Adan, Kas, Kayadhu, Pus, and Arunavati. Hence statement 3 is correct. Two dams are being constructed on the river, namely Upper Painganga and Lower Painganga. This dam is also known as the Isapur Dam.   Question 31 of 35 31. Question The ratio of Adam’s age 4 years ago and Eve’s age after 4 years is 1: 1. Presently, the ratio of their ages is 4 : 3. Find the ratio between Adam’s age 4 years hence and Eve’s age 4 years ago. a) 4: 3 b) 3: 4 c) 9: 5 d) 7: 3 Correct Solution (c) Currently, the ratio of their ages is 4: 3. Suppose, their ages are: 4x and 3x. Adam’s age 4 years ago = 4x – 4 Eve’s age after 4 years = 3x + 4 The ratio of Adam’s age 4 years ago and Eve’s age after 4 years is 1: 1 Therefore, = 4x−4/(3x+4) = 1/1 Solving, we get x = 8 We are required to find the ratio between Adam’s age 4 years hence and Eve’s age 4 years ago. Adam’s age: (4x + 4) Eve’s age: (3x – 4) Putting the value of x, we get: = 4x+4/ (3x−4) = [(4×8)+4] [(3×8)−4] = 9/5 So, the required ratio is 9: 5   Incorrect Solution (c) Currently, the ratio of their ages is 4: 3. Suppose, their ages are: 4x and 3x. Adam’s age 4 years ago = 4x – 4 Eve’s age after 4 years = 3x + 4 The ratio of Adam’s age 4 years ago and Eve’s age after 4 years is 1: 1 Therefore, = 4x−4/(3x+4) = 1/1 Solving, we get x = 8 We are required to find the ratio between Adam’s age 4 years hence and Eve’s age 4 years ago. Adam’s age: (4x + 4) Eve’s age: (3x – 4) Putting the value of x, we get: = 4x+4/ (3x−4) = [(4×8)+4] [(3×8)−4] = 9/5 So, the required ratio is 9: 5   Question 32 of 35 32. Question There is a Hockey match of Dhyanchand tomorrow, at Motera stadium. In recent years, it has rained only 73 days each year. Unfortunately, the weatherman has predicted rain for tomorrow. When it actually rains, the weatherman correctly forecasts rain 80% of the time. When it doesn’t rain, he incorrectly forecasts rain 20% of the time. What is the probability that it will rain on the day of Dhyanchand’s Hockey match? a) 1/5 b) 1/3 c) 1/2 d) 1/4 Correct Solution (c) The sample space is defined by two mutually-exclusive events – it rains or it does not rain. Additionally, a third event occurs when the weatherman predicts rain. Notation for these events appears below. Event A1 It rains on Dhyanchand’s Hockey match. Event A2 It does not rain on Dhyanchand’s Hockey match Event B The weatherman predicts rain. In terms of probabilities, we know the following: P (A1) = 73/365 = 0.2 [It rains 73 days out of the year.] P (A2) = 292/365 = 0.8 [It does not rain 292 days out of the year.] P (B|A1) = 0.8 [When it rains, the weatherman predicts rain 80% of the time.] P (B|A2) = 0.2 [When it does not rain, the weatherman predicts rain 20% of the time.] We want to know P (A1 |B), the probability it will rain on the day of Dhyanchand’s Hockey match, given a forecast for rain by the weatherman. The answer can be determined from Bayes’ theorem, as shown below. P(A1 |B) = [P(A1 ) P(B|A1 )] [P(A1 )P(B|A1 )+ P(A2 )P(B|A2 )] P(A1 |B) = [0.2 × 0.8] [(0.2 × 0.8)+ (0.8 × 0.2)] = 0.2 × 0.8 2(0.2 × 0.8) = ½ Incorrect Solution (c) The sample space is defined by two mutually-exclusive events – it rains or it does not rain. Additionally, a third event occurs when the weatherman predicts rain. Notation for these events appears below. Event A1 It rains on Dhyanchand’s Hockey match. Event A2 It does not rain on Dhyanchand’s Hockey match Event B The weatherman predicts rain. In terms of probabilities, we know the following: P (A1) = 73/365 = 0.2 [It rains 73 days out of the year.] P (A2) = 292/365 = 0.8 [It does not rain 292 days out of the year.] P (B|A1) = 0.8 [When it rains, the weatherman predicts rain 80% of the time.] P (B|A2) = 0.2 [When it does not rain, the weatherman predicts rain 20% of the time.] We want to know P (A1 |B), the probability it will rain on the day of Dhyanchand’s Hockey match, given a forecast for rain by the weatherman. The answer can be determined from Bayes’ theorem, as shown below. P(A1 |B) = [P(A1 ) P(B|A1 )] [P(A1 )P(B|A1 )+ P(A2 )P(B|A2 )] P(A1 |B) = [0.2 × 0.8] [(0.2 × 0.8)+ (0.8 × 0.2)] = 0.2 × 0.8 2(0.2 × 0.8) = ½ Question 33 of 35 33. Question A four-digit number N is of the form XYZZ. Each letter stands for a digit. The successor of N is of the form XPQQ. The predecessor of N is of the form XYZU then the value of Z + Q + U? a) 12 b) 15 c) 17 d) 21 Correct Solution (b) The above given conditions are possible only when U = 8, Z = 9 and Q = 0. Sum of these digits is 17. Incorrect Solution (b) The above given conditions are possible only when U = 8, Z = 9 and Q = 0. Sum of these digits is 17. Question 34 of 35 34. Question Find the least value of “b” for which 67b326 is divisible by 3? a) 3 b) 0 c) 2 d) 1 Correct Solution (b) The required number is b. Then, = 6+7+b+3+2+6=24+b is divisible by 3. Therefore, least value of b is 0.   Incorrect Solution (b) The required number is b. Then, = 6+7+b+3+2+6=24+b is divisible by 3. Therefore, least value of b is 0.   Question 35 of 35 35. Question The sum of the digits of a two-digit number is 11. If 45 is added to the number, then the digits are reversed. Find the number. a) 65 b) 56 c) 38 d) 74 Correct Solution (c) Given x + y = 11………(i) 10x + y + 45 = 10y + x ……….. (ii) From equation (i) and (ii), we get X = 3 and y =8 So, the number is 38.   Incorrect Solution (c) Given x + y = 11………(i) 10x + y + 45 = 10y + x ……….. (ii) From equation (i) and (ii), we get X = 3 and y =8 So, the number is 38.   window.wpProQuizInitList = window.wpProQuizInitList || []; window.wpProQuizInitList.push({ id: '#wpProQuiz_3619', init: { quizId: 3619, mode: 1, globalPoints: 70, timelimit: 1800, resultsGrade: [0], bo: 704, qpp: 0, catPoints: [70], formPos: 0, lbn: "Test-summary", json: {"31840":{"type":"single","id":31840,"catId":0,"points":2,"correct":[1,0,0,0]},"31843":{"type":"single","id":31843,"catId":0,"points":2,"correct":[0,0,0,1]},"31846":{"type":"single","id":31846,"catId":0,"points":2,"correct":[0,1,0,0]},"31848":{"type":"single","id":31848,"catId":0,"points":2,"correct":[0,0,0,1]},"31849":{"type":"single","id":31849,"catId":0,"points":2,"correct":[0,0,0,1]},"31852":{"type":"single","id":31852,"catId":0,"points":2,"correct":[1,0,0,0]},"31853":{"type":"single","id":31853,"catId":0,"points":2,"correct":[0,1,0,0]},"31855":{"type":"single","id":31855,"catId":0,"points":2,"correct":[1,0,0,0]},"31858":{"type":"single","id":31858,"catId":0,"points":2,"correct":[0,1,0,0]},"31859":{"type":"single","id":31859,"catId":0,"points":2,"correct":[0,0,0,1]},"31861":{"type":"single","id":31861,"catId":0,"points":2,"correct":[0,1,0,0]},"31864":{"type":"single","id":31864,"catId":0,"points":2,"correct":[0,0,1,0]},"31867":{"type":"single","id":31867,"catId":0,"points":2,"correct":[1,0,0,0]},"31870":{"type":"single","id":31870,"catId":0,"points":2,"correct":[0,0,0,1]},"31871":{"type":"single","id":31871,"catId":0,"points":2,"correct":[0,0,1,0]},"31874":{"type":"single","id":31874,"catId":0,"points":2,"correct":[0,0,1,0]},"31877":{"type":"single","id":31877,"catId":0,"points":2,"correct":[1,0,0,0]},"31878":{"type":"single","id":31878,"catId":0,"points":2,"correct":[0,0,0,1]},"31879":{"type":"single","id":31879,"catId":0,"points":2,"correct":[0,0,1,0]},"31882":{"type":"single","id":31882,"catId":0,"points":2,"correct":[0,0,1,0]},"31883":{"type":"single","id":31883,"catId":0,"points":2,"correct":[0,0,1,0]},"31886":{"type":"single","id":31886,"catId":0,"points":2,"correct":[0,1,0,0]},"31887":{"type":"single","id":31887,"catId":0,"points":2,"correct":[0,0,0,1]},"31889":{"type":"single","id":31889,"catId":0,"points":2,"correct":[0,1,0,0]},"31891":{"type":"single","id":31891,"catId":0,"points":2,"correct":[0,0,1,0]},"31893":{"type":"single","id":31893,"catId":0,"points":2,"correct":[0,1,0,0]},"31894":{"type":"single","id":31894,"catId":0,"points":2,"correct":[0,0,1,0]},"31896":{"type":"single","id":31896,"catId":0,"points":2,"correct":[0,0,1,0]},"31897":{"type":"single","id":31897,"catId":0,"points":2,"correct":[0,0,1,0]},"31898":{"type":"single","id":31898,"catId":0,"points":2,"correct":[0,1,0,0]},"31901":{"type":"single","id":31901,"catId":0,"points":2,"correct":[0,0,1,0]},"31902":{"type":"single","id":31902,"catId":0,"points":2,"correct":[0,0,1,0]},"31903":{"type":"single","id":31903,"catId":0,"points":2,"correct":[0,1,0,0]},"31904":{"type":"single","id":31904,"catId":0,"points":2,"correct":[0,1,0,0]},"31907":{"type":"single","id":31907,"catId":0,"points":2,"correct":[0,0,1,0]}} } }); All the Best IASbaba

DAILY CURRENT AFFAIRS IAS | UPSC Prelims and Mains Exam – 8th April 2024

Archives (PRELIMS & MAINS Focus)   Netaji Subhash Chandra Bose Island Syllabus Prelims – Geography Context: The unchecked population growth of spotted deer (Chital) is creating problems in Netaji Subhash Chandra Boss Island. Background:- Introduced to the Andaman and Nicobar Islands for game hunting in the early 1900s by the British, the herbivore multiplied unchecked for years in the absence of large predators.It has become an “invasive” problem for the Union Territory’s authorities. About  Netaji Subhash Chandra Bose Island Earlier Ross Island, now officially known as Netaji Subhash Chandra Bose Island, is an island of the Andaman Islands. The Island is situated three kilometres East of Central Port Blair. It belongs to the South Andaman administrative district, Andaman and Nicobar Islands. The Andaman and Nicobar Islands are strategically located in the Indian Ocean between the Indian mainland, Myanmar, and Indonesia, and are close to important maritime routes. The islands were first controlled by the Dutch, then by the British, and were taken over by the advancing Japanese military during World War II. The Azad Hind government was able to obtain de jure control of the islands from the Japanese by the end of 1943, and Subhash Chandra Bose arrived in Port Blair on December 29. Keeping in mind the historical significance of the Andaman & Nicobar Islands and to honour the memory of Netaji Subhas Chandra Bose, Ross Islands was renamed as Netaji Subhash Chandra Bose Dweep by Prime Minister during his visit to the Island in 2018. Neil Island and Havelock Island were also renamed Shaheed Dweep and Swaraj Dweep. All the three islands are major tourist spots. Source: Indian Express Green Hydrogen Syllabus Prelims & Mains – Environment Context: The Ministry of New and Renewable Energy (MNRE) has announced a Rs-496-crore (until 2025-26) scheme to support pilot projects that either test the viability of green hydrogen as a vehicle fuel or develop secure supporting infrastructure such as refuelling stations. Background: Big Indian commercial vehicle manufacturers such as Tata Motors, Volvo Eicher, and Ashok Leyland are doubling down on efforts to develop hydrogen-powered trucks and buses by ramping up research and development, and building manufacturing capacities. About Green Hydrogen : Hydrogen is colourless, and green hydrogen is ‘green’ only by virtue of the way it is produced, and the source of the energy used to manufacture it. Green hydrogen refers to hydrogen that is produced from the electrolysis of water — splitting it into hydrogen and oxygen — using an electrolyser powered by renewable energy.This is considered to be a virtually emission-free pathway for hydrogen production — it is ‘end-to-end’ green because it is powered by green energy, uses water as feedstock, and emits no carbon on consumption. Currently, most hydrogen produced for industrial consumption and applications is ‘grey’ hydrogen, which is produced from natural gas through energy-intensive processes, and has high carbon emissions.Except for a difference in the production pathway and emissions, green hydrogen is essentially the same as grey — or hydrogen categorised by any other colour. Challenges to the large-scale use of green hydrogen in the transportation sector: The foremost among challenges is the prohibitive cost of production, followed by challenges of storage and transportation at scale. Green hydrogen-powered vehicles are not yet seen as a suitable alternative to four-wheel battery electric vehicles (BEVs) due to challenges arising from fuel costs and building supporting infrastructure. Hydrogen is extremely flammable, which means that special care would be needed in handling the fuel at retail stations compared to diesel, petrol, or even CNG. Robust and fool-proof handling and safety standards need to be developed before pushing large-scale adoption. Currently, most cylinders manufactured in India are designed to carry compressed natural gas (CNG). But hydrogen is stored at a much higher pressure, and CNG cylinders cannot carry hydrogen. For cylinders to carry a high mass of hydrogen, the carbon fibre needs to be stronger, which makes high-pressure hydrogen cylinders expensive. This is a key barrier to the adoption of hydrogen as a transport fuel. For the same reason, the existing natural gas pipeline infrastructure is also not seen as viable. Source: Indian Express RAKHIGARHI Syllabus Prelims – History Context: The NCERT recently introduced certain revisions to the history syllabus of Class 12 students, highlighting that Harappans were based out in Rakhigarhi. Background: These revisions reflect a deeper understanding of the Harappan civilization and its historical context, shedding light on their origins and societal structures. About RAKHIGARHI Rakhigarhi is an archaeological site located in Hisar, Haryana, and is recognized as one of the largest Harappan (Indus Valley Civilization) sites in the Indian subcontinent. Rakhigarhi is situated in the plains of the Saraswati River, about 27 km from the seasonal Ghaggar river. The site showcases continuity from the Harappan age to the present times. It is recognized for its extensive Harappan heritage, offering insights into ancient urban life and culture. Excavations were carried out at Rakhigarhi to study its evolution from 6000 BC (Pre-Harappan phase) to 2500 BC. In order to study the genetic history of the Harappans, DNA was extracted from the skeletal remains excavated at Rakhigarhi. Previous Year Question Q1. Which of the following characterizes/characterize the people of Indus Civilization? They possessed great palaces and temples. They worshipped both male and female deities. They employed horse-drawn chariots in warfare. Select the correct statement/statements using the codes given below: 1 and 2 only 2 only 1, 2 and 3 None of the statements given above is correct Source: Hindu DEMOGRAPHIC DIVIDEND Syllabus Prelims – Economy Context: The World Bank warned that South Asian nations run the risk of “squandering its demographic dividend” because job creation is not keeping up with the growth in the working-age population. Background: According to data in the report, between 2000 and 2023, When the working-age population grew by an average of 19 million per year, the region added 10 million employments annually on average. About DEMOGRAPHIC DIVIDEND: The United Nations Population Fund defines demographic dividend as “the economic growth potential that can result from shifts in a population’s age structure, mainly when the share of the working-age population is larger than the non-working-age share of the population”. The demographic dividend in India presents various opportunities related to economic growth due to increased economic activities from a higher working-age population and lower dependent population. Features of India’s Demographic Dividend: Large and Growing Working-Age Population: The working-age population (15-64 years old) in India has surpassed the dependent population since 2018. Uniqueness: The window of opportunity for India’s demographic dividend spans five decades from 2005-06 to 2055-56, which is longer than any other country in the world. Increasing Education Levels: Education levels in India are rising, providing a more skilled and productive workforce. Increasing Gender Equality: There has been significant progress in recent years toward gender equality, including increasing levels of education and workforce participation among women. Source: Business Standard Previous Year Question India is regarded as a country of ‘Demographic Dividend’. This is due to: Its high population in the age group below 15 years Its high population in the age group of 15-64 years Its high population in the age group above 65 years Its high total population PROJECT AKASHTEER Syllabus Prelims – Current Event Context: The Indian Army has initiated the induction of control and reporting systems under ‘Project Akashteer’ to bolster its air defence capabilities. Background: Akashteer revolutionizes air defence operations, empowering India’s armed forces with cutting-edge capabilities. About PROJECT AKASHTEER: The ‘Project Akashteer’ is a significant initiative aimed at enhancing the air defence capabilities of the Indian Army. It is developed by Bharat Electronics Limited (BEL) as part of the ‘Atmanirbhar Bharat’ initiative. This project is designed to automate air defence control and reporting processes by digitizing them. The system integrates radar and communication systems at all levels into a unified network, providing an unprecedented level of situational awareness and control. This enables swift engagement of hostile targets, significantly reduces the risk of fratricide, and ensures the safety of friendly aircraft in contested airspace. A noteworthy aspect of ‘Akashteer’ is its emphasis on mobility and resilience. The system’s control centers, designed to be vehicle-based and mobile, can maintain operational capabilities even in challenging communication environments. The project is expected to significantly enhance the operational efficiency and integration of the Army’s air defence mechanisms. The total cost of the project is nearly Rs 1,982 crore. Source: Economic Times India – Italy Relations Syllabus Mains – GS 2 Context: The Union Cabinet had  approved the Migration and Mobility Agreement between India and Italy last December. Background: India and Italy are ancient civilisations with links going back 2,000 years. Italian port cities were important trading posts on the spice route. The Venetian merchant Marco Polo traveled to India in the 13th century and wrote about his experiences.After Independence, political relations between India and Italy were established in 1947. Since then, there has been a regular exchange of visits at political and official levels between both countries, including several visits by Heads of States. India and Italy Relations: There has been a regular exchange of visits at political and official levels between both countries. For instance, the Indian PM paid his first official visit to Italy in October 2021 to attend the G20 Summit. In March 2023, the Italian PM paid her first-ever state visit to India as the guest of honour of Raisina Dialogue. Italy is India’s 4th largest trading partner in the EU. The bilateral trade between the two countries was valued at US$ 14.25 billion in 2022-23, with the balance of trade being in India’s favour. India – Italy Military Cooperation Group (MCG) is a forum established to boost defence cooperation. India and Italy signed a defence cooperation agreement (in 2023) to promote cooperation in varied defence domains such as security and defence policy, and defence industrial cooperation among others. The relationship between India and Italy was elevated to a Strategic Partnership in March 2023 during the visit of the Italian Prime Minister to India. A connection was identified between the Indo-Pacific, in which India plays a leading role, and the enlarged Mediterranean where Italy sits at the centre of the Mediterranean Sea and acts as a natural bridge towards the Indo-Pacific. Italy’s decision to withdraw from the Belt and Road Initiative aligns with common goals in global connectivity between Rome and New Delhi. Italy supported India’s entry into the Wassenaar Arrangement and Australia Group. The Blue-Raman project (with an Italian company at its core) will bring the Mediterranean and the Indian Ocean closer in exchange for digital data through a submarine cable system. Italy supported India’s major initiatives including the International Solar Alliance (ISA), the India-Middle East-Europe Economic Corridor (IMEC), and the Global Biofuels Alliance (GBA). Concerns/Challenges/Issues in India-Italy Relations: India and Italy, despite having historical trade ties dating back to the Roman era have relatively low the current volume of trade and investment. Italian Mariners’ case sparked a conflict over legal jurisdiction and functional immunity, which ended only in 2020 with the verdict of the Permanent Court of Arbitration. In the Agusta Westland helicopter scandal, the defence firm owned by Italian company Leonardo was accused of bribery, resulting in India cancelling the procurement deal and banning Leonardo from the Indian defence market in 2015. In 2021, the India-Italy-Japan trilateral partnership was launched yet it has not been operationalised. Way Forward: Strengthen trade ties to increase trade volume, explore new avenues for investment, and promote economic cooperation. If the India and EU sign a trade pact in the coming time, Italy and India can enjoy major two-way trade benefits. It would also add fodder to the reinvigorated EU-India partnership. Italy and India can converge on multilateral issues, sharing a common imperative to shape and endorse a new global agenda together. Celebrate and promote the culinary heritage of both countries through food festivals and gastronomic events, with Italian pasta and pizza finding a place on Indian tables, and Indian spices adding a punch to Italian dishes. Source: MEA Practice MCQs Daily Practice MCQs Q1.) With reference to the ‘Project Akashteer’, consider the following statements: The ‘Project Akashteer’ is a significant initiative aimed at enhancing the air defence capabilities of the Indian Army. It is developed by Bharat Electronics Limited (BEL) as part of the ‘Atmanirbhar Bharat’ initiative. This project is designed to automate air defence control and reporting processes by digitizing them. How many of the statements given above are correct? Only one Only two All three None Q2.)  Consider the following: Growing working-age population Increasing education levels Increasing gender equality How many of the above are the features of India’s demographic dividend? Only one Only two All three None Q3.)With reference to Rakhigarhi, consider the following statements: Rakhigarhi is recognized as one of the largest Harappan sites in the Indian subcontinent. It is located in Rajasthan. The site showcases continuity from the Harappan age to the present times. How many of the statements given above are correct? Only one Only two All three None Comment the answers to the above questions in the comment section below!! ANSWERS FOR ’  8th April  2024 – Daily Practice MCQs’ will be updated along with tomorrow’s Daily Current Affairs.st ANSWERS FOR  6th April – Daily Practice MCQs Answers- Daily Practice MCQs Q.1) – d Q.2) – c Q.3) – b

Daily Prelims CA Quiz

UPSC Quiz – 2024 : IASbaba’s Daily Current Affairs Quiz 8th April 2024

For Previous Daily Quiz (ARCHIVES) – CLICK HERE The Current Affairs questions are based on sources like ‘The Hindu’, ‘Indian Express’ and ‘PIB’, which are very important sources for UPSC Prelims Exam. The questions are focused on both the concepts and facts. The topics covered here are generally different from what is being covered under ‘Daily Current Affairs/Daily News Analysis (DNA) and Daily Static Quiz’ to avoid duplication. The questions would be published from Monday to Saturday before 2 PM. One should not spend more than 10 minutes on this initiative. Gear up and Make the Best Use of this initiative. Do remember that, “the difference between Ordinary and EXTRA-Ordinary is PRACTICE!!” Important Note: Don’t forget to post your marks in the comment section. Also, let us know if you enjoyed today’s test 🙂After completing the 5 questions, click on ‘View Questions’ to check your score, time taken, and solutions.To take the Test Click Here

[DAY 30] 60 DAY RAPID REVISION (RaRe) SERIES for UPSC Prelims 2024 – GEOGRAPHY, CURRENT AFFAIRS & CSAT TEST SERIES!

Archives Hello Friends The 60 Days Rapid Revision (RaRe) Series is IASbaba’s Flagship Initiative recommended by Toppers and loved by the aspirants’ community every year. It is the most comprehensive program which will help you complete the syllabus, revise and practice tests on a daily basis. The Programme on a daily basis includes Daily Prelims MCQs from Static (Monday – Saturday) Daily Static Quiz will cover all the topics of static subjects – Polity, History, Geography, Economics, Environment and Science and technology. 20 questions will be posted daily and these questions are framed from the topics mentioned in the schedule. It will ensure timely and streamlined revision of your static subjects. Daily Current Affairs MCQs (Monday – Saturday) Daily 5 Current Affairs questions, based on sources like ‘The Hindu’, ‘Indian Express’ and ‘PIB’, would be published from Monday to Saturday according to the schedule. Daily CSAT Quiz (Monday – Friday) CSAT has been an Achilles heel for many aspirants. Daily 5 CSAT Questions will be published. Note – Daily Test of 20 static questions, 10 current affairs, and 5 CSAT questions. (35 Prelims Questions) in QUIZ FORMAT will be updated on a daily basis. To Know More about 60 Days Rapid Revision (RaRe) Series – CLICK HERE   60 Day Rapid Revision (RaRe) Series Schedule – CLICK HERE  Important Note Comment your Scores in the Comment Section. This will keep you accountable, responsible and sincere in days to come. It will help us come out with the Cut-Off on a Daily Basis. Let us know if you enjoyed today’s test 🙂  You can post your comments in the given format  (1) Your Score (2) Matrix Meter (3) New Learning from the Test Time limit: 0 Test-summary 0 of 35 questions completed Questions: 1 2 3 4 5 6 7 8 9 10 11 12 13 14 15 16 17 18 19 20 21 22 23 24 25 26 27 28 29 30 31 32 33 34 35 Information The following Test is based on the syllabus of 60 Days Plan-2023 for UPSC IAS Prelims 2022. To view Solutions, follow these instructions: Click on – ‘Start Test’ button Solve Questions Click on ‘Test Summary’ button Click on ‘Finish Test’ button Now click on ‘View Questions’ button – here you will see solutions and links. You have already completed the test before. Hence you can not start it again. Test is loading... You must sign in or sign up to start the test. You have to finish following test, to start this test: Results 0 of 35 questions answered correctly Your time: Time has elapsed You have scored 0 points out of 0 points, (0) Average score     Your score     Categories Not categorized 0% Your result has been entered into leaderboard Loading Name: E-Mail: Captcha: maximum of 70 points Pos. Name Entered on Points Result Table is loading No data available 1 2 3 4 5 6 7 8 9 10 11 12 13 14 15 16 17 18 19 20 21 22 23 24 25 26 27 28 29 30 31 32 33 34 35 Answered Review Question 1 of 35 1. Question In the context of coral reefs, consider the following statements Atolls are separated from the coastline by small and shallow lagoons. Barrier reefs run parallel to the coastline and a deep and wide lagoon is located between the coastline and the reef. Fringing reefs are shaped circularly and are surrounded by seas on all four sides and have shallow waters in the centre called a lagoon. How many of the above statements are correct? a) Only one b) Only two c) All three d) None Correct Solution (a) Statement 1 Statement 2 Statement 3 Incorrect Correct Incorrect Fringing reefs are separated from the coastline by small and shallow lagoons. They evolve and develop near the continent and remain close to the coastline. Barrier reefs run parallel to the coastline and a deep and wide lagoon is located between the coastline and the reef. They are found offshore on the continental shelf. Atolls are shaped circularly and are surrounded by seas on all four sides and have shallow waters in the centre called a lagoon. Incorrect Solution (a) Statement 1 Statement 2 Statement 3 Incorrect Correct Incorrect Fringing reefs are separated from the coastline by small and shallow lagoons. They evolve and develop near the continent and remain close to the coastline. Barrier reefs run parallel to the coastline and a deep and wide lagoon is located between the coastline and the reef. They are found offshore on the continental shelf. Atolls are shaped circularly and are surrounded by seas on all four sides and have shallow waters in the centre called a lagoon. Question 2 of 35 2. Question Which of the following is the correct order of sea ports of India from North to South? a) Haldia - Paradip - Mumbai - Kandla b) Haldia - Kandla - Mumbai - Paradip c) Kandla - Haldia - Paradip - Mumbai d) Kandla - Haldia – Mumbai - Paradip Correct Solution (c) Incorrect Solution (c) Question 3 of 35 3. Question With reference to the Continental Slope, consider the following statements It connects the continental shelf and the ocean basins. It is the gently sloping seaward extension of continental plate. Its boundary indicates the end of the continents and various reliefs like canyons and trenches are present here. How many of the above statements are correct? a) Only one b) Only two c) All three d) None Correct Solution (b) Statement 1 Statement 2 Statement 3 Correct Incorrect Correct The Continental Slope connects the continental shelf and the ocean basins. It begins where the bottom of the continental shelf sharply drops off into a steep slope. The gradient of the slope region varies between 2-5°. Its depth varies between 200 and 3,000 m. The Continental Shelf is the gently sloping seaward extension of continental plate. These extended margins of each continent are occupied by relatively shallow seas and gulfs. Continental Shelf is not gentle slope rather a sharp slope.   The Continental Slope boundary indicates the end of the continents and various reliefs like canyons and trenches are present here. Incorrect Solution (b) Statement 1 Statement 2 Statement 3 Correct Incorrect Correct The Continental Slope connects the continental shelf and the ocean basins. It begins where the bottom of the continental shelf sharply drops off into a steep slope. The gradient of the slope region varies between 2-5°. Its depth varies between 200 and 3,000 m. The Continental Shelf is the gently sloping seaward extension of continental plate. These extended margins of each continent are occupied by relatively shallow seas and gulfs. Continental Shelf is not gentle slope rather a sharp slope.   The Continental Slope boundary indicates the end of the continents and various reliefs like canyons and trenches are present here. Question 4 of 35 4. Question Consider the following statements Tsunamis are a series of waves which are generated by the movement of the sea floor only due to earthquakes. The Indian National Centre for Ocean Information System (INCOIS) through Indian Tsunami Early Warning Centre (ITEWC) is the nodal agency to provide tsunami advisories to India. Choose the correct code: a) 1 only b) 2 only c) Both 1 and 2 d) Neither 1 nor 2 Correct Solution (b) Statement 1 Statement 2 Incorrect Correct Tsunamis are a series of waves which are generated by the movement of the sea floor due to various geophysical phenomena such as earthquakes, landslides and volcanic eruptions.   The Indian National Centre for Ocean Information System INCOIS through Indian Tsunami Early Warning Centre (ITEWC) is the nodal agency to provide tsunami advisories to India.   The INCOIS was established in 1999 as an autonomous body under the Ministry of Earth Sciences. It is coordinating with the Disaster Management Officials (DMOs) for implementation of Tsunami Ready programme in India and conducts IOWave Tsunami mock exercises biannually to strengthen the readiness to handle the emergency situations with stakeholders.   Incorrect Solution (b) Statement 1 Statement 2 Incorrect Correct Tsunamis are a series of waves which are generated by the movement of the sea floor due to various geophysical phenomena such as earthquakes, landslides and volcanic eruptions.   The Indian National Centre for Ocean Information System INCOIS through Indian Tsunami Early Warning Centre (ITEWC) is the nodal agency to provide tsunami advisories to India.   The INCOIS was established in 1999 as an autonomous body under the Ministry of Earth Sciences. It is coordinating with the Disaster Management Officials (DMOs) for implementation of Tsunami Ready programme in India and conducts IOWave Tsunami mock exercises biannually to strengthen the readiness to handle the emergency situations with stakeholders.   Question 5 of 35 5. Question Consider the following statements regarding the distribution of water on the Earth’s surface The highest percentage of water is found in Ice Caps. The percentage of water in groundwater is more than lakes. The percentage of water in soil moisture is more than streams and rivers. How many of the above statements are correct? a) Only one b) Only two c) All three d) None Correct Solution (b) The distribution of water on the Earth’s surface is extremely uneven. Only 3% of water on the surface is fresh; the remaining 97% resides in the ocean. Of freshwater, 69% resides in glaciers, 30% underground, and less than 1% is located in lakes, rivers, and swamps. The below chart gives the exact details about it.   Incorrect Solution (b) The distribution of water on the Earth’s surface is extremely uneven. Only 3% of water on the surface is fresh; the remaining 97% resides in the ocean. Of freshwater, 69% resides in glaciers, 30% underground, and less than 1% is located in lakes, rivers, and swamps. The below chart gives the exact details about it.   Question 6 of 35 6. Question Consider the following statements about Ocean Mean Temperature (OMT) It is more stable and consistent when compared to Sea Mean Temperature. It is measured up to a depth of 26⁰C isotherm which is at depth of 300 meters. Which of the statements given above is/are correct? a) 1 only b) 2 only c) Both 1 and 2 d) Neither 1 nor 2 Correct Solution (a) Statement 1 Statement 2 Correct Incorrect Ocean Mean Temperature (OMT) is more stable and consistent when compared to Sea Surface Temperature (SST) because SST is restricted to few millimetres of top ocean layer which is largely influenced by strong winds and evaporation. OMT is measured up to a depth of 26⁰C isotherm which is at depth of 50-100 meters. OMT is analysed by measuring ocean thermal energy during the January – March period and it also predicts Indian summer monsoon. Incorrect Solution (a) Statement 1 Statement 2 Correct Incorrect Ocean Mean Temperature (OMT) is more stable and consistent when compared to Sea Surface Temperature (SST) because SST is restricted to few millimetres of top ocean layer which is largely influenced by strong winds and evaporation. OMT is measured up to a depth of 26⁰C isotherm which is at depth of 50-100 meters. OMT is analysed by measuring ocean thermal energy during the January – March period and it also predicts Indian summer monsoon. Question 7 of 35 7. Question Consider the following statements regarding South China Sea It is connected to the East China Sea through the Taiwan Strait. The nine-dash line lies in the South China Sea. It lies between China, Taiwan, the Philippines, Brunei, Malaysia, Indonesia and Vietnam. Which of the statements given above is/are correct? a) 1 and 2 b) 1 and 3 c) 2 and 3 d) All of the above Correct Solution (d) Statement 1 Statement 2 Statement 3 Correct Correct Correct South China Sea is connected to the East China Sea through the Taiwan Strait. The nine-dash line lies in the South China Sea. It lies between China, Taiwan, the Philippines, Brunei, Malaysia, Indonesia and Vietnam. It is bounded in the North by the shores of South China, in the West by the Indochinese Peninsula, in the East by the Islands of Taiwan and the North-Western Philippines, and in the South by Borneo, Eastern Sumatra, and the Bangka Belitung Islands. Incorrect Solution (d) Statement 1 Statement 2 Statement 3 Correct Correct Correct South China Sea is connected to the East China Sea through the Taiwan Strait. The nine-dash line lies in the South China Sea. It lies between China, Taiwan, the Philippines, Brunei, Malaysia, Indonesia and Vietnam. It is bounded in the North by the shores of South China, in the West by the Indochinese Peninsula, in the East by the Islands of Taiwan and the North-Western Philippines, and in the South by Borneo, Eastern Sumatra, and the Bangka Belitung Islands. Question 8 of 35 8. Question Consider the following statements Spring tides are formed when the sun, the moon and the earth are in a straight line. Neap tides are formed when the sun and moon are at right angles to each other. Which of the statements given above is/are correct? a) 1 only b) 2 only c) Both 1 and 2 d) Neither 1 nor 2 Correct Solution (c) Statement 1 Statement 2 Correct Correct Spring tides are formed when the sun, the moon and the earth are in a straight line. They occur twice a month, one on full moon period and another during new moon period. Neap tides are formed when the sun and moon are at right angles to each other. It also occurs twice and there is a seven day interval between the spring tides and neap tides. Incorrect Solution (c) Statement 1 Statement 2 Correct Correct Spring tides are formed when the sun, the moon and the earth are in a straight line. They occur twice a month, one on full moon period and another during new moon period. Neap tides are formed when the sun and moon are at right angles to each other. It also occurs twice and there is a seven day interval between the spring tides and neap tides. Question 9 of 35 9. Question Consider the following statements The Socotra Island is an island of Italy located in the Arctic Ocean. The Reunion Island is the overseas island of French located in the Indian Ocean. Choose the correct code: a) 1 only b) 2 only c) Both 1 and 2 d) Neither 1 nor 2 Correct Solution (b) Statement 1 Statement 2 Incorrect Correct The Socotra Island is an island of Yemen located between the Arabian Sea and the Guardafui Channel. The Reunion island is the overseas island of French located in the Indian Ocean.   Incorrect Solution (b) Statement 1 Statement 2 Incorrect Correct The Socotra Island is an island of Yemen located between the Arabian Sea and the Guardafui Channel. The Reunion island is the overseas island of French located in the Indian Ocean.   Question 10 of 35 10. Question Consider the following ocean currents Humboldt Gulf Stream Labrador Oyashio Which of the above are Cold Ocean currents? a) 3 and 4 only b) 1, 3 and 4 only c) 2, 3 and 4 only d) 1, 2, 3 and 4 Correct Solution (b) All the currents given in the question except the Gulf stream are cold currents Ocean currents are streams of water flowing constantly on the ocean surface in definite directions. The ocean currents may be warm or cold Generally, warm ocean currents originate near the equator and move towards the poles. The cold currents carry water from polar or higher latitudes to tropical or lower altitudes. The Labrador Ocean current, Humboldt(peruvian) and oyoshio current is cold while the Gulf Stream is a warm current. The ocean current influence the temperature conditions of the area. Warm currents bring about warm temperatures over the land surface. Incorrect Solution (b) All the currents given in the question except the Gulf stream are cold currents Ocean currents are streams of water flowing constantly on the ocean surface in definite directions. The ocean currents may be warm or cold Generally, warm ocean currents originate near the equator and move towards the poles. The cold currents carry water from polar or higher latitudes to tropical or lower altitudes. The Labrador Ocean current, Humboldt(peruvian) and oyoshio current is cold while the Gulf Stream is a warm current. The ocean current influence the temperature conditions of the area. Warm currents bring about warm temperatures over the land surface. Question 11 of 35 11. Question Consider the following statements with respect to Tides Tides are the rhythmic rise and fall of ocean water which generally happens twice in a day The strong gravitational pull exerted by the Sun and the Moon on the Earth’s surface causes the tides. During the full Moon and new Moon days, the Earth experiences Neap tides. How many of the above statements are correct? a) Only one b) Only two c) All three d) None Correct Solution (b) Statement 1 Statement 2 Statement 3 Correct Correct Incorrect The rhythmic rise and fall of ocean water twice a day is called a tide. It is high tide when water covers much of the shore by rising to its highest level. It is low tide when water falls to its lowest level and recedes from the shore. Thus it happens twice a day generally. The strong gravitational pull exerted by the Sun and the Moon on the Earth’s surface causes the tides. The water of the Earth closer to the Moon gets pulled under the influence of the Moon’s gravitational force and causes high tides. During the full Moon and new Moon days, the Sun, the Moon and the Earth are in the same line and the tides are highest. These tides are called spring tides. ( Not Neap tides) NEAP TIDES When the Moon is in its first and last quarter the ocean water gets drawn in diagonally opposite directions by the gravitational pull of the Sun and Moon resulting in low tides. These tides are Neap tides Incorrect Solution (b) Statement 1 Statement 2 Statement 3 Correct Correct Incorrect The rhythmic rise and fall of ocean water twice a day is called a tide. It is high tide when water covers much of the shore by rising to its highest level. It is low tide when water falls to its lowest level and recedes from the shore. Thus it happens twice a day generally. The strong gravitational pull exerted by the Sun and the Moon on the Earth’s surface causes the tides. The water of the Earth closer to the Moon gets pulled under the influence of the Moon’s gravitational force and causes high tides. During the full Moon and new Moon days, the Sun, the Moon and the Earth are in the same line and the tides are highest. These tides are called spring tides. ( Not Neap tides) NEAP TIDES When the Moon is in its first and last quarter the ocean water gets drawn in diagonally opposite directions by the gravitational pull of the Sun and Moon resulting in low tides. These tides are Neap tides Question 12 of 35 12. Question Consider the following statements with respect to Tidal energy Tidal energy can be best harnessed by building dams at narrow openings of the sea. During high tides the energy of the tides is used to turn the turbine installed in the dam to produce electricity The Gulf of Kachchh in India has a huge potential for harnessing Tidal energy How many of the above statements are correct? a) Only one b) Only two c) All three d) None Correct Solution (c) Statement 1 Statement 2 Statement 3 Correct Correct Correct Energy generated from tides is called tidal energy. Tidal energy can be harnessed by building dams at narrow openings in the sea.   During high tides, the energy of the tides is used to turn the turbine installed in the dam to produce electricity. Russia, France and the Gulf of Kachchh in India have huge tidal mill farms The Gulf of Kutch near Gujarat have an estimated potential of 7000 MW and 1200 MW, respectively and even the Gangetic delta in the Sundarbans of West Bengal has a potential of 100 MW Incorrect Solution (c) Statement 1 Statement 2 Statement 3 Correct Correct Correct Energy generated from tides is called tidal energy. Tidal energy can be harnessed by building dams at narrow openings in the sea.   During high tides, the energy of the tides is used to turn the turbine installed in the dam to produce electricity. Russia, France and the Gulf of Kachchh in India have huge tidal mill farms The Gulf of Kutch near Gujarat have an estimated potential of 7000 MW and 1200 MW, respectively and even the Gangetic delta in the Sundarbans of West Bengal has a potential of 100 MW Question 13 of 35 13. Question Which of the following is not a minor relief feature in the ocean? a) Seamount b) Trenches c) Guyots d) Submarine Canyons Correct Solution (b) Statement 1 Statement 2 Statement 3 Statement 4 Incorrect Correct Incorrect Incorrect SEAMOUNT It is a mountain with pointed summits, rising from the sea floor that does not reach the surface of the ocean. Seamounts are volcanic in origin. These can be 3,000-4,500m tall. The Emperor seamount, an extension of the Hawaiian Islands in the Pacific Ocean, is a good example. TRENCHES These areas are the deepest parts of the oceans. The trenches are relatively steep-sided, narrow basins. They are some 3-5 km deeper than the surrounding ocean floor. They occur at the bases of continental slopes and along island arcs and are associated with active volcanoes. This is a major relief feature in the ocean.     GUYOT Guyot is a flat-topped seamount. They show evidence of gradual subsidence through stages to become flat-topped submerged mountains. It is estimated that more than 10,000 seamounts and guyots exist in the Pacific Ocean alone. SUBMARINE CANYON These are deep valleys, some comparable to the Grand Canyon of the Colorado River. They are sometimes found cutting across the continental shelves and slopes, often extending from the mouths of large rivers. The Hudson Canyon is the best-known canyon in the world. Incorrect Solution (b) Statement 1 Statement 2 Statement 3 Statement 4 Incorrect Correct Incorrect Incorrect SEAMOUNT It is a mountain with pointed summits, rising from the sea floor that does not reach the surface of the ocean. Seamounts are volcanic in origin. These can be 3,000-4,500m tall. The Emperor seamount, an extension of the Hawaiian Islands in the Pacific Ocean, is a good example. TRENCHES These areas are the deepest parts of the oceans. The trenches are relatively steep-sided, narrow basins. They are some 3-5 km deeper than the surrounding ocean floor. They occur at the bases of continental slopes and along island arcs and are associated with active volcanoes. This is a major relief feature in the ocean.     GUYOT Guyot is a flat-topped seamount. They show evidence of gradual subsidence through stages to become flat-topped submerged mountains. It is estimated that more than 10,000 seamounts and guyots exist in the Pacific Ocean alone. SUBMARINE CANYON These are deep valleys, some comparable to the Grand Canyon of the Colorado River. They are sometimes found cutting across the continental shelves and slopes, often extending from the mouths of large rivers. The Hudson Canyon is the best-known canyon in the world. Question 14 of 35 14. Question Consider the following Ocean Currents Types 1. Peruvian Current Cold Current 2. Brazil Current Warm Current 3. Norwegian Current Cold Current 4. Benguela Current Warm Current How many pairs are correctly matched? a) Only one pair b) Two pairs c) Three pairs d) Four pairs Correct Solution (b) Blue arrow – cold current   Red arrow – warm current   Incorrect Solution (b) Blue arrow – cold current   Red arrow – warm current   Question 15 of 35 15. Question Which of the following factors affects the ocean salinity? Evaporation Precipitation Winds Ocean currents Select the correct answer using the code given below: a) 1, 2 and 4 only b) 2, 3 and 4 only c) 1, 2 and 3 only d) 1, 2, 3 and 4 Correct Solution (d) Statement 1 Statement 2 Statement 3 Statement 4 Correct Correct Correct Correct Evaporation of ocean water and formation of sea ice both increase the salinity of the ocean. However these “salinity raising” factors are continually counterbalanced by processes that decrease salinity such as the continuous input of fresh water from rivers, precipitation of rain and snow, and melting of ice.   Where more rain or snow falls over the ocean, it dilutes the salts in the seawater there. As a result, the water becomes fresher with time. If seawater becomes saltier, it may mean that rates of evaporation have increased or that precipitation has decreased over time. Winds also help in the redistribution of salts in the oceans and seas as winds drive away saline water to fewer saline areas resulting in a decrease of salinity in the former and increases in the latter. In other words, in the areas of upwelling of waterless saline water moves up from below whereas, in the areas where the water is piled up, salinity is increased. For example, trade winds drive away saline waters from the western coast of the continents and pile up salinity increases. Ocean currents affect the spatial distribution of salinity by mixing seawater. Equatorial warm currents drive away salts from the western coastal areas of the continent and accumulate them along with the eastern coastal areas. This adds to the high salinity of the Mexican Gulf. The North Atlantic drift, the extension of the gulf stream increases salinity along the north-western coasts of Europe. Similarly, salinity is reduced along the north-eastern coast. America due to cold labrador current. Ocean currents have the least impact on salinity in enclosed waters, but currents have a significant impact on salinity in marginal seas with open sea contact through wide openings. For example, the North Atlantic drift raises the salinity of the Norwegian and the North Seas. Incorrect Solution (d) Statement 1 Statement 2 Statement 3 Statement 4 Correct Correct Correct Correct Evaporation of ocean water and formation of sea ice both increase the salinity of the ocean. However these “salinity raising” factors are continually counterbalanced by processes that decrease salinity such as the continuous input of fresh water from rivers, precipitation of rain and snow, and melting of ice.   Where more rain or snow falls over the ocean, it dilutes the salts in the seawater there. As a result, the water becomes fresher with time. If seawater becomes saltier, it may mean that rates of evaporation have increased or that precipitation has decreased over time. Winds also help in the redistribution of salts in the oceans and seas as winds drive away saline water to fewer saline areas resulting in a decrease of salinity in the former and increases in the latter. In other words, in the areas of upwelling of waterless saline water moves up from below whereas, in the areas where the water is piled up, salinity is increased. For example, trade winds drive away saline waters from the western coast of the continents and pile up salinity increases. Ocean currents affect the spatial distribution of salinity by mixing seawater. Equatorial warm currents drive away salts from the western coastal areas of the continent and accumulate them along with the eastern coastal areas. This adds to the high salinity of the Mexican Gulf. The North Atlantic drift, the extension of the gulf stream increases salinity along the north-western coasts of Europe. Similarly, salinity is reduced along the north-eastern coast. America due to cold labrador current. Ocean currents have the least impact on salinity in enclosed waters, but currents have a significant impact on salinity in marginal seas with open sea contact through wide openings. For example, the North Atlantic drift raises the salinity of the Norwegian and the North Seas. Question 16 of 35 16. Question Consider the following terminologies associated with the water footprint Blue water footprint: It is the volume of freshwater that evaporated from rainwater store in the soil as soil moisture to produce the goods and services. Green water footprint: It is the volume of water evaporated from the groundwater during production of products. Grey water footprint: It is the volume of polluted water that associates with the production of all goods and services for the individual or community. How many of the above pairs are correctly matched? a) Only one b) Only two c) All three d) None Correct Solution (a) Statement 1 Statement 2 Statement 3 Incorrect Incorrect Correct Bluewater footprint: The blue water footprint is the volume of freshwater that evaporated from the global blue water resources (surface water and ground water) to produce the goods and services consumed by the individual or community.   Green water footprint: The green water footprint is the volume of water evaporated from the global green water resources (rainwater stored in the soil as soil moisture) during production or those incorporated in products. Grey water footprint: The grey water footprint is the volume of polluted water that associates with the production of all goods and services for the individual or community. The latter can be estimated as the volume of water that is required to dilute pollutants to such an extent that the quality of the water remains at or above agreed water quality standards. Incorrect Solution (a) Statement 1 Statement 2 Statement 3 Incorrect Incorrect Correct Bluewater footprint: The blue water footprint is the volume of freshwater that evaporated from the global blue water resources (surface water and ground water) to produce the goods and services consumed by the individual or community.   Green water footprint: The green water footprint is the volume of water evaporated from the global green water resources (rainwater stored in the soil as soil moisture) during production or those incorporated in products. Grey water footprint: The grey water footprint is the volume of polluted water that associates with the production of all goods and services for the individual or community. The latter can be estimated as the volume of water that is required to dilute pollutants to such an extent that the quality of the water remains at or above agreed water quality standards. Question 17 of 35 17. Question With reference to ocean currents consider the following statements: The best fishing grounds of the world exist in the regions where warm and cold currents meet. Ocean currents assist in maintaining the Earth’s heat balance. Which of the statements given above is/are correct? a) 1 only b) 2 only c) Both 1 and 2 d) Neither 1 nor 2 Correct Solution (c) Statement 1 Statement 2 Correct Correct The mixing of warm and cold currents helps to replenish the oxygen and favour the growth of plankton, the primary food for the fish population. The best fishing grounds in the world exist mainly in these mixing zones. Example Newfoundland in USA and off the coastlands of Japan Ocean currents have several direct and indirect influences on human activities. The West coasts of the continents in tropical and subtropical latitudes (except close to the equator) are bordered by cool waters. Their average temperatures are relatively low with narrow diurnal and annual ranges. There is fog, but generally, the areas are arid. West coasts of the continents in the middle and higher latitudes are bordered by warm waters which cause a distinct marine climate. They are characterized by cool summers and relatively mild winters with a narrow annual range of temperatures. Warm currents flow parallel to the east coasts of the continents in tropical and subtropical latitudes. This results in warm and rainy climates. These areas lie in the western margins of the subtropical anti-cyclones. Incorrect Solution (c) Statement 1 Statement 2 Correct Correct The mixing of warm and cold currents helps to replenish the oxygen and favour the growth of plankton, the primary food for the fish population. The best fishing grounds in the world exist mainly in these mixing zones. Example Newfoundland in USA and off the coastlands of Japan Ocean currents have several direct and indirect influences on human activities. The West coasts of the continents in tropical and subtropical latitudes (except close to the equator) are bordered by cool waters. Their average temperatures are relatively low with narrow diurnal and annual ranges. There is fog, but generally, the areas are arid. West coasts of the continents in the middle and higher latitudes are bordered by warm waters which cause a distinct marine climate. They are characterized by cool summers and relatively mild winters with a narrow annual range of temperatures. Warm currents flow parallel to the east coasts of the continents in tropical and subtropical latitudes. This results in warm and rainy climates. These areas lie in the western margins of the subtropical anti-cyclones. Question 18 of 35 18. Question With reference to vertical distribution of salinity of oceans, consider the following statements Salinity at surfaces is very much fixed Salinity at depths increases by the loss of water or evaporation In a distinct zone called the halocline the salinity increases sharply. How many of the above statements are correct? a) Only one b) Only two c) All three d) None Correct Solution (a) Statement 1 Statement 2 Statement 3 Incorrect Incorrect Correct Salinity at the surface increases by the loss of water to ice or evaporation or decreased by the input of fresh waters, such as from the rivers. Salinity at depth is very much fixed because there is no way that water is ‘lost’, or the salt is ‘added.’ There is a marked difference in the salinity between the surface zones and the deep zones of the oceans. The lower salinity water rests above the higher salinity dense water. Salinity, generally, increases with depth and there is a distinct zone called the halocline, where salinity increases sharply. Other factors being constant, increasing the salinity of seawater causes its density to increase. High-salinity seawater, generally, sinks below the lower-salinity water. This leads to stratification by salinity. Incorrect Solution (a) Statement 1 Statement 2 Statement 3 Incorrect Incorrect Correct Salinity at the surface increases by the loss of water to ice or evaporation or decreased by the input of fresh waters, such as from the rivers. Salinity at depth is very much fixed because there is no way that water is ‘lost’, or the salt is ‘added.’ There is a marked difference in the salinity between the surface zones and the deep zones of the oceans. The lower salinity water rests above the higher salinity dense water. Salinity, generally, increases with depth and there is a distinct zone called the halocline, where salinity increases sharply. Other factors being constant, increasing the salinity of seawater causes its density to increase. High-salinity seawater, generally, sinks below the lower-salinity water. This leads to stratification by salinity. Question 19 of 35 19. Question With reference to La Nina, consider the following statements: La Nina is a natural ocean-atmospheric phenomenon marked by cooler-than-average sea surface temperatures across the central and eastern Pacific Ocean. La Nina is one part of the El Nino Southern Oscillation (ENSO) cycle, which is characterized by opposing warm and cool phases of oceanic and atmospheric conditions in the tropical Pacific Ocean. The strongest La Niñas tend to be stronger than the strongest El Ninos. How many of the statements given above is/are correct? a) Only one b) Only two c) All three d) None Correct Solution (b) Statement 1 Statement 2 Statement 3 Correct Correct Incorrect La Nina is a natural ocean-atmospheric phenomenon marked by cooler-than average sea surface temperatures across the central and eastern Pacific Ocean near the equator and is translated from Spanish as “little girl.” La Nina is one part of the El Nino Southern Oscillation (ENSO) cycle, which is characterized by opposing warm and cool phases of oceanic and atmospheric conditions in the tropical Pacific Ocean. Consecutive La Ninas following a transition through ENSO neutral conditions are not uncommon and can be referred to as a “double-dip.” We see some subtle but important differences between El Niño and La Niña in terms of their patterns and behavior. First, the sea surface temperature anomalies (difference from the long-term average) during El Niño tend to be centered farther east than for La Niña, especially for the stronger El Niño episodes. Second, the strongest El Niños tend to be stronger than the strongest La Niñas. Incorrect Solution (b) Statement 1 Statement 2 Statement 3 Correct Correct Incorrect La Nina is a natural ocean-atmospheric phenomenon marked by cooler-than average sea surface temperatures across the central and eastern Pacific Ocean near the equator and is translated from Spanish as “little girl.” La Nina is one part of the El Nino Southern Oscillation (ENSO) cycle, which is characterized by opposing warm and cool phases of oceanic and atmospheric conditions in the tropical Pacific Ocean. Consecutive La Ninas following a transition through ENSO neutral conditions are not uncommon and can be referred to as a “double-dip.” We see some subtle but important differences between El Niño and La Niña in terms of their patterns and behavior. First, the sea surface temperature anomalies (difference from the long-term average) during El Niño tend to be centered farther east than for La Niña, especially for the stronger El Niño episodes. Second, the strongest El Niños tend to be stronger than the strongest La Niñas. Question 20 of 35 20. Question Consider the following statements about landforms formed by wave action. Bars are deposits of sand and gravel laid down by waves and currents which separate the shoreline from the sea. Tombolo is a deposition landform in which an island is attached to the mainland by a narrow piece of land such as a spit or bar. Which of the statements given above is/are correct? a) 1 only b) 2 only c) Both 1 and 2 d) Neither 1 nor 2 Correct Solution (c) Statement 1 Statement 2 Correct Correct Bars are deposits of sand and gravel laid down by waves and currents which separate the shoreline from the sea. They are submerged or partly exposed ridge of sand or coarse sediment that is built by waves offshore from a beach. Tombolo is a deposition landform in which an island is attached to the mainland by a narrow piece of land such as a spit or bar. A tombolo is a sediment deposit at the coast formed by wave refraction and diffraction at the edges of an obstacle (natural or artificial) originally detached from the mainland.   Incorrect Solution (c) Statement 1 Statement 2 Correct Correct Bars are deposits of sand and gravel laid down by waves and currents which separate the shoreline from the sea. They are submerged or partly exposed ridge of sand or coarse sediment that is built by waves offshore from a beach. Tombolo is a deposition landform in which an island is attached to the mainland by a narrow piece of land such as a spit or bar. A tombolo is a sediment deposit at the coast formed by wave refraction and diffraction at the edges of an obstacle (natural or artificial) originally detached from the mainland.   Question 21 of 35 21. Question Consider the following statements regarding the powers of the Central Consumer Protection Authority (CCPA): It has the authority to ban the endorser of a false or misleading advertisement from endorsing any product or service. It can issue safety notices to alert consumers about dangerous, hazardous, or unsafe goods, or services. It has the power to issue directions and guidelines aimed at preventing unfair trade practices and protecting consumers’ interests. How many of the above statements are correct? a) Only one b) Only two c) All three d) None Correct Solution (c) Central Consumer Protection Authority (CCPA) is a regulatory body that was established under the Consumer Protection Act 2019. It aims to protect the rights of consumers by cracking down on unfair trade practices, false and misleading advertisements, and e-commerce issues that are detrimental to the interests of the public and consumers. The powers of the Central Consumer Protection Authority (CCPA) are: It has the authority to ban the endorser of a false or misleading advertisement from endorsing any product or service. The duration of the prohibition can vary, with a longer period for subsequent contraventions. However, there’s a provision that exempts endorsers from liability if they have exercised due diligence to verify the claims made in the advertisement. Hence statement 1 is correct. It has the authority to inquire into or investigate matters related to violations of consumer rights or unfair trade practices, on its initiative, upon receiving a complaint, or upon direction from the central government. It can issue safety notices to alert consumers about dangerous, hazardous, or unsafe goods, or services. Hence statement 2 is correct. It can file complaints before the relevant Consumer Disputes Redressal Commission seeking orders or directions. This includes actions such as recalling unsafe goods and services, reimbursement of the price paid, and discontinuation of unfair trade practices and misleading advertisements. It has the power to issue directions and guidelines aimed at preventing unfair trade practices and protecting consumers’ interests. Hence statement 3 is correct. Incorrect Solution (c) Central Consumer Protection Authority (CCPA) is a regulatory body that was established under the Consumer Protection Act 2019. It aims to protect the rights of consumers by cracking down on unfair trade practices, false and misleading advertisements, and e-commerce issues that are detrimental to the interests of the public and consumers. The powers of the Central Consumer Protection Authority (CCPA) are: It has the authority to ban the endorser of a false or misleading advertisement from endorsing any product or service. The duration of the prohibition can vary, with a longer period for subsequent contraventions. However, there’s a provision that exempts endorsers from liability if they have exercised due diligence to verify the claims made in the advertisement. Hence statement 1 is correct. It has the authority to inquire into or investigate matters related to violations of consumer rights or unfair trade practices, on its initiative, upon receiving a complaint, or upon direction from the central government. It can issue safety notices to alert consumers about dangerous, hazardous, or unsafe goods, or services. Hence statement 2 is correct. It can file complaints before the relevant Consumer Disputes Redressal Commission seeking orders or directions. This includes actions such as recalling unsafe goods and services, reimbursement of the price paid, and discontinuation of unfair trade practices and misleading advertisements. It has the power to issue directions and guidelines aimed at preventing unfair trade practices and protecting consumers’ interests. Hence statement 3 is correct. Question 22 of 35 22. Question The term “two-state solution” is mentioned in news in the context of which of the following country? a) China b) Yemen c) Syria d) Israel Correct Solution (d) The two-state solution is used in the context of the Israel–Palestine issue. The two-state solution would create an independent Israel and Palestine and is the mainstream approach to resolving the conflict. The 1993 Oslo Accords marked the first time that the State of Israel and the Palestine Liberation Organization (PLO) formally recognized one another and publicly committed to negotiate a solution to their decades-long conflict. Its vision requires Israel to abandon its opposition to Palestinian claims (to national sovereignty). Hence option d is correct.   Incorrect Solution (d) The two-state solution is used in the context of the Israel–Palestine issue. The two-state solution would create an independent Israel and Palestine and is the mainstream approach to resolving the conflict. The 1993 Oslo Accords marked the first time that the State of Israel and the Palestine Liberation Organization (PLO) formally recognized one another and publicly committed to negotiate a solution to their decades-long conflict. Its vision requires Israel to abandon its opposition to Palestinian claims (to national sovereignty). Hence option d is correct.   Question 23 of 35 23. Question Consider the following statements about Golan Heights: It is a region located in south-western Syria. Israel captured it during the Six-Day War in 1967. Which of the statements given above is/are correct? a) 1 only b) 2 only c) Both 1 and 2 d) Neither 1 nor 2 Correct Solution (c) Golan Heights is a region located in southwestern Syria. Hence statement 1 is correct. Southern Syria is visible from the top of the Golan Heights. This gives Israel an advantage in monitoring Syrian movements. Rainwater from the Golan’s catchment feeds into the Jordan River. Israel captured it during the Six-Day War in 1967 and annexed it in 1981. Hence statement 2 is correct. This move was not recognised internationally, but the US did so unilaterally in 2019. Incorrect Solution (c) Golan Heights is a region located in southwestern Syria. Hence statement 1 is correct. Southern Syria is visible from the top of the Golan Heights. This gives Israel an advantage in monitoring Syrian movements. Rainwater from the Golan’s catchment feeds into the Jordan River. Israel captured it during the Six-Day War in 1967 and annexed it in 1981. Hence statement 2 is correct. This move was not recognised internationally, but the US did so unilaterally in 2019. Question 24 of 35 24. Question Consider the following statements: The Fifth Schedule of the Constitution ​​provides for the formation of autonomous administrative divisions called Autonomous District Councils (ADCs). Autonomous District Councils (ADCs) have up to 30 members with a term of five years. Autonomous District Councils (ADCs) are empowered to make legislative laws on matters like land, forests, fisheries, and social security with due approval from the governor. How many of the above statements are correct? a) Only one b) Only two c) All three d) None Correct Solution (b) The Sixth Schedule of the Constitution ​​provides for the formation of autonomous administrative divisions called Autonomous District Councils (ADCs). Hence statement 1 is incorrect. The sixth schedule applies to Assam, Meghalaya, Tripura, and Mizoram. Autonomous District Councils (ADCs) have up to 30 members with a term of five years. Hence statement 2 is correct. ADCs can constitute village courts within their jurisdiction to hear trials of cases involving the tribes. Governors specify the jurisdiction of high courts for each of these cases. Autonomous District Councils (ADCs) are empowered to make legislative laws on matters like land, forests, fisheries, and social security with due approval from the governor. Hence statement 3 is correct. The acts of Parliament or the state legislaturedo not apply to autonomous districts and autonomous regions or apply with specified modifications and exceptions.   Incorrect Solution (b) The Sixth Schedule of the Constitution ​​provides for the formation of autonomous administrative divisions called Autonomous District Councils (ADCs). Hence statement 1 is incorrect. The sixth schedule applies to Assam, Meghalaya, Tripura, and Mizoram. Autonomous District Councils (ADCs) have up to 30 members with a term of five years. Hence statement 2 is correct. ADCs can constitute village courts within their jurisdiction to hear trials of cases involving the tribes. Governors specify the jurisdiction of high courts for each of these cases. Autonomous District Councils (ADCs) are empowered to make legislative laws on matters like land, forests, fisheries, and social security with due approval from the governor. Hence statement 3 is correct. The acts of Parliament or the state legislaturedo not apply to autonomous districts and autonomous regions or apply with specified modifications and exceptions.   Question 25 of 35 25. Question With reference to the Bletchley Declaration, consider the following statements: The central objective of the Declaration is to address risks and responsibilities associated with frontier AI in a comprehensive and collaborative manner. The Declaration establishes a global AI ethics board to oversee all AI development. The Declaration was adopted by all the members of the United Nations General Assembly (UNGA). How many of the above statements are correct? a) Only one b) Only two c) All three d) None Correct Solution (a) The central objective of the Bletchley Declaration is to address risks and responsibilities associated with frontier AI in a comprehensive and collaborative manner. It emphasizes the necessity of aligning AI systems with human intent and urges a deeper exploration of AI’s full capabilities. Hence statement 1 is correct. The Bletchley Declaration does not establish a global AI ethics board to oversee all AI development. Instead, it sets forth a shared understanding of the opportunities and risks posed by AI and the need for governments to work together to meet the most significant challenges posed by the technology. Hence statement 2 is incorrect. The Bletchley Declaration is a recent initiative by the UK government to promote international cooperation in ensuring the safe use of artificial intelligence (AI) systems. It was signed by 27 countries, including India, China, the United States, and the United Kingdom, as well as the European Union. However, it is not an official document of the United Nations or any other international organization. It’s not something that was voted on by the UNGA or any other body. Hence statement 3 is incorrect. Incorrect Solution (a) The central objective of the Bletchley Declaration is to address risks and responsibilities associated with frontier AI in a comprehensive and collaborative manner. It emphasizes the necessity of aligning AI systems with human intent and urges a deeper exploration of AI’s full capabilities. Hence statement 1 is correct. The Bletchley Declaration does not establish a global AI ethics board to oversee all AI development. Instead, it sets forth a shared understanding of the opportunities and risks posed by AI and the need for governments to work together to meet the most significant challenges posed by the technology. Hence statement 2 is incorrect. The Bletchley Declaration is a recent initiative by the UK government to promote international cooperation in ensuring the safe use of artificial intelligence (AI) systems. It was signed by 27 countries, including India, China, the United States, and the United Kingdom, as well as the European Union. However, it is not an official document of the United Nations or any other international organization. It’s not something that was voted on by the UNGA or any other body. Hence statement 3 is incorrect. Question 26 of 35 26. Question Consider the following statements about the Loss and Damage (L&D) Fund: It is a fund created to provide financial assistance to nations most vulnerable and impacted by the effects of climate change. The World Bank will be the interim host of the fund for a period of four years. All countries can contribute to the fund voluntarily. All developed countries are eligible to apply for the fund. Choose the correct code: a) Only one b) Only two c) Only three d) All four Correct Solution (c) The Loss and Damage (L&D) Fund is a fund created to provide financial assistance to nations most vulnerable and impacted by the effects of climate change. Hence statement 1 is correct. The World Bank will be the interim host of the fund for a period of four years. Hence statement 2 is correct. All countries can contribute to the fund voluntarily. Hence statement 3 is correct. Countries have already committed at least $450 million to the fund. All developing countries are eligible to apply for the fund. Hence statement 4 is incorrect. A certain percentage of the fund has been set apart for least developed countries and small island developing states. Incorrect Solution (c) The Loss and Damage (L&D) Fund is a fund created to provide financial assistance to nations most vulnerable and impacted by the effects of climate change. Hence statement 1 is correct. The World Bank will be the interim host of the fund for a period of four years. Hence statement 2 is correct. All countries can contribute to the fund voluntarily. Hence statement 3 is correct. Countries have already committed at least $450 million to the fund. All developing countries are eligible to apply for the fund. Hence statement 4 is incorrect. A certain percentage of the fund has been set apart for least developed countries and small island developing states. Question 27 of 35 27. Question Consider the following statements about Vajrayana Buddhism: Its four main schools are Nyingma, Kagyu, Sakya, and Gelug. Its common deities include Tara, Avalokiteshvara, Manjushri, and It is regarded as the third turning of the wheel of Dharma. How many of the above statements are correct? a) Only one b) Only two c) All three d) None Correct Solution (c) Vajrayana Buddhism also known as Tantric Buddhism, is a form of Buddhism that developed in India and later spread to Tibet, Bhutan, Mongolia, and other regions. It claims that tantras, mantras, and yantras are superior vehicles to liberation since they contain numerous sophisticated tantric ritual practices. Its four main schools are Nyingma, Kagyu, Sakya, and Gelug. Hence statement 1 is correct. It views enlightenment as something that can be attained in a single lifetime. Its common deities include Tara, Avalokiteshvara, Manjushri, and Vajrapani. Hence statement 2 is correct. It draws upon elements of Mahayana Buddhism but also includes unique practices and teachings, such as tantra, deity yoga, meditative techniques, and esoteric rituals. It is regarded as the third turning of the wheel of Dharma. Hence statement 3 is correct. Tantric Buddhism uses art for teaching, healing, and meditation. In tantrism, art is seen as a powerful instrument for aiding spiritual development through rituals. Paintings of deities are used as a tool to evoke the subject or in some cases make contact directly. Incorrect Solution (c) Vajrayana Buddhism also known as Tantric Buddhism, is a form of Buddhism that developed in India and later spread to Tibet, Bhutan, Mongolia, and other regions. It claims that tantras, mantras, and yantras are superior vehicles to liberation since they contain numerous sophisticated tantric ritual practices. Its four main schools are Nyingma, Kagyu, Sakya, and Gelug. Hence statement 1 is correct. It views enlightenment as something that can be attained in a single lifetime. Its common deities include Tara, Avalokiteshvara, Manjushri, and Vajrapani. Hence statement 2 is correct. It draws upon elements of Mahayana Buddhism but also includes unique practices and teachings, such as tantra, deity yoga, meditative techniques, and esoteric rituals. It is regarded as the third turning of the wheel of Dharma. Hence statement 3 is correct. Tantric Buddhism uses art for teaching, healing, and meditation. In tantrism, art is seen as a powerful instrument for aiding spiritual development through rituals. Paintings of deities are used as a tool to evoke the subject or in some cases make contact directly. Question 28 of 35 28. Question Consider the following statements about Methotrexate (MTX): It is used to treat various medical conditions, primarily cancers, autoimmune diseases, and certain inflammatory conditions. A highly fluorescent material developed using phosphorene, cystine, and gold(Ph-Cys-Au) is used to detect MTX. Choose the correct code: a) 1 only b) 2 only c) Both 1 and 2 d) Neither 1 nor 2 Correct Solution (c) Methotrexate (MTX) is used to treat various medical conditions, primarily cancers, autoimmune diseases, and certain inflammatory conditions. Hence statement 1 is correct. It belongs to a class of drugs known as antimetabolite. The MTX overdose in blood plasma is hazardous if it remains in the system for more than 10 hours, resulting in poisoning effects on the lungs, ulcers of the stomach, and heart stroke. A highly fluorescent material developed using phosphorene, cystine, and gold (Ph-Cys-Au) is used to detect MTX. Hence statement 2 is correct. It has exceptional optical properties and thus can be used as a visual sensing platform for detecting the anti-cancer drug MTX overdosage. Incorrect Solution (c) Methotrexate (MTX) is used to treat various medical conditions, primarily cancers, autoimmune diseases, and certain inflammatory conditions. Hence statement 1 is correct. It belongs to a class of drugs known as antimetabolite. The MTX overdose in blood plasma is hazardous if it remains in the system for more than 10 hours, resulting in poisoning effects on the lungs, ulcers of the stomach, and heart stroke. A highly fluorescent material developed using phosphorene, cystine, and gold (Ph-Cys-Au) is used to detect MTX. Hence statement 2 is correct. It has exceptional optical properties and thus can be used as a visual sensing platform for detecting the anti-cancer drug MTX overdosage. Question 29 of 35 29. Question Consider the following statements regarding the key findings of UNEP’s Emissions Gap Report 2023: The world is heading towards a 4°C temperature rise by 2100, compared to pre-industrial levels. This is the fourteenth Emissions Gap Report. To limit global warming to 2°C, emissions must be cut by 28% by 2030. How many of the above statements are correct? a) Only one b) Only two c) All three d) None Correct Solution (b) The key findings of UNEP’s Emissions Gap Report 2023: The world is heading towards a 3°C temperature rise by 2100, compared to pre-industrial levels. Hence statement 1 is incorrect. Global emissions rose by 1.2% in 2022 from 2021, nearly returning to pre-pandemic levels. This fourteenth Emissions Gap Report is published ahead of the twenty-eighth session of the Conference of the Parties to the United Nations Framework on Climate Change (COP 28). Hence statement 2 is correct. The 2016 edition of the report had projected a warming of up to 3.4 degrees Celsius in a business-as-usual scenario. However, according to the Emissions Gap Report 2023, the current projection for the increase in emissions by 2030 is about 3%. Still, this is not enough to meet the 1.5°C target. To limit global warming to 2°C, emissions must be cut by 28% by 2030 and by 42% by 2030 for the 1.5°C target. Hence statement 3 is correct. Incorrect Solution (b) The key findings of UNEP’s Emissions Gap Report 2023: The world is heading towards a 3°C temperature rise by 2100, compared to pre-industrial levels. Hence statement 1 is incorrect. Global emissions rose by 1.2% in 2022 from 2021, nearly returning to pre-pandemic levels. This fourteenth Emissions Gap Report is published ahead of the twenty-eighth session of the Conference of the Parties to the United Nations Framework on Climate Change (COP 28). Hence statement 2 is correct. The 2016 edition of the report had projected a warming of up to 3.4 degrees Celsius in a business-as-usual scenario. However, according to the Emissions Gap Report 2023, the current projection for the increase in emissions by 2030 is about 3%. Still, this is not enough to meet the 1.5°C target. To limit global warming to 2°C, emissions must be cut by 28% by 2030 and by 42% by 2030 for the 1.5°C target. Hence statement 3 is correct. Question 30 of 35 30. Question Consider the following statements about the Amplifi 2.0 Portal: It was launched by the Ministry of Housing and Urban Affairs. It aims to make raw data from Indian cities available on a single platform for academics, researchers, and stakeholders to help data-driven policymaking. Choose the correct code: a) 1 only b) 2 only c) Both 1 and 2 d) Neither 1 nor 2 Correct Solution (c) The Amplifi 2.0 Portal was launched by the Ministry of Housing and Urban Affairs. Hence statement 1 is correct. Amplifi – Assessment and Monitoring Platform for Liveable, Inclusive, and Future-Ready Urban India portal. The portal provides data on various services for several cities such as: Total diesel consumption Number of water quality samples tested Average annual healthcare expenditure Total population residing in slums Recorded fatalities from road accidents It aims to make raw data from Indian cities available on a single platform for academics, researchers, and stakeholders to help data-driven policymaking. Hence statement 2 is correct. Currently, 258 urban local bodies have been onboarded and data for 150 cities is accessible on the platform. Eventually, data from the more than 4,000 Urban Local Bodies will be made available on the portal. Note: According to the 2011 Census, 37.7 crore people (31% of the population) lived in urban areas. By 2030, this is projected to increase by 60 crores or 40%. In 2011, urban areas contributed 63% to the country’s GDP and this is projected to increase to 75% by 2030. Incorrect Solution (c) The Amplifi 2.0 Portal was launched by the Ministry of Housing and Urban Affairs. Hence statement 1 is correct. Amplifi – Assessment and Monitoring Platform for Liveable, Inclusive, and Future-Ready Urban India portal. The portal provides data on various services for several cities such as: Total diesel consumption Number of water quality samples tested Average annual healthcare expenditure Total population residing in slums Recorded fatalities from road accidents It aims to make raw data from Indian cities available on a single platform for academics, researchers, and stakeholders to help data-driven policymaking. Hence statement 2 is correct. Currently, 258 urban local bodies have been onboarded and data for 150 cities is accessible on the platform. Eventually, data from the more than 4,000 Urban Local Bodies will be made available on the portal. Note: According to the 2011 Census, 37.7 crore people (31% of the population) lived in urban areas. By 2030, this is projected to increase by 60 crores or 40%. In 2011, urban areas contributed 63% to the country’s GDP and this is projected to increase to 75% by 2030. Question 31 of 35 31. Question A series is given, with one term missing. Choose the correct alternative from the given ones that will complete the series. 113, 127, 131, 137, 139,141, ?, 151, 157, 163, 167, 173. a) 143 b) 156 c) 149 d) 147 Correct Solution (c) 149 is the next possible prime number. The above series is series of prime numbers. Incorrect Solution (c) 149 is the next possible prime number. The above series is series of prime numbers. Question 32 of 35 32. Question A, B, C rent a pasture. A puts 10 oxen for 7 months, B puts 12 oxen for 5 months and C puts 15 oxen for 3 months for grazing. If the rent of the pasture is Rs. 175, how much must C pay as his share of rent? a) Rs. 45 b) Rs. 50 c) Rs. 55 d) Rs. 60 Correct Solution (a) Ratio of shares of A, B, C A : B : C = (10 x 7) : (12 x 5) : (15 x 3) = 70 : 60 : 45 = 14 : 12 : 9. C’s rent = Rs. 175 x (9/35) = Rs. 45 Incorrect Solution (a) Ratio of shares of A, B, C A : B : C = (10 x 7) : (12 x 5) : (15 x 3) = 70 : 60 : 45 = 14 : 12 : 9. C’s rent = Rs. 175 x (9/35) = Rs. 45 Question 33 of 35 33. Question O In a certain coded language, SUBLIME is coded as BEILMSU, NASTY is coded as ANSTY, then DIVINE is coded as a) DIEVIN b) DEIINV c) VNIIED d) NIVIED Correct Solution (b) All the alphabets in the word are written in alphabetical order within that word. Incorrect Solution (b) All the alphabets in the word are written in alphabetical order within that word. Question 34 of 35 34. Question A man divides Rs. 8600 among 5 sons, 4 daughters and 2 nephews . If each daughter receives four times as much as each nephews and each son receives five times as much as each nephews, how much does each daughter receive? a) 600 b) 800 c) 900 d) 1200 Correct Solution (b) Let the share of each nephew be Rs. y. Then, share of each daughter = Rs. 4y ; share of each son = Rs. 5y; ∴ 8600=5x+4y+2z  …. (1) y=4z,x=5z Putting these value in equation (1) 8600=25z+16z+2z 8600=43z z=200 y=4z=800 ∴ Each daughter will receive =800 rs Incorrect Solution (b) Let the share of each nephew be Rs. y. Then, share of each daughter = Rs. 4y ; share of each son = Rs. 5y; ∴ 8600=5x+4y+2z  …. (1) y=4z,x=5z Putting these value in equation (1) 8600=25z+16z+2z 8600=43z z=200 y=4z=800 ∴ Each daughter will receive =800 rs Question 35 of 35 35. Question The product of two numbers is 9375 and the quotient, when the larger one is divided  by the smaller is  15. The sum of the numbers is a) 495 b) 425 c) 250 d) 400 Correct Solution (d) Let the numbers be x and y. Then, xy = 9375 and x/y = 15. ⇒ {xy / (x/y)} = 9375/15 = y2 = 625 = y = 25 = x = 15y = 15 x 25 = 375. Sum of the numbers = 375 + 25 = 400. Incorrect Solution (d) Let the numbers be x and y. Then, xy = 9375 and x/y = 15. ⇒ {xy / (x/y)} = 9375/15 = y2 = 625 = y = 25 = x = 15y = 15 x 25 = 375. Sum of the numbers = 375 + 25 = 400. window.wpProQuizInitList = window.wpProQuizInitList || []; window.wpProQuizInitList.push({ id: '#wpProQuiz_3615', init: { quizId: 3615, mode: 1, globalPoints: 70, timelimit: 1800, resultsGrade: [0], bo: 704, qpp: 0, catPoints: [70], formPos: 0, lbn: "Test-summary", json: {"31760":{"type":"single","id":31760,"catId":0,"points":2,"correct":[1,0,0,0]},"31762":{"type":"single","id":31762,"catId":0,"points":2,"correct":[0,0,1,0]},"31765":{"type":"single","id":31765,"catId":0,"points":2,"correct":[0,1,0,0]},"31768":{"type":"single","id":31768,"catId":0,"points":2,"correct":[0,1,0,0]},"31769":{"type":"single","id":31769,"catId":0,"points":2,"correct":[0,1,0,0]},"31771":{"type":"single","id":31771,"catId":0,"points":2,"correct":[1,0,0,0]},"31772":{"type":"single","id":31772,"catId":0,"points":2,"correct":[0,0,0,1]},"31773":{"type":"single","id":31773,"catId":0,"points":2,"correct":[0,0,1,0]},"31776":{"type":"single","id":31776,"catId":0,"points":2,"correct":[0,1,0,0]},"31777":{"type":"single","id":31777,"catId":0,"points":2,"correct":[0,1,0,0]},"31778":{"type":"single","id":31778,"catId":0,"points":2,"correct":[0,1,0,0]},"31781":{"type":"single","id":31781,"catId":0,"points":2,"correct":[0,0,1,0]},"31783":{"type":"single","id":31783,"catId":0,"points":2,"correct":[0,1,0,0]},"31784":{"type":"single","id":31784,"catId":0,"points":2,"correct":[0,1,0,0]},"31786":{"type":"single","id":31786,"catId":0,"points":2,"correct":[0,0,0,1]},"31788":{"type":"single","id":31788,"catId":0,"points":2,"correct":[1,0,0,0]},"31789":{"type":"single","id":31789,"catId":0,"points":2,"correct":[0,0,1,0]},"31790":{"type":"single","id":31790,"catId":0,"points":2,"correct":[1,0,0,0]},"31793":{"type":"single","id":31793,"catId":0,"points":2,"correct":[0,1,0,0]},"31795":{"type":"single","id":31795,"catId":0,"points":2,"correct":[0,0,1,0]},"31798":{"type":"single","id":31798,"catId":0,"points":2,"correct":[0,0,1,0]},"31800":{"type":"single","id":31800,"catId":0,"points":2,"correct":[0,0,0,1]},"31802":{"type":"single","id":31802,"catId":0,"points":2,"correct":[0,0,1,0]},"31804":{"type":"single","id":31804,"catId":0,"points":2,"correct":[0,1,0,0]},"31806":{"type":"single","id":31806,"catId":0,"points":2,"correct":[1,0,0,0]},"31809":{"type":"single","id":31809,"catId":0,"points":2,"correct":[0,0,1,0]},"31811":{"type":"single","id":31811,"catId":0,"points":2,"correct":[0,0,1,0]},"31814":{"type":"single","id":31814,"catId":0,"points":2,"correct":[0,0,1,0]},"31817":{"type":"single","id":31817,"catId":0,"points":2,"correct":[0,1,0,0]},"31820":{"type":"single","id":31820,"catId":0,"points":2,"correct":[0,0,1,0]},"31821":{"type":"single","id":31821,"catId":0,"points":2,"correct":[0,0,1,0]},"31823":{"type":"single","id":31823,"catId":0,"points":2,"correct":[1,0,0,0]},"31824":{"type":"single","id":31824,"catId":0,"points":2,"correct":[0,1,0,0]},"31827":{"type":"single","id":31827,"catId":0,"points":2,"correct":[0,1,0,0]},"31829":{"type":"single","id":31829,"catId":0,"points":2,"correct":[0,0,0,1]}} } }); All the Best IASbaba

DAILY CURRENT AFFAIRS IAS | UPSC Prelims and Mains Exam – 5th April 2024

Archives (PRELIMS & MAINS Focus)   Dark Energy Syllabus Prelims & Mains – Science Context: An international team of researchers has just released the most comprehensive “three-dimensional” map of the universe, which, scientists hope, could reveal some clues about dark energy. Background:- Dark energy, the mysterious form of energy that makes up about 68% of the universe, has intrigued physicists and astronomers for decades. Dark energy has been noted as “the most profound mystery in all of science” About Dark Energy Everything we see – the planets, moons, massive galaxies, you, me, this website – makes up less than 5% of the universe. About 27% is dark matter and 68% is dark energy. While dark matter attracts and holds galaxies together, dark energy repels and causes the expansion of our universe. Takeaways from the research: Using Dark Energy Spectroscopic Instrument or DESI, which is mounted over the Mayall 4-Meter Telescope in Arizona, United States, researchers have been able to measure light from six million galaxies — some of which existed as far back as 11 billion years ago — to prepare the most detailed map of the universe as yet with very precise information about the distances between these galaxies. The hypothesis of dark energy comes mainly from the observed phenomenon of the universe expanding at a rapid rate. The vast empty spaces between stars and galaxies have been measured to be expanding at an accelerating pace, despite the countervailing force of gravitation that has the effect of pulling things together. Scientists have been unable to find any explanation for this rapid expansion, and have been forced to hypothesise that there must be some “dark” energy causing this expansion. Understanding the nature of dark energy is one of the fundamental problems in science right now, because it can offer key insights into the origin and evolution of the universe, as well as its eventual fate. Source: The Indian Express Electric Vehicles Syllabus Prelims & Mains -Science & Environment Context: In March, the Union government had approved a new e-vehicle policy with the aim to facilitate the entry of global EV manufacturers in the country. Background: Attracting global EV players will provide Indian consumers with access to latest technology, boost the Make in India initiative, strengthen the EV ecosystem by promoting healthy competition among EV players leading to high volume of production, economies of scale, lower cost of production, reduce imports of crude Oil, lower trade deficit, reduce air pollution, particularly in cities, and will have a positive impact on health and environment. About the policy and Indias EV landscape: The policy involves lowering the duty for EV imports — this has been a long-standing demand for EV majors like Tesla — for companies setting up a manufacturing plant with a minimum investment of Rs 4,150 crore. Alongside, the policy also lays out clear localisation targets that companies have to achieve — 25 per cent by the third year and 50 per cent by the fifth — in order to boost domestic value addition. Access to one of the largest and fastest growing markets in the world — India is the third largest auto market behind China and the US — at a time when others are slowing down, will be a big draw for electric vehicle manufacturers like Tesla. While the EV market is currently small in the country, it is gaining traction — in 2023, sales of electric vehicles surpassed 1.5 million, dominated by two-wheelers and three-wheelers. The scope for growth in various segments is immense, especially considering the government wants to increase the share of electric vehicles to 30 per cent by 2030. Source: Indian Express AI WASHING Syllabus Prelims – Current Event Context: The US Securities and Exchange Commission (SEC) has taken action against companies engaging in AI Washing. Background: SEC Chairman Gary Gensler emphasized the importance of truthfulness in AI-related marketing. He warned that companies should ensure their representations about AI usage are accurate and not deceptive. About AI WASHING AI washing is a marketing tactic where companies exaggerate the involvement of Artificial Intelligence (AI)in their products and services. This term is derived from “greenwashing”, which refers to companies making misleading claims about their environmental impact. AI washing refers to the practice of making exaggerated or false claims about the use of artificial intelligence (AI) in marketing, advertising, or business operations. Companies engage in AI washing to make their offerings seem more advanced than they are, capitalizing on the growing interest in AI technology. This can mislead consumers and investors who may assume that the company is using advanced AI technologies. For instance, a company might claim that its product is “powered by AI” when in reality, the AI component is minimal or non-existent. This can be misleading as consumers might assume that the product is more sophisticated or capable than it actually is. AI washing has become a global phenomenon, with regulatory bodies like the US Securities and Exchange Commission (SEC) taking action against companies for misleading AI claims. Source: Down To Earth SHALLOW FAKES Syllabus Prelims : Current Event Context: Shallow fakes, also known as cheap fakes, are a cause for concern, especially during elections. Background: With rapid dissemination on social media, shallow fakes can go viral quickly. False or misleading content can become ingrained in public discourse. As we approach the Lok Sabha elections, staying informed and discerning between genuine and manipulated content is essential. About SHALLOW FAKES Shallow fakes are manipulated images, videos, and voice clips created without AI technology. Unlike deepfakes, which rely on sophisticated AI algorithms, shallow fakes use traditional editing tools to deceive viewers. They are increasingly used in political contexts, particularly during elections, to spread misinformation. Characteristics: Quality: The term “shallow” implies that these fakes are lower in quality compared to deepfakes. Methods: Shallow fakes involve conventional photo editing or altering video speed to change speech patterns. Misleading Context: They often mis-caption or mis-contextualize existing images or videos, falsely claiming they are from a different time or place. Difference Between Shallow Fakes and Deepfakes: Deepfakes: Utilize AI and machine learning to create photorealistic and audio-realistic content. Shallow fakes: Rely less on complex editing techniques and more on connecting partial truths to small lies using traditional tools. Source: Indian Express ANTI DUMPING Syllabus Prelims – Science Context: India has initiated an anti-dumping probe into the import of a chemical used in the rubber industry from China and Japan. Background: This investigation was prompted by a complaint filed by a domestic player. The specific chemical under scrutiny is ‘Insoluble Sulphur’. The Directorate General of Trade Remedies (DGTR) is currently examining the alleged dumping of this chemical. About ANTI DUMPING Dumping occurs when a company exports a product at a price lower than what it typically charges in its own home market. In response to this unfair trade practice, countries may impose an anti-dumping duty (ADD) on foreign imports that are believed to be priced below fair market value. Anti-Dumping Duty (ADD): ADD is a customs duty applied to imports to protect domestic industries from dumped goods. It rectifies the situation arising from the dumping of goods and its trade-distortive effects. In the long term, ADD can reduce international competition for domestic companies producing similar goods. The World Trade Organization (WTO) permits the use of anti-dumping measures as an instrument of fair competition. ADD is different from countervailing duties, which target goods that have received government subsidies in the originating or exporting country. Directorate General of Trade Remedies (DGTR): The Directorate General of Trade Remedies (DGTR) operates under the Ministry of Commerce and Industry in India. It serves as the apex national authority responsible for administering various trade remedial measures, including anti-dumping investigations, countervailing duties, and safeguard measures. The DGTR plays a crucial role in providing trade defence support to domestic industries and exporters facing trade remedy investigations from other countries. Its objective is to ensure fair trade practices and protect the interests of Indian businesses in the global marketplace. Source: Economic Times Previous Year Question Q1. Which one of the following is associated with the issue of control and phasing out of the use of ozone depleting substances? Bretton Woods Conference Montreal Protocol Kyoto Protocol Nagoya Protocol Forced Labour Syllabus Mains – GS 2 Context: The International Labour Organization (ILO) released a report titled ‘Profits and poverty: The economics of forced labour’, which has found that forced labour generates illegal profits worth USD 36 billion per year. Background: “Forced labour perpetuates cycles of poverty and exploitation and strikes at the heart of human dignity. We now know that the situation has only got worse. The international community must urgently come together to take action to end this injustice,” said the ILO Director General. Forced Labour: According to ILO, forced or compulsory labour is “all work or service that is extracted from any person under the menace of any penalty and for which said person has not offered himself voluntarily”. Forced labour is defined, for purposes of measurement, as work that is both Involuntary and under penalty or menace of a penalty (coercion). Involuntary work refers to any work undertaken without the free and informed consent of the worker. Coercion refers to the means used to compel someone to work without their free and informed consent. Key Highlights of the Report: Forced labour generates illegal profits worth USD 36 billion per year, which is a 37% increase since 2014. This increase is attributed to both a growth in the number of people forced into labour and higher profits generated from the exploitation of victims. Total annual illegal profits from forced labour are highest in Europe and Central Asia (USD 84 billion), followed by Asia and the Pacific (USD 62 billion), the Americas (USD 52 billion), Africa (USD 20 billion), and the Arab States (USD 18 billion). Traffickers and criminals are estimated to generate close to USD 10,000 per victim, up from USD 8,269 a decade ago. Forced commercial sexual exploitation accounts for more than two-thirds (73%) of the total illegal profits, despite accounting for only 27% of the total number of victims in privately imposed labour. After forced commercial sexual exploitation, the sector with the highest annual illegal profits from forced labour is industry (USD 35 billion), followed by services (USD 20.8 billion), agriculture (USD 5.0 billion), and domestic work (USD 2.6 billion). The industry sector includes mining and quarrying, manufacturing, construction, and utilities. The services sector encompasses activities related to wholesale and trade, accommodation and food service activities, art and entertainment, personal services, administrative and support services, education, health and social services, and transport and storage. The agriculture sector includes forestry, hunting as well as the cultivation of crops, livestock production, and fishing. There were 27.6 million people engaged in forced labour on any given day in 2021, representing an increase of 2.7 million since 2016. Recommendations made by the Report: The report emphasises the urgent need for investment in enforcement measures to stem illegal profit flows and hold perpetrators accountable. It highlights the importance of strengthening legal frameworks, providing training for enforcement officials, extending labour inspection into high-risk sectors, and better coordination between labour and criminal law enforcement. While law enforcement measures are crucial, the report underscores that forced labour cannot be ended through enforcement actions alone. It must be part of a comprehensive approach that prioritises addressing root causes and safeguarding victims. Promoting fair recruitment processes is deemed crucial as forced labour cases can often be traced back to recruitment abuses. Ensuring the freedom of workers to associate and bargain collectively is also essential in combating forced labour. Source: Hindu Practice MCQs Daily Practice MCQs Q1.)With reference to the Anti-Dumping Duty, consider the following statements: Anti-Dumping Duty is a customs duty applied to imports to protect domestic industries from dumped goods. The World Trade Organization permits the use of anti-dumping measures as an instrument of fair competition. The Directorate General of Trade Remedies serves as the apex national authority responsible for administering various trade remedial measures including anti-dumping investigations. How many of the above given statements are not correct? Only one Only two All three None Q2.) Consider the following statements: Shallow fakes are manipulated images, videos, and voice clips created without AI technology. Deepfakes uses sophisticated AI and machine learning algorithms to create highly realistic and often indistinguishable counterfeit content. Which of the statements given above is/ are correct? 1 only 2 only Both 1 and 2 Neither 1 nor 2 Q3.)Which one of the following best describes the term “AI washing”? Conveying a false impression that a company’s products are eco-friendly and environmentally sound. Practice of making exaggerated claims about the use of Artificial Intelligence (AI) in marketing, advertising, or business operations. Ignoring the moral consequences while involving Artificial Intelligence (AI) in business operations. Making mandatory provisions for involving Artificial Intelligence (AI) in a government project/programme. Comment the answers to the above questions in the comment section below!! ANSWERS FOR ’  5th April  2024 – Daily Practice MCQs’ will be updated along with tomorrow’s Daily Current Affairs.st ANSWERS FOR  4th April – Daily Practice MCQs Answers- Daily Practice MCQs Q.1) – c Q.2) – c Q.3) – b

Daily Prelims CA Quiz

UPSC Quiz – 2024 : IASbaba’s Daily Current Affairs Quiz 6th April 2024

For Previous Daily Quiz (ARCHIVES) – CLICK HERE The Current Affairs questions are based on sources like ‘The Hindu’, ‘Indian Express’ and ‘PIB’, which are very important sources for UPSC Prelims Exam. The questions are focused on both the concepts and facts. The topics covered here are generally different from what is being covered under ‘Daily Current Affairs/Daily News Analysis (DNA) and Daily Static Quiz’ to avoid duplication. The questions would be published from Monday to Saturday before 2 PM. One should not spend more than 10 minutes on this initiative. Gear up and Make the Best Use of this initiative. Do remember that, “the difference between Ordinary and EXTRA-Ordinary is PRACTICE!!” Important Note: Don’t forget to post your marks in the comment section. Also, let us know if you enjoyed today’s test 🙂After completing the 5 questions, click on ‘View Questions’ to check your score, time taken, and solutions.To take the Test Click Here

[DAY 30] 60 DAY RAPID REVISION (RaRe) SERIES for UPSC Prelims 2024 – ENVIRONMENT, CURRENT AFFAIRS & CSAT TEST SERIES!

Archives Hello Friends The 60 Days Rapid Revision (RaRe) Series is IASbaba’s Flagship Initiative recommended by Toppers and loved by the aspirants’ community every year. It is the most comprehensive program which will help you complete the syllabus, revise and practice tests on a daily basis. The Programme on a daily basis includes Daily Prelims MCQs from Static (Monday – Saturday) Daily Static Quiz will cover all the topics of static subjects – Polity, History, Geography, Economics, Environment and Science and technology. 20 questions will be posted daily and these questions are framed from the topics mentioned in the schedule. It will ensure timely and streamlined revision of your static subjects. Daily Current Affairs MCQs (Monday – Saturday) Daily 5 Current Affairs questions, based on sources like ‘The Hindu’, ‘Indian Express’ and ‘PIB’, would be published from Monday to Saturday according to the schedule. Daily CSAT Quiz (Monday – Friday) CSAT has been an Achilles heel for many aspirants. Daily 5 CSAT Questions will be published. Note – Daily Test of 20 static questions, 10 current affairs, and 5 CSAT questions. (35 Prelims Questions) in QUIZ FORMAT will be updated on a daily basis. To Know More about 60 Days Rapid Revision (RaRe) Series – CLICK HERE   60 Day Rapid Revision (RaRe) Series Schedule – CLICK HERE  Important Note Comment your Scores in the Comment Section. This will keep you accountable, responsible and sincere in days to come. It will help us come out with the Cut-Off on a Daily Basis. Let us know if you enjoyed today’s test 🙂  You can post your comments in the given format  (1) Your Score (2) Matrix Meter (3) New Learning from the Test Time limit: 0 Test-summary 0 of 35 questions completed Questions: 1 2 3 4 5 6 7 8 9 10 11 12 13 14 15 16 17 18 19 20 21 22 23 24 25 26 27 28 29 30 31 32 33 34 35 Information The following Test is based on the syllabus of 60 Days Plan-2023 for UPSC IAS Prelims 2022. To view Solutions, follow these instructions: Click on – ‘Start Test’ button Solve Questions Click on ‘Test Summary’ button Click on ‘Finish Test’ button Now click on ‘View Questions’ button – here you will see solutions and links. You have already completed the test before. Hence you can not start it again. Test is loading... You must sign in or sign up to start the test. You have to finish following test, to start this test: Results 0 of 35 questions answered correctly Your time: Time has elapsed You have scored 0 points out of 0 points, (0) Average score     Your score     Categories Not categorized 0% Your result has been entered into leaderboard Loading Name: E-Mail: Captcha: maximum of 70 points Pos. Name Entered on Points Result Table is loading No data available 1 2 3 4 5 6 7 8 9 10 11 12 13 14 15 16 17 18 19 20 21 22 23 24 25 26 27 28 29 30 31 32 33 34 35 Answered Review Question 1 of 35 1. Question Which of the following are examples of adaptations found in the animals? Short straight and sharp claws in Polar bears Large ears of Elephants Moving in isolation behaviour of Penguins Long beak of Toucan bird in rainforests Select the correct answer using the code given below: a) Only one b) Only two c) Only three d) All four Correct Solution (b) Statement 1 Statement 2 Statement 3 Statement 4 Incorrect Correct Incorrect Correct Polar bears have long curved and sharp claws which help them in walking on ice and to catch and hold prey. Presence of white fur reduces their visibility in the snowy background which helps them in catching prey. It also protects them from the extreme cold. Elephants have large ears which help the elephant to keep cool in the hot and humid climate of the rainforest. They also help them to hear even very soft sounds.   To prevent themselves freezing to death, penguins huddle together in tightly-packed groups to conserve heat and shelter themselves from the intense winds of the Antarctica. Penguins on the outskirts regularly muscle their way inside the huddle.   As there is competition for food in rainforests, some animals are adapted to get food which is not easily reachable. A striking example is that of the bird Toucan, which possesses a long, large beak. This helps a toucan to reach the fruits on branches which are otherwise too weak to support its weight.   Notes: Adaptations are found in the natural world among animals, which help them in surviving in their habitat. Other adaptations found in the animals- are- Polar bears – They also have a layer of fat under their skin. They are so well-insulated that they have to move slowly and rest often to avoid getting overheated. Physical activities on warm days necessitate cooling. So, the polar bear goes for swimming. They have a strong sense of smell so that they can catch their prey for food. Penguins – They also have a thick skin and a lot of fat to protect them from cold. Like polar bears, penguins are also good swimmers. Their bodies are streamlined and their feet have webs, making them good swimmers. Incorrect Solution (b) Statement 1 Statement 2 Statement 3 Statement 4 Incorrect Correct Incorrect Correct Polar bears have long curved and sharp claws which help them in walking on ice and to catch and hold prey. Presence of white fur reduces their visibility in the snowy background which helps them in catching prey. It also protects them from the extreme cold. Elephants have large ears which help the elephant to keep cool in the hot and humid climate of the rainforest. They also help them to hear even very soft sounds.   To prevent themselves freezing to death, penguins huddle together in tightly-packed groups to conserve heat and shelter themselves from the intense winds of the Antarctica. Penguins on the outskirts regularly muscle their way inside the huddle.   As there is competition for food in rainforests, some animals are adapted to get food which is not easily reachable. A striking example is that of the bird Toucan, which possesses a long, large beak. This helps a toucan to reach the fruits on branches which are otherwise too weak to support its weight.   Notes: Adaptations are found in the natural world among animals, which help them in surviving in their habitat. Other adaptations found in the animals- are- Polar bears – They also have a layer of fat under their skin. They are so well-insulated that they have to move slowly and rest often to avoid getting overheated. Physical activities on warm days necessitate cooling. So, the polar bear goes for swimming. They have a strong sense of smell so that they can catch their prey for food. Penguins – They also have a thick skin and a lot of fat to protect them from cold. Like polar bears, penguins are also good swimmers. Their bodies are streamlined and their feet have webs, making them good swimmers. Question 2 of 35 2. Question Consider the following pairs: Name of Interaction Activity 1.     Mutualism Both the species are benefitted 2.     Amensalism Both the species are unaffected 3.     Competition Both the species are disadvantaged How many of the above pairs is/are correctly matched? a) None of the pairs b) One pair only c) Two pairs only d) Three pairs only Correct Solution (c) 1.       2.       3.       Mutualism Amensalism Competition Correct Incorrect Correct Both the species are benefitted.   One species is harmed whereas the other is unaffected.   Both the species are disadvantaged.     Notes: Species A Species B Name of Interaction                +                + Mutualism                –                – Competition                +                – Predation                +                – Parasitism                +                0 Commensalism                –                0 Ammensalism  Examples of different Interactions: Mutualism: Humans and Plants: The human requires oxygen for life and plants use the carbon dioxide for photosynthesis. Here both human and plants are mutually benefited. Humans use the oxygen given by the plants. In return, plants use carbon dioxide, which is exhaled by the Humans. Oxpeckers and Rhinos: The bird oxpecker lives on the rhino and removes all bugs and parasites on the animal skin by eating them. The Rhino provides the bird with food and in return, the bugs removed from the skin of the rhino. Both rhino and the oxpecker are benefited. Competition: An example among animals could be the case of Cheetah and lions since both species feed on similar prey, they are negatively impacted by the presence of the other because they will have less food, however they still persist together, despite the prediction that under competition one will displace the other. Predation: Tiger (predator) eating deer (prey), frog eating insects Parasitism: Among the animals, ticks, mites and lice are external parasites or ectoparasites. The fungal parasites include Erysiphe (powdery mildew), Ustilago (smut), Puccinia (rust), etc.; which cause diseases that result in serious losses of economically important crops. Commensalism: Association between suckerfish (or Remora or Echeneis) and shark- The sucker fish has the dorsal fin modified as a sucker(or adhesive) disc, with the help of which it is attached to the body of shark so that the suckerfish gets free transportation. The sucker fish releases the attachment after sometime and swims in search of food. Ammensalism: When cattle trample on grass, the grass is crushed. However, the cattle do not benefit from this action nor is harmed in the process. Incorrect Solution (c) 1.       2.       3.       Mutualism Amensalism Competition Correct Incorrect Correct Both the species are benefitted.   One species is harmed whereas the other is unaffected.   Both the species are disadvantaged.     Notes: Species A Species B Name of Interaction                +                + Mutualism                –                – Competition                +                – Predation                +                – Parasitism                +                0 Commensalism                –                0 Ammensalism  Examples of different Interactions: Mutualism: Humans and Plants: The human requires oxygen for life and plants use the carbon dioxide for photosynthesis. Here both human and plants are mutually benefited. Humans use the oxygen given by the plants. In return, plants use carbon dioxide, which is exhaled by the Humans. Oxpeckers and Rhinos: The bird oxpecker lives on the rhino and removes all bugs and parasites on the animal skin by eating them. The Rhino provides the bird with food and in return, the bugs removed from the skin of the rhino. Both rhino and the oxpecker are benefited. Competition: An example among animals could be the case of Cheetah and lions since both species feed on similar prey, they are negatively impacted by the presence of the other because they will have less food, however they still persist together, despite the prediction that under competition one will displace the other. Predation: Tiger (predator) eating deer (prey), frog eating insects Parasitism: Among the animals, ticks, mites and lice are external parasites or ectoparasites. The fungal parasites include Erysiphe (powdery mildew), Ustilago (smut), Puccinia (rust), etc.; which cause diseases that result in serious losses of economically important crops. Commensalism: Association between suckerfish (or Remora or Echeneis) and shark- The sucker fish has the dorsal fin modified as a sucker(or adhesive) disc, with the help of which it is attached to the body of shark so that the suckerfish gets free transportation. The sucker fish releases the attachment after sometime and swims in search of food. Ammensalism: When cattle trample on grass, the grass is crushed. However, the cattle do not benefit from this action nor is harmed in the process. Question 3 of 35 3. Question Consider the following pairs of organisms and their mode of nutrition: Organism Mode of nutrition 1.     Mushroom Saprotrophic   2.     Insectivorous plants Parasitic 3.     Algae Autotrophic How many of the above pairs is/are correctly matched? a) None of the pairs b) One pair only c) Two pairs only d) Three pairs only Correct Solution (c) 1.       2.       3.       Mushroom Insectivorous Plants Algae Correct Incorrect Correct Mushroom is a Saprotroph, which means it takes its nutrition from dead and decaying organic matter. It falls under the category of fungus.   Insectivorous plants are partial heterotrophs whereas other plants are autotrophs. Insectivores plants are not the examples of parasitic nutrition in plants. Parasitic plants cannot photosynthesize and hence, are depended on other living organisms for their nutrition. Insectivorous plants can photosynthesize but lack only in nitrogen. So, to fulfil this requirement, they consume the digestive nutrients in the insects.   Algae contains chlorophyll can also prepare its own food by photosynthesis. The mode of nutrition in which organisms make food themselves from simple substances is called autotrophic nutrition.   Incorrect Solution (c) 1.       2.       3.       Mushroom Insectivorous Plants Algae Correct Incorrect Correct Mushroom is a Saprotroph, which means it takes its nutrition from dead and decaying organic matter. It falls under the category of fungus.   Insectivorous plants are partial heterotrophs whereas other plants are autotrophs. Insectivores plants are not the examples of parasitic nutrition in plants. Parasitic plants cannot photosynthesize and hence, are depended on other living organisms for their nutrition. Insectivorous plants can photosynthesize but lack only in nitrogen. So, to fulfil this requirement, they consume the digestive nutrients in the insects.   Algae contains chlorophyll can also prepare its own food by photosynthesis. The mode of nutrition in which organisms make food themselves from simple substances is called autotrophic nutrition.   Question 4 of 35 4. Question Consider the following statements regarding Biomagnification: It is the process of a buildup of certain chemical substances or toxins at the higher trophic levels of a food chain. In order for biomagnifications to occur, the pollutants must be short-lived. Choose the correct code: a) 1 only b) 2 only c) Both 1 and 2 d) Neither 1 nor 2 Correct Solution (a) Statement 1 Statement 2 Correct Incorrect Biomagnification is the process of a buildup of certain chemical substances or toxins at the higher trophic levels of a food chain. In order for biomagnifications to occur, the pollutants must be long-lived. Notes: Biomagnifications- It refers to the tendency of pollutants to concentrate as they move from one trophic level to the next. In biomagnifications there is an increase in concentration of a pollutant from one link in a food chain to another. In order for biomagnifications to occur, the pollutant must be: long-lived Mobile soluble in fats Biologically active It can be seen in the case of Mercury or DDT. The concentration of Mercury/DDT increases at the successive trophic levels. DDT concentration in zooplankton gets magnified when DDT contaminated water is consumed by the Zooplanktons. In the successive trophic levels like, small fish, big fish, and at top carnivore, the magnification is much higher, which proves the event of Biomagnifications. A pollutant having properties like long life, mobile, soluble in fat, biologically active will lead to the process of biomagnifications. Incorrect Solution (a) Statement 1 Statement 2 Correct Incorrect Biomagnification is the process of a buildup of certain chemical substances or toxins at the higher trophic levels of a food chain. In order for biomagnifications to occur, the pollutants must be long-lived. Notes: Biomagnifications- It refers to the tendency of pollutants to concentrate as they move from one trophic level to the next. In biomagnifications there is an increase in concentration of a pollutant from one link in a food chain to another. In order for biomagnifications to occur, the pollutant must be: long-lived Mobile soluble in fats Biologically active It can be seen in the case of Mercury or DDT. The concentration of Mercury/DDT increases at the successive trophic levels. DDT concentration in zooplankton gets magnified when DDT contaminated water is consumed by the Zooplanktons. In the successive trophic levels like, small fish, big fish, and at top carnivore, the magnification is much higher, which proves the event of Biomagnifications. A pollutant having properties like long life, mobile, soluble in fat, biologically active will lead to the process of biomagnifications. Question 5 of 35 5. Question In the context of environment and ecology, consider the following statements regarding ‘standing crop’: It is the total amount of food grains available in a region at a given point in time. It is measured in terms of the mass of living organisms. Choose the correct code: a) 1 only b) 2 only c) Both 1 and 2 d) Neither 1 nor 2 Correct Solution (b) Statement 1 Statement 2 Incorrect Correct Each trophic level has a certain mass of living material at a particular time called as the standing crop. A standing crop is the total dried biomass of the living organisms present in a given environment. It is not limited to the food grains. The standing crop is measured as the mass of living organisms (biomass) or the number in a unit area. The biomass of a species is expressed in terms of fresh or dry weight.  Note: The total amount or weight, or energy content, of (a portion of) organisms existing in a specific area at a particular time, is known as standing crop. The non-living (inorganic nutrients) like carbon, nitrogen, phosphorus etc. present in the soil at that time is called the standing state. Incorrect Solution (b) Statement 1 Statement 2 Incorrect Correct Each trophic level has a certain mass of living material at a particular time called as the standing crop. A standing crop is the total dried biomass of the living organisms present in a given environment. It is not limited to the food grains. The standing crop is measured as the mass of living organisms (biomass) or the number in a unit area. The biomass of a species is expressed in terms of fresh or dry weight.  Note: The total amount or weight, or energy content, of (a portion of) organisms existing in a specific area at a particular time, is known as standing crop. The non-living (inorganic nutrients) like carbon, nitrogen, phosphorus etc. present in the soil at that time is called the standing state. Question 6 of 35 6. Question Consider the following statements: Fungi lives in symbiosis with the plant and passes phosphorus from the soil to the plant. Azotobacter and Azospirillum are free-living bacteria that fix atmospheric nitrogen in the soil. Which of the statements given above is/are correct? a) 1 only b) 2 only c) Both 1 and 2 d) Neither 1 nor 2 Correct Solution (c) Statement 1 Statement 2 Correct Correct Fungi are known to form symbiotic associations with plants (mycorrhiza) where the fungi absorb phosphorus from soil and passes it to the plant. Plants having such associations show other benefits also, such as resistance to root-borne pathogens, tolerance to salinity and drought, and an overall increase in plant growth and development.   In addition to nitrogen-fixing bacteria like Rhizobium that live in the plant roots, there are some free-living bacteria in the soil like Azospirillum and Azotobacter that fix atmospheric nitrogen in the soil. These bacteria fix atmospheric nitrogen into organic forms, which is used by the plant as nutrient.   Incorrect Solution (c) Statement 1 Statement 2 Correct Correct Fungi are known to form symbiotic associations with plants (mycorrhiza) where the fungi absorb phosphorus from soil and passes it to the plant. Plants having such associations show other benefits also, such as resistance to root-borne pathogens, tolerance to salinity and drought, and an overall increase in plant growth and development.   In addition to nitrogen-fixing bacteria like Rhizobium that live in the plant roots, there are some free-living bacteria in the soil like Azospirillum and Azotobacter that fix atmospheric nitrogen in the soil. These bacteria fix atmospheric nitrogen into organic forms, which is used by the plant as nutrient.   Question 7 of 35 7. Question An invasive species can be any kind of living organism that is not native to an ecosystem and causes harm. Consider the following statements: An invasive species does not have to come from another country. Higher average temperatures and changes in rain and snow patterns caused by climate change will enable some invasive plant species to move into new areas. Select the correct answer using the code given below: a) 1 only b) 2 only c) Both 1 and 2 d) Neither 1 nor 2 Correct Solution (c) Statement 1 Statement 2 Correct Correct An invasive species does not have to come from another country. Higher average temperatures and changes in rain and snow patterns caused by climate change will enable some invasive plant species to move into new areas. Notes: Invasive Species- Invasive species have devastating effects on wildlife. Human health and economies are also at risk from invasive species. Many of our commercial, agricultural, and recreational activities depend on healthy native ecosystems. Species that grow and reproduce quickly, and spread aggressively, with potential to cause harm, are given the label “invasive.” An invasive species does not have to come from another country. Invasive species are primarily spread by human activities, often unintentionally. People, and the goods we use, travel around the world very quickly, and they often carry uninvited species with them. When a new and aggressive species is introduced into an ecosystem, it may not have any natural predators or controls. It can breed and spread quickly, taking over an area. Native wildlife may not have evolved defenses against the invader, or they may not be able to compete with a species that has no predators. One way to curb the spread of invasive species is to plant native plants and remove any invasive plants in your garden. Examples if Invasive Species in India: Alternanthera philoxeroides Cassia uniflora Chromolaena odorata Eichhornia crassipes Lantana camara Parthenium hysterophorus Prosopis juliflora etc Incorrect Solution (c) Statement 1 Statement 2 Correct Correct An invasive species does not have to come from another country. Higher average temperatures and changes in rain and snow patterns caused by climate change will enable some invasive plant species to move into new areas. Notes: Invasive Species- Invasive species have devastating effects on wildlife. Human health and economies are also at risk from invasive species. Many of our commercial, agricultural, and recreational activities depend on healthy native ecosystems. Species that grow and reproduce quickly, and spread aggressively, with potential to cause harm, are given the label “invasive.” An invasive species does not have to come from another country. Invasive species are primarily spread by human activities, often unintentionally. People, and the goods we use, travel around the world very quickly, and they often carry uninvited species with them. When a new and aggressive species is introduced into an ecosystem, it may not have any natural predators or controls. It can breed and spread quickly, taking over an area. Native wildlife may not have evolved defenses against the invader, or they may not be able to compete with a species that has no predators. One way to curb the spread of invasive species is to plant native plants and remove any invasive plants in your garden. Examples if Invasive Species in India: Alternanthera philoxeroides Cassia uniflora Chromolaena odorata Eichhornia crassipes Lantana camara Parthenium hysterophorus Prosopis juliflora etc Question 8 of 35 8. Question Nitrogen deficiency can be prevented by using organic methods which requires time, but it result in a more even distribution of the added nitrogen over time. Which of the following organic methods are used for adding nitrogen to the soil: Adding composted manure to the soil Planting a green manure crop, such as borage Planting nitrogen fixing plants like peas or beans Adding coffee grounds to the soil Adding Nitrogen as a Plant Fertilizer Select the correct code: a) 1, 2 and 5 only b) 3, 4 and 5 only c) 1, 3 and 5 only d) 1, 2, 3 and 4 only Correct Solution (d) Statement 1 Statement 2 Statement 3 Statement 4 Statement 5 Correct Correct Correct Correct Incorrect Organic method Organic method Organic method Organic method Non-organic method Notes: Nitrogen fixation- Plants need nitrogen to make themselves. Without nitrogen, a plant cannot make the proteins, amino acids and even its very DNA. This is why when there is a nitrogen deficiency in the soil, plants are stunted. They simply cannot make their own cells. If there is nitrogen all around us, as it makes up 78 percent of the air we breathe. In order for plants to use the nitrogen in the air, it must be converted in some way to nitrogen in the soil. This can happen through nitrogen fixation, or nitrogen can be ‘recycled’ by composting plants and manure. There are two routes to go when fixing a nitrogen deficiency in the soil, either organic or non-organic. Organic: To correct a nitrogen deficiency using organic methods requires time, but will result in a more even distribution of the added nitrogen over time. Adding composted manure to the soil Planting a green manure crop, such as borage Planting nitrogen fixing plants like peas or beans Adding coffee grounds to the soil Non-Organic: Using a nitrogen fertilizer to fix a nitrogen deficiency in the soil will give a big, fast boost of nitrogen to the soil, but will fade quickly. Incorrect Solution (d) Statement 1 Statement 2 Statement 3 Statement 4 Statement 5 Correct Correct Correct Correct Incorrect Organic method Organic method Organic method Organic method Non-organic method Notes: Nitrogen fixation- Plants need nitrogen to make themselves. Without nitrogen, a plant cannot make the proteins, amino acids and even its very DNA. This is why when there is a nitrogen deficiency in the soil, plants are stunted. They simply cannot make their own cells. If there is nitrogen all around us, as it makes up 78 percent of the air we breathe. In order for plants to use the nitrogen in the air, it must be converted in some way to nitrogen in the soil. This can happen through nitrogen fixation, or nitrogen can be ‘recycled’ by composting plants and manure. There are two routes to go when fixing a nitrogen deficiency in the soil, either organic or non-organic. Organic: To correct a nitrogen deficiency using organic methods requires time, but will result in a more even distribution of the added nitrogen over time. Adding composted manure to the soil Planting a green manure crop, such as borage Planting nitrogen fixing plants like peas or beans Adding coffee grounds to the soil Non-Organic: Using a nitrogen fertilizer to fix a nitrogen deficiency in the soil will give a big, fast boost of nitrogen to the soil, but will fade quickly. Question 9 of 35 9. Question Which of the following statements is/are correct regarding Coastal Regulation Zone? The Coastal Regulation Zone-I is Ecologically Sensitive Area which lie between low and high tide line. The Coastal Regulation Zone IV is the territorial area under which Exploration of natural gas and extraction of salt are permitted. States have the authority to approve proposals for urban (CRZ-II) and rural (CRZ-III) areas. Select the correct answer using the codes given below: a) 1 and 2 only b) 2 and 3 only c) 1 and 3 only d) 1, 2 and 3 Correct Solution (c) Statement 1 Statement 2 Statement 3 Correct Incorrect Correct The Coastal Regulation Zone-I is Ecologically Sensitive Area which lie between low and high tide line. The Coastal Regulation Zone I is the territorial area under which Exploration of natural gas and extraction of salt are permitted. States have the authority to approve proposals for urban (CRZ-II) and rural (CRZ-III) areas. Coastal Regulation Zone: CRZ along the country has been placed in four categories, which are as follows: CRZ I – Ecologically Sensitive Areas They lie between low and high tide line. Exploration of natural gas and extraction of salt are permitted.   CRZ II – Shore Line Areas The areas that have been developed up to or close to the shoreline. Unauthorized structures are not allowed to construct in this zone.   CRZ III – Undisturbed Area Rural and Urban localities which fall outside I and II. Only certain activities related to agriculture even some public facilities are allowed in this zone.   CRZ IV – Territorial Area An area covered between Low Tide Line and 12 Nautical Miles seaward. Fishing and allied activities are permitted in this zone. Solid waste should be let off in this zone. Changes made to the regulatory framework- The system of granting clearances has been changed. States will have the authority to approve proposals for urban (CRZ-II) and rural (CRZ-III) areas. The Ministry of Environment, Forests and Climate Change will grant clearances for ecologically sensitive areas (CRZ-I), and areas falling between the low tide line and 12 nautical miles seaward. The modifications also include demarcation of a 20-metre no development zone for all islands and guidelines to deal with sensitive areas. Incorrect Solution (c) Statement 1 Statement 2 Statement 3 Correct Incorrect Correct The Coastal Regulation Zone-I is Ecologically Sensitive Area which lie between low and high tide line. The Coastal Regulation Zone I is the territorial area under which Exploration of natural gas and extraction of salt are permitted. States have the authority to approve proposals for urban (CRZ-II) and rural (CRZ-III) areas. Coastal Regulation Zone: CRZ along the country has been placed in four categories, which are as follows: CRZ I – Ecologically Sensitive Areas They lie between low and high tide line. Exploration of natural gas and extraction of salt are permitted.   CRZ II – Shore Line Areas The areas that have been developed up to or close to the shoreline. Unauthorized structures are not allowed to construct in this zone.   CRZ III – Undisturbed Area Rural and Urban localities which fall outside I and II. Only certain activities related to agriculture even some public facilities are allowed in this zone.   CRZ IV – Territorial Area An area covered between Low Tide Line and 12 Nautical Miles seaward. Fishing and allied activities are permitted in this zone. Solid waste should be let off in this zone. Changes made to the regulatory framework- The system of granting clearances has been changed. States will have the authority to approve proposals for urban (CRZ-II) and rural (CRZ-III) areas. The Ministry of Environment, Forests and Climate Change will grant clearances for ecologically sensitive areas (CRZ-I), and areas falling between the low tide line and 12 nautical miles seaward. The modifications also include demarcation of a 20-metre no development zone for all islands and guidelines to deal with sensitive areas. Question 10 of 35 10. Question Consider the following statements regarding Organic Farming: Organic farming is an integrated farming system that strives for sustainability, enhancement of soil fertility and biological diversity. It relies primarily on local renewable resources. It maximizes recycling of plant nutrients and organic matter. It uses genetically modified organisms and plant growth regulators to optimize agricultural productivity. How many of the above statements are correct? a) Only one b) Only two c) Only three d) All four Correct Solution (c) Statement 1 Statement 2 Statement 3 Statement 4 Correct Correct Correct Incorrect Organic farming is an integrated farming system that strives for sustainability, enhancement of soil fertility and biological diversity. It relies primarily on local renewable resources. It maximizes recycling of plant nutrients and organic matter. Genetically modified organisms and plant growth regulators are prohibited in Organic Farming.   Notes: Organic farming- Organic farming is a holistic system designed to optimize the agricultural productivity and to reduce the impacts on diverse communities like soil organisms, plants, livestock and humans. Organic farming is a contemporary method of crop and livestock production that involves tools which do not use chemicals and artificial tools of cropping. It is an integrated farming system that strives for sustainability, enhancement of soil fertility and biological diversity. Key characteristics of organic farming are: It relies primarily on local, renewable resources It makes efficient use of solar energy and the production potential of biological systems It maintains the fertility of the soil It maximizes recycling of plant nutrients and organic matter It does not use organisms or substances foreign to nature (e.g GMOs, chemical fertilizers or pesticides) Substances which are prohibited in organic farming are: Chemical fertilizers and pesticides Genetically modified organisms Nanomaterials Incorrect Solution (c) Statement 1 Statement 2 Statement 3 Statement 4 Correct Correct Correct Incorrect Organic farming is an integrated farming system that strives for sustainability, enhancement of soil fertility and biological diversity. It relies primarily on local renewable resources. It maximizes recycling of plant nutrients and organic matter. Genetically modified organisms and plant growth regulators are prohibited in Organic Farming.   Notes: Organic farming- Organic farming is a holistic system designed to optimize the agricultural productivity and to reduce the impacts on diverse communities like soil organisms, plants, livestock and humans. Organic farming is a contemporary method of crop and livestock production that involves tools which do not use chemicals and artificial tools of cropping. It is an integrated farming system that strives for sustainability, enhancement of soil fertility and biological diversity. Key characteristics of organic farming are: It relies primarily on local, renewable resources It makes efficient use of solar energy and the production potential of biological systems It maintains the fertility of the soil It maximizes recycling of plant nutrients and organic matter It does not use organisms or substances foreign to nature (e.g GMOs, chemical fertilizers or pesticides) Substances which are prohibited in organic farming are: Chemical fertilizers and pesticides Genetically modified organisms Nanomaterials Question 11 of 35 11. Question Consider the following statements regarding seed banks: Seeds are stored under low temperatures that keep seeds dormant till they are needed for replanting. Seed banks does not prevent the loss of Agro-biodiversity. Seeds that are stored in seed banks can be made easily available to scientists and researchers who wish to study these seeds. How many of the above statements are correct? a) Only one b) Only two c) All three d) None Correct Solution (b) Statement 1 Statement 2 Statement 3 Correct Incorrect Correct Seeds are stored under low temperatures that keep seeds dormant till they are needed for replanting. Seed banks do prevent the loss of Agro-biodiversity. Seeds that are stored in seed banks can be made easily available to scientists and researchers who wish to study these seeds especially if such research could lead to improvement of crop production. Note: Seed Bank- A seed bank is a kind of gene bank that stores seeds of food crops or any rare species of plants. Seed banks protect us against the extinction of plant species due to natural catastrophes, outbreak of diseases and even deliberate non-use for a long time. Seed banks make our diminishing biodiversity available to future generations. Seeds in a seed bank are dried to suitably low moisture content and stored at -18°C or below. The bank also documents the plan’s identity, sampling location, seed quantity, farming systems in which the crop was grown etc. Svalbard Global Seed Vault is a facility on a remote island in the Arctic Ocean and it houses the world’s largest collection of seeds. The seeds can be used in event of global catastrophe or when some species is lost due to natural disasters; hence it called the doomsday vault. India’s seed vault is located at Chang La, Ladakh in the Himalayas which stores over 5000 seed accessions (One accession consists of a set of seeds of one species collected from different locations or different populations). The vault is a joint venture of National Bureau of Plant Genetic Resources (which comes under Indian Council of Agriculture Research) and the Defense Institute of High-Altitude Research (under Defense Research and Development Organization). Incorrect Solution (b) Statement 1 Statement 2 Statement 3 Correct Incorrect Correct Seeds are stored under low temperatures that keep seeds dormant till they are needed for replanting. Seed banks do prevent the loss of Agro-biodiversity. Seeds that are stored in seed banks can be made easily available to scientists and researchers who wish to study these seeds especially if such research could lead to improvement of crop production. Note: Seed Bank- A seed bank is a kind of gene bank that stores seeds of food crops or any rare species of plants. Seed banks protect us against the extinction of plant species due to natural catastrophes, outbreak of diseases and even deliberate non-use for a long time. Seed banks make our diminishing biodiversity available to future generations. Seeds in a seed bank are dried to suitably low moisture content and stored at -18°C or below. The bank also documents the plan’s identity, sampling location, seed quantity, farming systems in which the crop was grown etc. Svalbard Global Seed Vault is a facility on a remote island in the Arctic Ocean and it houses the world’s largest collection of seeds. The seeds can be used in event of global catastrophe or when some species is lost due to natural disasters; hence it called the doomsday vault. India’s seed vault is located at Chang La, Ladakh in the Himalayas which stores over 5000 seed accessions (One accession consists of a set of seeds of one species collected from different locations or different populations). The vault is a joint venture of National Bureau of Plant Genetic Resources (which comes under Indian Council of Agriculture Research) and the Defense Institute of High-Altitude Research (under Defense Research and Development Organization). Question 12 of 35 12. Question With respect to ‘Zero Budget Natural Farming’ (ZBNF), consider the following statements: It aims at enhancing farm biodiversity and ecosystem services. It uses in situ resources to rejuvenate soil instead of using external chemical fertilizers. Four pillars of ZBNF include Jiwamitra, Bijamrita, Acchadana and Waaphasa. Government promotes ZBNF through Paramparagat Krishi Vikas Yojna and Rashtriya Krishi Vikas Yojna. How many of the above statements are correct? a) Only one b) Only two c) Only three d) All four Correct Solution (d) Statement 1 Statement 2 Statement 3 Statement 4 Correct Correct Correct Correct ZBNF aims at enhancing farm biodiversity and ecosystem services. ZBNF uses in situ resources to rejuvenate soil instead of using external chemical fertilizers. Four pillars of ZBNF include Jiwamitra, Bijamrita, Acchadana and Waaphasa. Government promotes ZBNF through Paramparagat Krishi Vikas Yojna and Rashtriya Krishi Vikas Yojna. Note: ZBNF is based on 4 pillars: Jeevamrutha: It is a mixture of fresh cow dung and aged cow urine (both from India’s indigenous cow breed), jaggery, pulse flour, water and soil; to be applied on farmland. Bijamrita: It is a concoction of neem leaves & pulp, tobacco and green chilies prepared for insect and pest management, that can be used to treat seeds. Acchadana (Mulching): It protects topsoil during cultivation and does not destroy it by tilling. Whapasa: It is the condition where there are both air molecules and water molecules present in the soil. Thereby helping in reducing irrigation requirement. Incorrect Solution (d) Statement 1 Statement 2 Statement 3 Statement 4 Correct Correct Correct Correct ZBNF aims at enhancing farm biodiversity and ecosystem services. ZBNF uses in situ resources to rejuvenate soil instead of using external chemical fertilizers. Four pillars of ZBNF include Jiwamitra, Bijamrita, Acchadana and Waaphasa. Government promotes ZBNF through Paramparagat Krishi Vikas Yojna and Rashtriya Krishi Vikas Yojna. Note: ZBNF is based on 4 pillars: Jeevamrutha: It is a mixture of fresh cow dung and aged cow urine (both from India’s indigenous cow breed), jaggery, pulse flour, water and soil; to be applied on farmland. Bijamrita: It is a concoction of neem leaves & pulp, tobacco and green chilies prepared for insect and pest management, that can be used to treat seeds. Acchadana (Mulching): It protects topsoil during cultivation and does not destroy it by tilling. Whapasa: It is the condition where there are both air molecules and water molecules present in the soil. Thereby helping in reducing irrigation requirement. Question 13 of 35 13. Question Consider the following statements regarding Hangul: It is the state animal of Himachal Pradesh. It is restricted to the Dachigam National Park. IUCN status is Endangered. How many of the above statements are correct? a) Only one b) Only two c) All three d) None Correct Solution (a) Statement 1 Statement 2 Statement 3 Incorrect Correct Incorrect The Kashmir stag also called Hangul is a subspecies of Central Asian Red Deer native to northern India. Snow Leopard is the state animal of Himachal Pradesh. It is restricted to the Dachigam National Park. The Kashmir stag IUCN status is Critically Endangered. Incorrect Solution (a) Statement 1 Statement 2 Statement 3 Incorrect Correct Incorrect The Kashmir stag also called Hangul is a subspecies of Central Asian Red Deer native to northern India. Snow Leopard is the state animal of Himachal Pradesh. It is restricted to the Dachigam National Park. The Kashmir stag IUCN status is Critically Endangered. Question 14 of 35 14. Question Consider the following pairs regarding the regions/states and the corresponding hotspots: Region/State Hotspots Assam Himalaya Mizoram Indo-Burma Andaman group of islands Sundaland Nicobar group of islands Western Ghats and Sri Lanka How many of the above pairs is/are correctly matched? a) One pair only b) Two pairs only c) Three pairs only d) All four pairs Correct Solution (b) Statement 1 Statement 2 Statement 3 Statement 4 Correct Correct Incorrect Incorrect Assam – Himalaya Mizoram – Indo-Burma Andaman group of Islands – Indo-Burma Nicobar group of Islands – Sundaland Incorrect Solution (b) Statement 1 Statement 2 Statement 3 Statement 4 Correct Correct Incorrect Incorrect Assam – Himalaya Mizoram – Indo-Burma Andaman group of Islands – Indo-Burma Nicobar group of Islands – Sundaland Question 15 of 35 15. Question Consider the following statement regarding Social Forestry? Among five different types of Social forestry, Silviculture is one of them. Extension forestry is a type of forestry that helps in increasing the boundaries of the forests Farm forestry is the management of trees for a specific purpose within a farming context. How many of the statements given above is/are correct? a) Only one b) Only two c) All three d) None Correct Solution (c) Statement 1 Statement 2 Statement 3 Correct Correct Correct There are 5 different types of social forestry, which are listed below: 1. Scientific Forestry or Silviculture 2. Farm Forestry 3. Community Forestry 4. Agro-Forestry 5. Extension Forestry Extension forestry is a type of forestry helps in increasing the boundaries of the forests. Planting of trees on the government wastelands, panchayat lands, village common lands. Farm forestry is the management of trees for a specific purpose within a farming context. Incorrect Solution (c) Statement 1 Statement 2 Statement 3 Correct Correct Correct There are 5 different types of social forestry, which are listed below: 1. Scientific Forestry or Silviculture 2. Farm Forestry 3. Community Forestry 4. Agro-Forestry 5. Extension Forestry Extension forestry is a type of forestry helps in increasing the boundaries of the forests. Planting of trees on the government wastelands, panchayat lands, village common lands. Farm forestry is the management of trees for a specific purpose within a farming context. Question 16 of 35 16. Question If the population of any species has been reduced by more than 90% over the last 10 years and its population size is less than 50 mature individuals, it will be categorized as: a) Endangered (EN) b) Vulnerable (VU) c) Critically Endangered (CR) d) Extinct (EX) Correct Solution (c) Critically Endangered Category: A species is categorized as Critically Endangered when it meets any of the following criteria: Reduction in the population of more than 90% over the last 10 years. The population size is less than 50 mature individuals. Quantitative analysis showing the probability of extinction in the wild in at least 50% in their 10 years. Incorrect Solution (c) Critically Endangered Category: A species is categorized as Critically Endangered when it meets any of the following criteria: Reduction in the population of more than 90% over the last 10 years. The population size is less than 50 mature individuals. Quantitative analysis showing the probability of extinction in the wild in at least 50% in their 10 years. Question 17 of 35 17. Question Consider the following statements: Stenohaline organisms are species that can only tolerate specific ranges of salinities. Euryhaline organisms can withstand different salinities and can cope with a wide range of different environments. Which of the statements given above is/are correct? a) 1 only b) 2 only c) Both 1 and 2 d) Neither 1 nor 2 Correct Solution (c) Statement 1 Statement 2 Correct Correct Stenohaline organisms are species that can only tolerate specific ranges of salinities. They are opposite of a stenohaline organism. A euryhaline organism can withstand different salinities and can cope with a wide range of different environments. Euryhaline organisms are commonly found in habitats such as estuaries and tide pools. Note: Osmoregulation involves active regulation of the water content within a living system irrespective of the water content of the surrounding environment. Different species of organisms use different mechanisms for osmoregulation. Therefore, in the context of osmoregulation, species could be divided into two categories; osmoconformers and osmoregulators. Under osmoconformers, stenohaline organisms are included, and under osmoregulators euryhaline organisms are included. Euryhaline organisms have the ability to survive in varying concentrations of salts while stenohaline organisms thrive at a limited range of salinity. This is the difference between euryhaline and stenohaline. Incorrect Solution (c) Statement 1 Statement 2 Correct Correct Stenohaline organisms are species that can only tolerate specific ranges of salinities. They are opposite of a stenohaline organism. A euryhaline organism can withstand different salinities and can cope with a wide range of different environments. Euryhaline organisms are commonly found in habitats such as estuaries and tide pools. Note: Osmoregulation involves active regulation of the water content within a living system irrespective of the water content of the surrounding environment. Different species of organisms use different mechanisms for osmoregulation. Therefore, in the context of osmoregulation, species could be divided into two categories; osmoconformers and osmoregulators. Under osmoconformers, stenohaline organisms are included, and under osmoregulators euryhaline organisms are included. Euryhaline organisms have the ability to survive in varying concentrations of salts while stenohaline organisms thrive at a limited range of salinity. This is the difference between euryhaline and stenohaline. Question 18 of 35 18. Question Consider the following statements regarding Bioremediation techniques: Bio-venting involves the release of substantial quantities of microorganisms to carry out a specific remediation task in a given contaminated environment. Bio-augmentation involves supplying of air and nutrients through wells to contaminated soil to stimulate the growth of microorganisms. Which of the above statements is/are correct? a) 1 only b) 2 only c) Both 1 and 2 d) Neither 1 nor 2 Correct Solution (d) Statement 1 Statement 2 Incorrect Incorrect Bio-venting involves supplying of air and nutrients through wells to contaminated soil to stimulate the growth of microorganisms. Bioaugmentation involves the release of substantial quantities of microorganisms to carry out a specific remediation task in a given contaminated environment. Note: Bioremediation: Bioremediation is the use of microorganisms (bacteria and fungi) to degrade the environmental contaminants into less toxic forms. The microorganisms may be indigenous to a contaminated area or they may be isolated from elsewhere and brought to the contaminated site. The process of bioremediation can be monitored indirectly by measuring the Oxidation Reduction Potential or redox in soil and groundwater, together with pH, temperature, oxygen content, electron acceptor/donor concentrations, and concentration of breakdown products (e.g., carbon dioxide) In-Situ Bioremediation: In Situ — It involves treatment of the contaminated material at the site. Bioventing: Supply of air and nutrients through wells to contaminated soil to stimulate the growth of indigenous bacteria. It is used for simple hydrocarbons and can be used where the contamination is deep under the surface. Biosparging: Injection of air under pressure below the water table to increase groundwater oxygen concentrations and enhance the rate of biological degradation of contaminants by naturally occurring bacteria Bioaugmentation: Microorganisms are imported to a contaminated site to enhance degradation process. Using bioremediation techniques, TERI has developed a mixture of bacteria called ‘Oilzapper and Oilivorous-S’ which degrades the pollutants of oil-contaminated sites, leaving behind no harmful residues. This technique is not only environment friendly, but also highly cost-effective. Ex Situ Bioremediation: Ex Situ — involves the removal of the contaminated material to be treated elsewhere. Land Farming: contaminated soil is excavated and spread over a prepared bed and periodically tilled until pollutants are degraded. The goal is to stimulate indigenous biodegradative microorganisms and facilitate their aerobic degradation of contaminants. Biopiles: it is a hybrid of land farming and composting. Essentially, engineered cells are constructed as aerated composted piles. Typically used for treatment of surface contamination with petroleum hydrocarbons. Bioreactors: it involves the processing of contaminated solid material (soil, sediment, sludge) or water through an engineered containment system. Composting: Composting is nature’s process of recycling decomposed organic materials into a rich soil known as compost. Incorrect Solution (d) Statement 1 Statement 2 Incorrect Incorrect Bio-venting involves supplying of air and nutrients through wells to contaminated soil to stimulate the growth of microorganisms. Bioaugmentation involves the release of substantial quantities of microorganisms to carry out a specific remediation task in a given contaminated environment. Note: Bioremediation: Bioremediation is the use of microorganisms (bacteria and fungi) to degrade the environmental contaminants into less toxic forms. The microorganisms may be indigenous to a contaminated area or they may be isolated from elsewhere and brought to the contaminated site. The process of bioremediation can be monitored indirectly by measuring the Oxidation Reduction Potential or redox in soil and groundwater, together with pH, temperature, oxygen content, electron acceptor/donor concentrations, and concentration of breakdown products (e.g., carbon dioxide) In-Situ Bioremediation: In Situ — It involves treatment of the contaminated material at the site. Bioventing: Supply of air and nutrients through wells to contaminated soil to stimulate the growth of indigenous bacteria. It is used for simple hydrocarbons and can be used where the contamination is deep under the surface. Biosparging: Injection of air under pressure below the water table to increase groundwater oxygen concentrations and enhance the rate of biological degradation of contaminants by naturally occurring bacteria Bioaugmentation: Microorganisms are imported to a contaminated site to enhance degradation process. Using bioremediation techniques, TERI has developed a mixture of bacteria called ‘Oilzapper and Oilivorous-S’ which degrades the pollutants of oil-contaminated sites, leaving behind no harmful residues. This technique is not only environment friendly, but also highly cost-effective. Ex Situ Bioremediation: Ex Situ — involves the removal of the contaminated material to be treated elsewhere. Land Farming: contaminated soil is excavated and spread over a prepared bed and periodically tilled until pollutants are degraded. The goal is to stimulate indigenous biodegradative microorganisms and facilitate their aerobic degradation of contaminants. Biopiles: it is a hybrid of land farming and composting. Essentially, engineered cells are constructed as aerated composted piles. Typically used for treatment of surface contamination with petroleum hydrocarbons. Bioreactors: it involves the processing of contaminated solid material (soil, sediment, sludge) or water through an engineered containment system. Composting: Composting is nature’s process of recycling decomposed organic materials into a rich soil known as compost. Question 19 of 35 19. Question Consider the following statement about the Oil spills: Oil spills at sea decreases the Dissolved Oxygen (DOs) in the water and cause harm to the organisms. Oil spills at sea decreases the Chemical oxygen demand (COD) in the water and cause harm to the organisms. Using bioremediation techniques, TERI has developed a mixture of bacteria called ‘Oilzapper’ which degrades the pollutants of oil-contaminated sites. How many of the statements given above is/are correct? a) Only one b) Only two c) All three d) None Correct Solution (b) Statement 1 Statement 2 Statement 3 Correct Incorrect Correct Oil spills at sea decreases the Dissolved Oxygen (DOs) in the water and cause harm to the organisms. Oil spills at sea increase the Chemical oxygen demand (COD) in the water and cause harm to the organisms. COD is often used as a measurement of pollutants in water, wastewater, and aqueous hazardous wastes. One application of the COD test is to measure soluble COD in wastewater, since characterization of total COD in wastewater is critical for accurate modeling of biotransformation in wastewater treatment processes. Using bioremediation techniques, TERI has developed a mixture of bacteria called ‘Oilzapper’ which degrades the pollutants of oil-contaminated sites, leaving behind no harmful residues. This technique is not only environment friendly, but also highly cost-effective. Note: Oil Spill: An oil spill refers to an uncontrolled release of gasoline, crude oil, other oil products, and fuels into the environment. An oil spill can pollute water, air, and land by destroying natural habitats. The production on continental shelves, petroleum exploration, and transportation in large vessels have increased the cases of oil spills. According to Marine Insight, nearly 706 million gallons of waste oil pollute the ocean every year. Oil waste disposal and land drainage practices increase pollution levels in water bodies. Ideally, accidents involving pipelines, oil tankers, or drilling rigs are the primary causes of oil spills in oceans, rivers, and bays. Students preparing for UPSC prelims and IAS exams should know the impact of oil spills on the environment. Impact of Oil Spill on the Environment and Humans: Oil spill causes health hazards to natives depending on seafood. Crude oil destroys the insulating capacity of bird feathers and coats them will oil. Marine life and bird coated in oil are prone to hypothermia and death. Oil forms a thin layer on the ocean surfaces and prevents sufficient sunlight from entering the ocean surface. This reduces the dissolved oxygen levels in water, making it difficult for marine life to breathe. Oil spills at sea increase the Chemical oxygen demand (COD) in the water and cause harm to the organisms. Ingesting toxic animals or preys kills predators, ultimately destroying the reproductive rate and habitat. Mangroves and saltwater marshes that protect the earth from natural disasters are prone to significant oil spills. This can be dangerous for the environment. Incorrect Solution (b) Statement 1 Statement 2 Statement 3 Correct Incorrect Correct Oil spills at sea decreases the Dissolved Oxygen (DOs) in the water and cause harm to the organisms. Oil spills at sea increase the Chemical oxygen demand (COD) in the water and cause harm to the organisms. COD is often used as a measurement of pollutants in water, wastewater, and aqueous hazardous wastes. One application of the COD test is to measure soluble COD in wastewater, since characterization of total COD in wastewater is critical for accurate modeling of biotransformation in wastewater treatment processes. Using bioremediation techniques, TERI has developed a mixture of bacteria called ‘Oilzapper’ which degrades the pollutants of oil-contaminated sites, leaving behind no harmful residues. This technique is not only environment friendly, but also highly cost-effective. Note: Oil Spill: An oil spill refers to an uncontrolled release of gasoline, crude oil, other oil products, and fuels into the environment. An oil spill can pollute water, air, and land by destroying natural habitats. The production on continental shelves, petroleum exploration, and transportation in large vessels have increased the cases of oil spills. According to Marine Insight, nearly 706 million gallons of waste oil pollute the ocean every year. Oil waste disposal and land drainage practices increase pollution levels in water bodies. Ideally, accidents involving pipelines, oil tankers, or drilling rigs are the primary causes of oil spills in oceans, rivers, and bays. Students preparing for UPSC prelims and IAS exams should know the impact of oil spills on the environment. Impact of Oil Spill on the Environment and Humans: Oil spill causes health hazards to natives depending on seafood. Crude oil destroys the insulating capacity of bird feathers and coats them will oil. Marine life and bird coated in oil are prone to hypothermia and death. Oil forms a thin layer on the ocean surfaces and prevents sufficient sunlight from entering the ocean surface. This reduces the dissolved oxygen levels in water, making it difficult for marine life to breathe. Oil spills at sea increase the Chemical oxygen demand (COD) in the water and cause harm to the organisms. Ingesting toxic animals or preys kills predators, ultimately destroying the reproductive rate and habitat. Mangroves and saltwater marshes that protect the earth from natural disasters are prone to significant oil spills. This can be dangerous for the environment. Question 20 of 35 20. Question Forest fires are uncontrolled fires that burn surface vegetation. Which of the following are the impacts of the forest fire? It replenishes carbon sinks. There is loss of valuable timber resources. It leads to the ozone layer depletion. Select the correct answer using the code given below. a) 2 and 3 only b) 1 and 2 only c) 2 only d) 1, 2 and 3 Correct Solution (a) Statement 1 Statement 2 Statement 3 Incorrect Correct Correct Forest fire causes depletion of carbon sinks. Forest fire causes loss of valuable timber resources. Forest fire leads to the depletion of ozone layer. Satellite observations of the ozone layer after widespread fires burned across Australia in 2020 revealed that it was being depleted by chemicals released in the smoke. These chemicals sped up the processes that normally regulate ozone in our atmosphere, reducing the UV protection this layer affords.   Notes:Forest Fires: Forest fires are uncontrolled fires that burn surface vegetation. Forest fires cause following adverse impacts: Loss of valuable timber resources. Depletion of carbon sinks. Degradation of water catchment areas resulting in loss of water. Loss of biodiversity and extinction of plants and animals. Loss of wildlife habitat and depletion of wildlife. Loss of natural regeneration and reduction in forest cover and production. Global warming resulting in rising temperature. Loss of carbon sink resource and an increase in the percentage of CO2 in the atmosphere. Change in the microclimate of the area making it unhealthy living conditions. Soil erosion affecting the productivity of soils and production. Ozone layer depletion. Health problems leading to diseases. Indirect effects on agricultural production leading to loss of livelihood. Incorrect Solution (a) Statement 1 Statement 2 Statement 3 Incorrect Correct Correct Forest fire causes depletion of carbon sinks. Forest fire causes loss of valuable timber resources. Forest fire leads to the depletion of ozone layer. Satellite observations of the ozone layer after widespread fires burned across Australia in 2020 revealed that it was being depleted by chemicals released in the smoke. These chemicals sped up the processes that normally regulate ozone in our atmosphere, reducing the UV protection this layer affords.   Notes:Forest Fires: Forest fires are uncontrolled fires that burn surface vegetation. Forest fires cause following adverse impacts: Loss of valuable timber resources. Depletion of carbon sinks. Degradation of water catchment areas resulting in loss of water. Loss of biodiversity and extinction of plants and animals. Loss of wildlife habitat and depletion of wildlife. Loss of natural regeneration and reduction in forest cover and production. Global warming resulting in rising temperature. Loss of carbon sink resource and an increase in the percentage of CO2 in the atmosphere. Change in the microclimate of the area making it unhealthy living conditions. Soil erosion affecting the productivity of soils and production. Ozone layer depletion. Health problems leading to diseases. Indirect effects on agricultural production leading to loss of livelihood. Question 21 of 35 21. Question Consider the following statements: Gaucher disease is the result of a build-up of certain fatty substances in certain organs, particularly in the spleen and liver. Tyrosinemia type I is an inherited metabolic disorder in which the body lacks an enzyme called fumarylacetoacetate hydrolase (FAH). Wilson’s disease is an inherited disorder that is caused by abnormal copper accumulation in your liver, brain, and other vital organs. How many of the above statements are correct? a) Only one b) Only two c) All three d) None Correct Solution (c) Context: The Union Health Ministry has made available generic drugs for rare diseases for the first time. Presently, they are made available for four rare diseases Tyrosinemia-Type 1, Gauchers Disease, Wilson’s Disease, and the Dravet-Lennox Gastaut Syndrome. There is no universally accepted definition of rare diseases; However, the World Health Organisation (WHO) defines rare disease as often debilitating lifelong disease or disorder condition with a prevalence of one or less per 1,000 population. Indian Council of Medical Research (ICMR) defines rare disease as a disease with one occurrence in 2,500 people. Gaucher disease is the result of a build-up of certain fatty substances in certain organs, particularly in the spleen and liver. Hence statement 1 is correct. Tyrosinemia type I is an inherited metabolic disorder in which the body lacks an enzyme called fumarylacetoacetate hydrolase (FAH). Hence statement 2 is correct. FAH is needed to break down the amino acid tyrosine. Wilson’s disease is an inherited disorder that is caused by abnormal copper accumulation in your liver, brain, and other vital organs. Hence statement 3 is correct. Dravet-Lennox gastaut syndrome is a rare drug-resistant epileptic condition, characterised by seizures.   Incorrect Solution (c) Context: The Union Health Ministry has made available generic drugs for rare diseases for the first time. Presently, they are made available for four rare diseases Tyrosinemia-Type 1, Gauchers Disease, Wilson’s Disease, and the Dravet-Lennox Gastaut Syndrome. There is no universally accepted definition of rare diseases; However, the World Health Organisation (WHO) defines rare disease as often debilitating lifelong disease or disorder condition with a prevalence of one or less per 1,000 population. Indian Council of Medical Research (ICMR) defines rare disease as a disease with one occurrence in 2,500 people. Gaucher disease is the result of a build-up of certain fatty substances in certain organs, particularly in the spleen and liver. Hence statement 1 is correct. Tyrosinemia type I is an inherited metabolic disorder in which the body lacks an enzyme called fumarylacetoacetate hydrolase (FAH). Hence statement 2 is correct. FAH is needed to break down the amino acid tyrosine. Wilson’s disease is an inherited disorder that is caused by abnormal copper accumulation in your liver, brain, and other vital organs. Hence statement 3 is correct. Dravet-Lennox gastaut syndrome is a rare drug-resistant epileptic condition, characterised by seizures.   Question 22 of 35 22. Question Consider the following statements about the Nugu Wildlife Sanctuary: It is a part of the Nilgiri Biosphere Reserve located in Tamil Nadu. It has been declared an eco-sensitive zone by the Ministry of Environment, Forests and Climate Change. Its vegetation includes southern mixed deciduous forest and dry deciduous scrub forest. How many of the above statements are correct? a) Only one b) Only two c) All three d) None Correct Solution (b) The backwaters of the Nugu dam form part of the sanctuary. It is a part of the Nilgiri Biosphere Reserve located in Karnataka. Hence statement 1 is incorrect. It is located near Bandipur Tiger Reserve. It has been declared an eco-sensitive zone by the Ministry of Environment, Forests and Climate Change. Hence statement 2 is correct. Its fauna includes tiger, elephant, leopard, wild dog, striped hyena, sloth bear, gaur, sambhar, chital, and four-horned antelope. In addition, the sanctuary also harbours two important riverine wildlife species, – the smooth-coated otter and the marsh crocodile. Its vegetation includes southern mixed deciduous forest and dry deciduous scrub forest. Hence statement 3 is correct. The Nugu Wildlife Sanctuary receives rainfall both from southwest and northeast monsoons. Incorrect Solution (b) The backwaters of the Nugu dam form part of the sanctuary. It is a part of the Nilgiri Biosphere Reserve located in Karnataka. Hence statement 1 is incorrect. It is located near Bandipur Tiger Reserve. It has been declared an eco-sensitive zone by the Ministry of Environment, Forests and Climate Change. Hence statement 2 is correct. Its fauna includes tiger, elephant, leopard, wild dog, striped hyena, sloth bear, gaur, sambhar, chital, and four-horned antelope. In addition, the sanctuary also harbours two important riverine wildlife species, – the smooth-coated otter and the marsh crocodile. Its vegetation includes southern mixed deciduous forest and dry deciduous scrub forest. Hence statement 3 is correct. The Nugu Wildlife Sanctuary receives rainfall both from southwest and northeast monsoons. Question 23 of 35 23. Question Consider the following statements: Budapest Convention on Cybercrime is the first international treaty that seeks to address Internet and cybercrime by harmonizing national laws and India is a signatory to this convention. The CERT-IN (Computer Emergency Response Team- India) is the national nodal agency for responding to computer security incidents or cyber security threats. Which of the statements given above is/are correct? a) 1 only b) 2 only c) Both 1 and 2 d) Neither 1 nor 2 Correct Solution (b) Budapest Convention on Cybercrime is the first international treaty that seeks to address Internet and cybercrime by harmonizing national laws and India is not a signatory to this convention. Hence statement 1 is incorrect. The CERT-IN (Computer Emergency Response Team- India) is the national nodal agency for responding to computer security incidents or cyber security threats. Hence statement 2 is correct. CERT-IN was formed in 2004 by the Government of India under Information Technology Act, 2000 Section (70B). The Ministry of Electronics and Information Technology (MeitY) is its nodal ministry. Its vision is a proactive contribution to securing India’s cyberspace. Incorrect Solution (b) Budapest Convention on Cybercrime is the first international treaty that seeks to address Internet and cybercrime by harmonizing national laws and India is not a signatory to this convention. Hence statement 1 is incorrect. The CERT-IN (Computer Emergency Response Team- India) is the national nodal agency for responding to computer security incidents or cyber security threats. Hence statement 2 is correct. CERT-IN was formed in 2004 by the Government of India under Information Technology Act, 2000 Section (70B). The Ministry of Electronics and Information Technology (MeitY) is its nodal ministry. Its vision is a proactive contribution to securing India’s cyberspace. Question 24 of 35 24. Question Consider the following statements: Under Article 233, high court judges are appointed by the Governor of the State in consultation with the High Court exercising jurisdiction in relation to such State. All India Judicial Service (AIJS) is a reform that aims to centralize the recruitment of judges at the level of additional district judges and district judges for all states. All India Judicial Service (AIJS) was first proposed by the Law Commission’s 1958 ‘Report on Reforms on Judicial Administration’. How many of the above statements are correct? a) Only one b) Only two c) All three d) None Correct Solution (b) Under Article 233, district court judges are appointed by the Governor of the State in consultation with the High Court exercising jurisdiction in relation to such State. The judges of a high court are appointed by the President. Hence statement 1 is incorrect. All India Judicial Service (AIJS) is a reform that aims to centralize the recruitment of judges at the level of additional district judges and district judges for all states. Hence statement 2 is correct. The original constitution didn’t provide for the creation of an All India Judicial Service (AIJS). All India Judicial Service (AIJS) was first proposed by the Law Commission’s 1958 ‘Report on Reforms on Judicial Administration’. Hence statement 3 is correct. Article 312 was amended by the 42nd Constitution (Amendment) Act, 1976 to provide for an AIJS by a Parliamentary law. It enables the creation of the AIJS at the District Judge level. Incorrect Solution (b) Under Article 233, district court judges are appointed by the Governor of the State in consultation with the High Court exercising jurisdiction in relation to such State. The judges of a high court are appointed by the President. Hence statement 1 is incorrect. All India Judicial Service (AIJS) is a reform that aims to centralize the recruitment of judges at the level of additional district judges and district judges for all states. Hence statement 2 is correct. The original constitution didn’t provide for the creation of an All India Judicial Service (AIJS). All India Judicial Service (AIJS) was first proposed by the Law Commission’s 1958 ‘Report on Reforms on Judicial Administration’. Hence statement 3 is correct. Article 312 was amended by the 42nd Constitution (Amendment) Act, 1976 to provide for an AIJS by a Parliamentary law. It enables the creation of the AIJS at the District Judge level. Question 25 of 35 25. Question Consider the following statements about the Indian Ocean Tuna Commission: It is an intergovernmental organization responsible for the management of tuna and tuna-like species in the Pacific Ocean. The decisions taken by the commission are binding on all members and non-contracting parties. Its headquarters are located in Victoria, Seychelles. How many of the above statements are correct? a) Only one b) Only two c) All three d) None Correct Solution (b) The Indian Ocean Tuna Commission is an intergovernmental organization responsible for the management of tuna and tuna-like species in the Indian Ocean. Hence statement 1 is incorrect. It was established in 1993 and came into force in 1996. It seeks to do this by promoting cooperation among its contracting parties (members) and cooperating with non-contracting parties in order to ensure the conservation and appropriate utilisation of fish stocks and encourage the sustainable development of fisheries. The decisions taken by the commission are binding on all members and non-contracting parties. Hence statement 2 is correct. It has 30 contracting parties (India joined as a member in 1995) and Liberia is a cooperating non-contracting party. Its headquarters are located in Victoria, Seychelles. Hence statement 3 is correct. Incorrect Solution (b) The Indian Ocean Tuna Commission is an intergovernmental organization responsible for the management of tuna and tuna-like species in the Indian Ocean. Hence statement 1 is incorrect. It was established in 1993 and came into force in 1996. It seeks to do this by promoting cooperation among its contracting parties (members) and cooperating with non-contracting parties in order to ensure the conservation and appropriate utilisation of fish stocks and encourage the sustainable development of fisheries. The decisions taken by the commission are binding on all members and non-contracting parties. Hence statement 2 is correct. It has 30 contracting parties (India joined as a member in 1995) and Liberia is a cooperating non-contracting party. Its headquarters are located in Victoria, Seychelles. Hence statement 3 is correct. Question 26 of 35 26. Question Consider the following statements regarding Psyche Spacecraft: It is an ISRO space mission to explore the origin of planetary cores by orbiting and studying the asteroid 16 Psyche. It is managed by the Jet Propulsion Laboratory (JPL) of NASA. It is the first mission to use laser optical communications beyond the Earth-Moon system. It is the first interplanetary spacecraft to use solar-powered hall effect thrusters for propulsion and orbital manoeuvring. Choose the correct code: a) Only one b) Only two c) Only three d) All four Correct Solution (c) Psyche Spacecraft is a NASA space mission to explore the origin of planetary cores by orbiting and studying the asteroid 16 Psyche. Hence statement 1 is incorrect. Scientists believe this asteroid is the nickel-iron core of an early planet, studying which could provide unique insights into the impenetrable iron core of our own planet. It is managed by the Jet Propulsion Laboratory (JPL) of NASA. Hence statement 2 is correct. The spacecraft will not land on the asteroid but will orbit it from August 2029 through late 2031. It is the first mission to use laser optical communications beyond the Earth-Moon system. Hence statement 3 is correct. The Psyche mission is expected to provide critical insights into the formation and evolution of our solar system. It is the first interplanetary spacecraft to use solar-powered hall effect thrusters for propulsion and orbital manoeuvring. Hence statement 4 is correct. Incorrect Solution (c) Psyche Spacecraft is a NASA space mission to explore the origin of planetary cores by orbiting and studying the asteroid 16 Psyche. Hence statement 1 is incorrect. Scientists believe this asteroid is the nickel-iron core of an early planet, studying which could provide unique insights into the impenetrable iron core of our own planet. It is managed by the Jet Propulsion Laboratory (JPL) of NASA. Hence statement 2 is correct. The spacecraft will not land on the asteroid but will orbit it from August 2029 through late 2031. It is the first mission to use laser optical communications beyond the Earth-Moon system. Hence statement 3 is correct. The Psyche mission is expected to provide critical insights into the formation and evolution of our solar system. It is the first interplanetary spacecraft to use solar-powered hall effect thrusters for propulsion and orbital manoeuvring. Hence statement 4 is correct. Question 27 of 35 27. Question The International Labour Organization (ILO) has eight core conventions. India has ratified six conventions out of eight conventions. Which of the following conventions are ratified by India? Equal Remuneration Convention Abolition of Forced Labour Convention Freedom of Association and Protection of Right to Organised Convention Discrimination (Employment Occupation) Convention How many of the above statements are correct? a) Only one b) Only two c) Only three d) All four Correct Solution (c) The International Labour Organization (ILO) has eight core conventions. India has ratified six conventions out of eight conventions. They are: Forced Labour Convention (No. 29) Abolition of Forced Labour Convention (No.105) Equal Remuneration Convention (No.100) Discrimination (Employment Occupation) Convention (No.111) Minimum Age Convention (No.138) Worst forms of Child Labour Convention (No.182)   ILO core conventions not ratified by India: Freedom of Association and Protection of Right to Organised Convention (No.87) Right to Organize and Collective Bargaining Convention (No.98) Hence option c is correct. Incorrect Solution (c) The International Labour Organization (ILO) has eight core conventions. India has ratified six conventions out of eight conventions. They are: Forced Labour Convention (No. 29) Abolition of Forced Labour Convention (No.105) Equal Remuneration Convention (No.100) Discrimination (Employment Occupation) Convention (No.111) Minimum Age Convention (No.138) Worst forms of Child Labour Convention (No.182)   ILO core conventions not ratified by India: Freedom of Association and Protection of Right to Organised Convention (No.87) Right to Organize and Collective Bargaining Convention (No.98) Hence option c is correct. Question 28 of 35 28. Question Consider the following statements about Axolotl:  They are neotenic species of salamander. They are classified as critically endangered in the IUCN List. They remain underwater throughout their life. They are found in Mexico City’s Lake Xochimilco. How many of the above statements are correct? a) Only one b) Only two c) Only three d) All four Correct Solution (d) The axolotl’s name is derived from the Latin words “atl” (water) and “xolotl” (the Aztec god of fire and lightning, who could take on the form of a salamander). Axolotl are neotenic species of salamander. Hence statement 1 is correct. Salamanders are a group of amphibians. They are neotenic which means they keep juvenile characteristics into adulthood. They retain juvenile features such as gills, tails, and a preference for living in water. They are classified as critically endangered in the IUCN List. Hence statement 2 is correct. The population density of Mexican Axolotl has reduced by 99.5% in under two decades. They can regenerate lost or damaged limbs, hearts, lungs, and even parts of their brains. They remain underwater throughout their life. Hence statement 3 is correct. Axolotl is a cultural icon in Mexico for their unique slimy appearance and for their ability to regrow limbs. They are found in Mexico City’s Lake Xochimilco. Hence statement 4 is correct. Incorrect Solution (d) The axolotl’s name is derived from the Latin words “atl” (water) and “xolotl” (the Aztec god of fire and lightning, who could take on the form of a salamander). Axolotl are neotenic species of salamander. Hence statement 1 is correct. Salamanders are a group of amphibians. They are neotenic which means they keep juvenile characteristics into adulthood. They retain juvenile features such as gills, tails, and a preference for living in water. They are classified as critically endangered in the IUCN List. Hence statement 2 is correct. The population density of Mexican Axolotl has reduced by 99.5% in under two decades. They can regenerate lost or damaged limbs, hearts, lungs, and even parts of their brains. They remain underwater throughout their life. Hence statement 3 is correct. Axolotl is a cultural icon in Mexico for their unique slimy appearance and for their ability to regrow limbs. They are found in Mexico City’s Lake Xochimilco. Hence statement 4 is correct. Question 29 of 35 29. Question Consider the following statements regarding the key findings of UNEP’s Emissions Gap Report 2023: The world is heading towards a 4°C temperature rise by 2100, compared to pre-industrial levels. This is the fourteenth Emissions Gap Report. To limit global warming to 2°C, emissions must be cut by 28% by 2030. How many of the above statements are correct? a) Only one b) Only two c) All three d) None Correct Solution (b) The key findings of UNEP’s Emissions Gap Report 2023: The world is heading towards a 3°C temperature rise by 2100, compared to pre-industrial levels. Hence statement 1 is incorrect. Global emissions rose by 1.2% in 2022 from 2021, nearly returning to pre-pandemic levels. This fourteenth Emissions Gap Report is published ahead of the twenty-eighth session of the Conference of the Parties to the United Nations Framework on Climate Change (COP 28). Hence statement 2 is correct. The 2016 edition of the report had projected a warming of up to 3.4 degrees Celsius in a business-as-usual scenario. However, according to the Emissions Gap Report 2023, the current projection for the increase in emissions by 2030 is about 3%. Still, this is not enough to meet the 1.5°C target. To limit global warming to 2°C, emissions must be cut by 28% by 2030 and by 42% by 2030 for the 1.5°C target. Hence statement 3 is correct. Incorrect Solution (b) The key findings of UNEP’s Emissions Gap Report 2023: The world is heading towards a 3°C temperature rise by 2100, compared to pre-industrial levels. Hence statement 1 is incorrect. Global emissions rose by 1.2% in 2022 from 2021, nearly returning to pre-pandemic levels. This fourteenth Emissions Gap Report is published ahead of the twenty-eighth session of the Conference of the Parties to the United Nations Framework on Climate Change (COP 28). Hence statement 2 is correct. The 2016 edition of the report had projected a warming of up to 3.4 degrees Celsius in a business-as-usual scenario. However, according to the Emissions Gap Report 2023, the current projection for the increase in emissions by 2030 is about 3%. Still, this is not enough to meet the 1.5°C target. To limit global warming to 2°C, emissions must be cut by 28% by 2030 and by 42% by 2030 for the 1.5°C target. Hence statement 3 is correct. Question 30 of 35 30. Question Consider the following statements about the Amplifi 2.0 Portal: It was launched by the Ministry of Housing and Urban Affairs. It aims to make raw data from Indian cities available on a single platform for academics, researchers, and stakeholders to help data-driven policymaking. Choose the correct code: a) 1 only b) 2 only c) Both 1 and 2 d) Neither 1 nor 2 Correct Solution (c) The Amplifi 2.0 Portal was launched by the Ministry of Housing and Urban Affairs. Hence statement 1 is correct. Amplifi – Assessment and Monitoring Platform for Liveable, Inclusive, and Future-Ready Urban India portal. The portal provides data on various services for several cities such as: Total diesel consumption Number of water quality samples tested Average annual healthcare expenditure Total population residing in slums Recorded fatalities from road accidents It aims to make raw data from Indian cities available on a single platform for academics, researchers, and stakeholders to help data-driven policymaking. Hence statement 2 is correct. Currently, 258 urban local bodies have been onboarded and data for 150 cities is accessible on the platform. Eventually, data from the more than 4,000 Urban Local Bodies will be made available on the portal.   Note: According to the 2011 Census, 37.7 crore people (31% of the population) lived in urban areas. By 2030, this is projected to increase by 60 crores or 40%. In 2011, urban areas contributed 63% to the country’s GDP and this is projected to increase to 75% by 2030.   Incorrect Solution (c) The Amplifi 2.0 Portal was launched by the Ministry of Housing and Urban Affairs. Hence statement 1 is correct. Amplifi – Assessment and Monitoring Platform for Liveable, Inclusive, and Future-Ready Urban India portal. The portal provides data on various services for several cities such as: Total diesel consumption Number of water quality samples tested Average annual healthcare expenditure Total population residing in slums Recorded fatalities from road accidents It aims to make raw data from Indian cities available on a single platform for academics, researchers, and stakeholders to help data-driven policymaking. Hence statement 2 is correct. Currently, 258 urban local bodies have been onboarded and data for 150 cities is accessible on the platform. Eventually, data from the more than 4,000 Urban Local Bodies will be made available on the portal.   Note: According to the 2011 Census, 37.7 crore people (31% of the population) lived in urban areas. By 2030, this is projected to increase by 60 crores or 40%. In 2011, urban areas contributed 63% to the country’s GDP and this is projected to increase to 75% by 2030.   Question 31 of 35 31. Question Two joggers left Delhi for Noida simultaneously. The first jogger stopped 42min later when he was 1 km short of Noida and the other one stopped 52 min later when he was 2 km short of Noida. If the first jogger jogged as many kilometres as the second, and the second as kilometres as first, the first one would need 17min less than the second. Find the distance between Delhi and Noida? a) 5 km b) 15 km c) 25 km d) 35 km Correct Solution (b) Let x and y be speeds of two joggers (km/min) and d be the distance. According to given data, 42x + 1 = d …(1) 52y + 2 = d …(2) d/y – d/x = 17 …(3) Solving (1), (2) & (3), x = 1/3 km/min y = 1/4 km/min d = 15 km Distance between Delhi and Noida is 15 km. Incorrect Solution (b) Let x and y be speeds of two joggers (km/min) and d be the distance. According to given data, 42x + 1 = d …(1) 52y + 2 = d …(2) d/y – d/x = 17 …(3) Solving (1), (2) & (3), x = 1/3 km/min y = 1/4 km/min d = 15 km Distance between Delhi and Noida is 15 km. Question 32 of 35 32. Question Distance between A and B is 72 km. Two men started walking from A and B at the same time towards each other. The person who started from A travelled uniformly with average speed of 4 km/hr. The other man travelled with varying speed as follows: In the first hour his speed 2 km/hr, in the second hour it was 2.5 km/hr, in the third hour it was 3 km/hr, and so on. When/ Where will they meet each other? a) 7 hours after starting b) 10 hours after starting c) Mid-way between A and B d) 35 km from A Correct Solution (c) In the first hour, they cover 6km. In the second hour they cover 6.5km. In the third hour, they cover 7km and so on. Finally, they cover 72km in 9 hours. Distance covered by A in 9 hours = 4*9 = 36kms They meet mid-way between A and B. Incorrect Solution (c) In the first hour, they cover 6km. In the second hour they cover 6.5km. In the third hour, they cover 7km and so on. Finally, they cover 72km in 9 hours. Distance covered by A in 9 hours = 4*9 = 36kms They meet mid-way between A and B. Question 33 of 35 33. Question Of the 1300 candidates, who were interviewed for a position at a Bank, 900 had a Car, 630 had a motorcycle and 690 had a mobile phone. 300 of them had both, a Car and a motorcycle, 230 had both, a motorcycle and a mobile phone and 360 had both, a Car and mobile phone and 100 had all three. How many candidates had none of the three? a) 100 b) 170 c) 70 d) 230 Correct Solution (b) Number of candidates who had none of the three = Total number of candidates – the number of candidates who had at least one of three devices. The total number of candidates = 1300. We know that, Total= A + B + C – {A ∩ B + B ∩ C + C ∩ A + 2(A ∩ B ∩ C)} + neither Therefore, 1300 = 900 + 630 + 690 – {300 + 360 + 230 +200} + neither => Neither = [1300 −(900 + 630 +690) + (300 + 360 + 230 +200)] The number of candidates having neither = 170 Incorrect Solution (b) Number of candidates who had none of the three = Total number of candidates – the number of candidates who had at least one of three devices. The total number of candidates = 1300. We know that, Total= A + B + C – {A ∩ B + B ∩ C + C ∩ A + 2(A ∩ B ∩ C)} + neither Therefore, 1300 = 900 + 630 + 690 – {300 + 360 + 230 +200} + neither => Neither = [1300 −(900 + 630 +690) + (300 + 360 + 230 +200)] The number of candidates having neither = 170 Question 34 of 35 34. Question A certain number when divided by 247 leaves a remainder 17, another number when divided by 361 leaves a remainder 52. What is the remainder when the sum of these two numbers is divided by 19? a) 8 b) 9 c) 12 d) 17 Correct Solution (c) N1 = 247x + 17 and N2 = 361y + 52 N1 + N2 = (19 × 13 × x + 17) + (19 ×19 × y + 52) Remainder when N1 + N2 is divided by 19, = (17+52)/19 = 12 Incorrect Solution (c) N1 = 247x + 17 and N2 = 361y + 52 N1 + N2 = (19 × 13 × x + 17) + (19 ×19 × y + 52) Remainder when N1 + N2 is divided by 19, = (17+52)/19 = 12 Question 35 of 35 35. Question The average age of a family of 6 members is 20 years. If the age of the youngest member be 5 years, the average age of the family at the birth of the youngest member was? a) 19 yrs b) 22 yrs c) 18 yrs d) 21 yrs Correct Solution (c) Total present age of the family (6*20) = 120 years Total age of the family 5 years ago = (120 – 6*5) = 90 years At that time, Total members in the family = 5 Therefore Average age at that time = 90/5 = 18 years   Incorrect Solution (c) Total present age of the family (6*20) = 120 years Total age of the family 5 years ago = (120 – 6*5) = 90 years At that time, Total members in the family = 5 Therefore Average age at that time = 90/5 = 18 years   window.wpProQuizInitList = window.wpProQuizInitList || []; window.wpProQuizInitList.push({ id: '#wpProQuiz_3612', init: { quizId: 3612, mode: 1, globalPoints: 70, timelimit: 1800, resultsGrade: [0], bo: 704, qpp: 0, catPoints: [70], formPos: 0, lbn: "Test-summary", json: {"31670":{"type":"single","id":31670,"catId":0,"points":2,"correct":[0,1,0,0]},"31672":{"type":"single","id":31672,"catId":0,"points":2,"correct":[0,0,1,0]},"31674":{"type":"single","id":31674,"catId":0,"points":2,"correct":[0,0,1,0]},"31677":{"type":"single","id":31677,"catId":0,"points":2,"correct":[1,0,0,0]},"31679":{"type":"single","id":31679,"catId":0,"points":2,"correct":[0,1,0,0]},"31681":{"type":"single","id":31681,"catId":0,"points":2,"correct":[0,0,1,0]},"31684":{"type":"single","id":31684,"catId":0,"points":2,"correct":[0,0,1,0]},"31687":{"type":"single","id":31687,"catId":0,"points":2,"correct":[0,0,0,1]},"31688":{"type":"single","id":31688,"catId":0,"points":2,"correct":[0,0,1,0]},"31689":{"type":"single","id":31689,"catId":0,"points":2,"correct":[0,0,1,0]},"31692":{"type":"single","id":31692,"catId":0,"points":2,"correct":[0,1,0,0]},"31693":{"type":"single","id":31693,"catId":0,"points":2,"correct":[0,0,0,1]},"31696":{"type":"single","id":31696,"catId":0,"points":2,"correct":[1,0,0,0]},"31698":{"type":"single","id":31698,"catId":0,"points":2,"correct":[0,1,0,0]},"31700":{"type":"single","id":31700,"catId":0,"points":2,"correct":[0,0,1,0]},"31702":{"type":"single","id":31702,"catId":0,"points":2,"correct":[0,0,1,0]},"31705":{"type":"single","id":31705,"catId":0,"points":2,"correct":[0,0,1,0]},"31708":{"type":"single","id":31708,"catId":0,"points":2,"correct":[0,0,0,1]},"31711":{"type":"single","id":31711,"catId":0,"points":2,"correct":[0,1,0,0]},"31712":{"type":"single","id":31712,"catId":0,"points":2,"correct":[1,0,0,0]},"31713":{"type":"single","id":31713,"catId":0,"points":2,"correct":[0,0,1,0]},"31714":{"type":"single","id":31714,"catId":0,"points":2,"correct":[0,1,0,0]},"31717":{"type":"single","id":31717,"catId":0,"points":2,"correct":[0,1,0,0]},"31719":{"type":"single","id":31719,"catId":0,"points":2,"correct":[0,1,0,0]},"31721":{"type":"single","id":31721,"catId":0,"points":2,"correct":[0,1,0,0]},"31724":{"type":"single","id":31724,"catId":0,"points":2,"correct":[0,0,1,0]},"31727":{"type":"single","id":31727,"catId":0,"points":2,"correct":[0,0,1,0]},"31730":{"type":"single","id":31730,"catId":0,"points":2,"correct":[0,0,0,1]},"31733":{"type":"single","id":31733,"catId":0,"points":2,"correct":[0,1,0,0]},"31736":{"type":"single","id":31736,"catId":0,"points":2,"correct":[0,0,1,0]},"31739":{"type":"single","id":31739,"catId":0,"points":2,"correct":[0,1,0,0]},"31742":{"type":"single","id":31742,"catId":0,"points":2,"correct":[0,0,1,0]},"31743":{"type":"single","id":31743,"catId":0,"points":2,"correct":[0,1,0,0]},"31745":{"type":"single","id":31745,"catId":0,"points":2,"correct":[0,0,1,0]},"31748":{"type":"single","id":31748,"catId":0,"points":2,"correct":[0,0,1,0]}} } }); All the Best IASbaba